Anda di halaman 1dari 226

UNC Mathematics Contest

Answer Sheet – Final round February 15, 1997

1. The area of the large rectangle is 120 square inches.

2. The area of the hexagon is 60 square inches.

3. The number of toothpicks for an m by n rectangle is .

4. (a) is a positive integer when n= 2, 3, 5, 9.


(b) The largest value of n is 10.

5. (a) The next largest integer with eight positive divisors is 30.
(b) N=156.

6. (a) The number of ways for 10 stools is 56.

(b) The number of ways for n stools is .

7. The radius of circle P is .

8. The area of the trapezoid is 25.

9. The probability is .

10. (a)

(b)

11 (a) There are 171 ways of expressing 20 as a sum of three positive integers.

(b) There are ways of expressing 20 as a sum of three positive


Integers.

(c) There are ways of expressing n as a sum of four positive


integers.
UNIVERSITY OF NORTHERN COLORADO

MATHEMATICS CONTEST

First Round
For all Colorado Students Grades 7-12
November 4, 2006

• An arithmetic progression is a sequence of the form a, a+d, a+2d, …


• n factorial is computed as n!= n(n − 1)(n − 2) K 3 ⋅ 2 ⋅ 1 .
• An isosceles triangle has two sides with equal length.
• The positive odd integers are 1, 3, 5, 7, 9, 11, 13, … .

1. In the 4 by 4 square the boxes can be filled


in with each of the numbers 1, 2, 3, 4 used
exactly once in each row and column. What is A?

2. Determine the sum of the first 500 digits of the unending decimal expansion for
3
= .2307692 K .
13

3. The length of each leg of an isosceles triangle


is 2 x − 1 . If the base is 5 x − 7 determine
all possible integer values of x .

4. The odd number 7 can be expressed as 16 − 9 = 4 2 − 3 2 , a difference of two squares. Express each
of the following odd integers as the difference of two squares:
(a) 17 (b) 83

5. The three roots of the cubic equation 9 x 3 − 36 x 2 + cx − 16 = 0 are in arithmetic progression.


Determine the value of c and the three roots.

Over
6. The perimeter of the triangle is 24 in., and
its area is 8 in.2 . What is the exact
area of the inscribed circle? [That is, express
the area as a fractional multiple of π ].

7. Let f ( x) = 5 x 4 − 6 x 3 − 3 x 2 + 8 x + 2 . Determine coefficients a, b, c, d and e so that


f (x ) = a + b( x − 2) + c( x − 2) 2 + d ( x − 2) 3 + e( x − 2) 4 .

8. Determine all n for which x n + y n factors. [As a reminder, x 3 + y 3 factors as


( x + y )( x 2 − xy + y 2 ) , and x 4 − y 4 factors as ( x 2 − y 2 )( x 2 + y 2 ) ].

9. An International Conference on Global Warming has 5 diplomats from the US, 3 diplomats
from Russia and 4 diplomats from China. These 12 diplomats are to be seated at the head
table in a single row.

Determine the number of possible seating arrangements if the diplomats from each country must be
seated together as a group. Express your answer using the n! notation.

10. A square P1 P2 P3 P4 is drawn in the coordinate plane with P1


at (1,0) and P3 at (3,0) . Let Pn denote the point ( x n , y n ) .
Compute the numerical value of the following product
of complex numbers: ( x1 + iy1 )( x 2 + iy 2 )( x3 + iy 3 )( x 4 + iy 4 ) .

11. A quaternary “number” is an arrangement of digits, each of which is 0, 1, 2, 3.


Some examples: 12203, 01130, 22222, 00031
(a) How many 5-digit quaternary numbers are there?
(b) How many 5-digit quaternary numbers are there in which the digit 3 appears at least once?

In each case express your answer using exponents; you need not multiply out your answers.
BRIEF SOLUTIONS TO FIRST ROUND
NOVEMBER 2006

1. A = 3; by trial and error.

2. Sum = 2246 ; There are 83 blocks of 2 + 3 + 0 + 7 + 6 + 9 = 27 , with 2 + 3 left over.


83 × 27 + 5 = 2246 .
3. x = 2, 3, or 4; 2 x − 1 + 2 x − 1 > 5 x − 7 gives 5 > x . 2 x − 1 + 5 x − 7 > 2 x − 1 gives x > 1.

4. (a) 17 = 9 2 − 8 2 (b) 83 = 42 2 − 412

5. 2 3, 4 3, 6 3 and c = 44; 36 9 = 4 = (a + d ) + a + (a − d ) = 3a and a = 4 3 is one root.

( ) ( )
− 16 9 = (a + d )a(a − d ) = a a 2 − d 2 = 4 3 16 9 − d 2 and d = 2 3 . So the roots are a+d, a, a-d, or
6 3 , 4 3 , 2 3 . Substituting any root into the cubic gives c = 44 .
4 1 1 1
6. A = π ; label the sides as a, b, c. Then ar + br + cr = 8 , where r is the radius. Using a + b + c = 24 , you
9 2 2 2
2
r 2 2 4
have (a + b + c) = 8 or 24r = 16 , r = . Then A = π   = π   .
2 3 3 9
7. a = 38, b = 84, c = 81, d = 34, e = 5; Let x − 2 = y ; Then x = y + 2 and
4 3 2
f (x ) = f (y + 2 ) = 5(y + 2 ) − 6(y + 2 ) − 3(y + 2 ) + 8(y + 2 )+ 2 =
4 3 2
5(x − 2) + 34(x − 2) + 81(x − 2) + 84(x − 2)+ 38 =

5 y 4 + 34 y 3 + 81 y 2 + 64 y + 38 =
4 3 2
5(x − 2 ) + 34(x − 2 ) + 81(x − 2 ) + 84(x − 2 )+ 38

8. For all n having an odd factor; or n ≠ 2 k ; For example x 6 + y 6 = (x 2 + y 2 )(x 4 − x 2 y 2 + y 4 )


9. 6 ⋅ 5 ! ⋅ 3! ⋅ 4! ; The US diplomats can be arranged in a group in 5 ! ways. Same for the other groups.
But then the three groups can be permuted around in 3 ! = 6 ways.
10. 15; Label the points as P1 = 1 , P2 = 2 + i , P3 = 3 , P4 = 2 − i and multiply:

P1 P2 P3 P4 = 1 ⋅ (2 + i )⋅ 3 ⋅ (2 − i ) = 3(4 + 1) = 15 .

11. (a) 4 5 ; There are four choices for each of five spots.
(b) 4 5 − 35 ; Take all from part (a) and subtract those where the digit 3 fails to appear.
If there is no 3, there are 35 such “numbers”.
UNIVERSITY OF NORTHERN COLORADO
MATHEMATICS CONTEST
FINAL ROUND
For Colorado Students Grades 7-12
February 3, 2007
• The sequence of Fibonacci numbers is 1, 1, 2, 3, 5, 8, 13, 21, K .
• The positive odd integers are 1, 3, 5, 7, 9, 11, 13, … .
• A regular decagon is a 10-sided figure all of whose sides are congruent.
_________________________________________________________________________________

1. Express the following sum as a whole number:

1 + 2 − 3 + 4 + 5 − 6 + 7 + 8 − 9 + 10 + 11 − 12 + L + 2005 + 2006 − 2007 .

2. In Grants Pass, Oregon 4 5 of the men are married to 3 7 of the women. What fraction of the
adult population is married? Give a possible generalization.

3. State the general rule illustrated here and prove it:

4. If x is a primitive cube root of one (this means that x 3 = 1 but x ≠ 1 ) compute the value of
1 1
x 2006 + 2006 + x 2007 + 2007 .
x x

5. Ten different playing cards have the numbers


1, 1, 2, 2, 3, 3, 4, 4, 5, 5 written on them
as shown. Three cards are selected at random
without replacement. What is the
probability that the sum of the
numbers on the three cards is divisible by 7?

Over
6. (a) Demonstrate that every odd number 2n + 1 can be expressed as a difference of two squares.
(b) Demonstrate which even numbers can be expressed as a difference of two squares.

1 1 1
7. (a) Express the infinite sum S = 1 +
+ 2 + 3 + L as a reduced fraction.
3 3 3
1 1 2 3 5
(b) Express the infinite sum T = + + + + + L as a (reduced) fraction. Here
5 25 125 625 3125
the denominators are powers of 5 and the numerators 1, 1, 2, 3, 5, K are the Fibonacci numbers
Fn where Fn = Fn −1 + Fn − 2 .

8. A regular decagon P1 P2 P3 K P10 is drawn


in the coordinate plane with P1 at (2,0)
and P6 at (8,0). If Pn denotes the point
( x n , y n ), compute the numerical value of
the following product of complex numbers:
(x1 + i y1 )(x2 + iy 2 )(x3 + iy3 )L (x10 + i y10 )
where i = − 1 as usual.

9. A circle is inscribed in an equilateral triangle whose side


length is 2. Then another circle is inscribed externally
tangent to the first circle but inside the triangle as shown.
And then another, and another. If this process continues
forever what is the total area of all the circles? Express
your answer as an exact multiple of π (and not as a
decimal approximation).

10. A quaternary “number” is an arrangement of digits, each of which is 0, 1, 2, 3.


Some examples: 001, 3220, 022113.
(a) How many 6-digit quaternary numbers are there in which each of 0, 1 appear at least once?
(b) How many n-digit quaternary numbers are there in which each of 0, 1, 2, appear at least
once? Test your answer with n = 3 .
(c) Generalize.
Brief Solutions Final Round
February 3, 2007

1. 670,338; (1 + 2 − 3) + (4 + 5 − 6) + (7 + 8 − 9) + L + (2005 + 2006 − 2007)


= 0 + 3 + 6 + L + 3 ⋅ 668 = 3(1 + 2 + 3 + L + 668) = 3(668)(369) / 2

12 12 24
2. 24 ; Restated, of the men are married to of the women. Then of the adult population
43 15 28 43
is married. To generalize, if a b of the men are married to c d of the women, then ca cb of the
men are married to ca da of the women. The proportion that is married is 2ca (cb + da ).

3. 12 + 2 2 + 32 + L + n 2 = 1 ⋅ n + 3(n − 1)+ 5(n − 2)+ L + (2n − 1)⋅1 . The picture tells the story. For
example, the fourth diagram shows one 4, three 3’s, five 2’s and seven 1’s. Stripping off layers of
1’s also gives 12 + 2 2 + 32 + 4 2 .

2
3 2006 2 2007 2 1  1
4. +1; Since x = 1 , x = x and x = 1 . The expression becomes x + 2 + 2 =  x +  =
x  x
2
−x 2 2 1 2 x3 2
  = 1 since x + x + 1 = 0 . Or, x + 2 + 2 = x + 2 + 2 = x + x + 1 + 1 = 0 + 1 = 1 . Or,
 x  x x
1 1 1
x 2 + 2 + 2 = + x + 2 = −1 + 2 = 1 since the sum of the vectors x and is − 1 .
x x x

5. 2 15 ; There are five ways to achieve a sum divisible by 7; 115 (2 ways), 133 ( 2 ways), 124 (8
10 
ways), 233 (2 ways), 455 (2 ways). Hence, there are 16 favorable ways out of 120 =   total
3
choices.

6. (a) 2n + 1 = (n + 1)2 − n 2

(b) Multiples of 4; If x = 4n , 4n = (n + 1)2 − (n − 1)2 . The even numbers not divisible by 4, namely
2, 6, 10, 14, K cannot be expressed as a product of two even numbers and hence cannot be
expressed as (a + b )(a − b ) = a 2 − b 2 . If a, b are both even each of (a + b ), (a − b ) is even. If both
a, b are odd each of (a + b ), (a − b ) is even. If one is even, one odd both of (a + b ), (a − b ) are odd
and so is the product, handled in part (a).
1 1 1 1 1  1 3
7. (a) 3 2 ; S = 1 + + 2 + L = 1 + 1 + + 2 + L or S = 1+ S and S = .
3 3 3 3 3  3 2

1 2 3 5
5T = 1 + + 2
+ 3
+ + L
5 5 5 54
1 1 2 3
(b) 5 19 ; T = + + + + L
5 52 53 54
1 1 2 1
4T = 1 + 2
+ 3
+ 4
+ L = 1+ T
5 5 5 5

8. 9,706,576; Translate the center of the decagon to the origin. Now the vertices represent the roots
of f ( x) = x10 − 310 = 0 . Since the Pn are each 5 more than the roots of f ( x) = 0 , they would be
the roots of f (x − 5) = 0 or ( x − 5)10 − 310 = 0 . The product then is the constant term, or
510 − 310 = 9,706,576 .

3π 3
9. ; Let r1 , and r2 be radii of the first and second circles; r1 = and the area of the first circle is
8 3

1 3
A1 = π 3 . From r1 + r2 = 2(r1 − r2 ), r2 = 3 9 and A2 = π 27 . Similarly r3 = r2 = ,
3 27
1 1 1  π 3
A3 = π 243 . Then the total area = π  + + + L = = 3π 8 .
 3 27 243  1− 1
9

() ()
10. (a) 4 6 − 2 ⋅ 36 + 2 6 ; from the total 4 6 subtract those that have no 0 36 or no 1 36 . Then add
()
back in those that have no 0 and no 1 2 6 .
(b) 4 n − 3 ⋅ 3n + 3 ⋅ 2 n − 1n

(c) Generalize
UNIVERSITY OF NORTHERN COLORADO

MATHEMATICS CONTEST

First Round
For all Colorado Students Grades 7-12
November 3, 2007

• The positive integers are 1, 2, 3, 4, 5, 6, 7, 8, 9, 10, 11, 12, ….


• The Pythagorean Theorem says that a 2 + b 2 = c 2 where a, b, and c are side lengths of a right
triangle and c is the hypotenuse.
• An isosceles triangle has two sides with equal length.

1. Points A, B, C, and D are evenly spaced on


the number line as shown. What reduced
fraction corresponds to point C? Express
your answer as a fraction a b .

2. For which value(s) of n, are n, 3n+1, 3n-1 the lengths of the three sides of a right triangle?

3. Determine the number of triangles of all sizes in the shape shown.

4. Triangle ABC has integer side lengths.


One side has length 13 and a second
side is twice the length of the third
side. What is the greatest possible perimeter?

5. [4, 4, 5, 12] is a collection of positive integers whose sum is 25 and whose product is 960.
[2, 3, 4, 8, 8] is another collection whose sum is also 25 but whose product is 1536. Among all
collections of positive integers whose sum is 25 what is the largest product that can be found?
6. Six friends are sitting around a campfire. Each person in turn announces the total of the ages of the
other five people. If 104, 105, 108, 114, 115, and 119 gives the six sums of each group of five
people, what is the age of the oldest person?

7. ABCD is a rectangle with AB = 5 ,


EC = 2 , AE = AD . Find AD .

8. (a) Four people – Andrew, Beth, Carolyn and Darcey – play a game that requires them to split up
into two teams of two players. In how many ways can they split up?

(b) Now suppose that Euler and Fibonacci join these four people. In how many ways can these
six people split up into three teams of two?

9. Triangle ABC is isosceles with AB=AC.


_
The measure of angle BAD is 30 and AD=AE.
Determine the measure of angle EDC.

A : 1
10. In the diagram to the right
B : 3 5
(a) what is the sum of the entries in row J? C : 7 9 11
D : 13 15 17 19
(b) what is the first (leftmost) number in row Z? M
Y :
Z :
Brief Solutions
FIRST ROUND NOVEMBER 2007

71 7 1 17 17 1  17  71
1. ; Since − = , each segment has length . Then C is + 3  =
60 4 3 12 60 3  60  60

2. n = 12 ; By the Pythagorean Theorem (3n + 1)2 = (3n − 1)2 + n 2 . After simplification 12n = n 2 or

n = 12 .

3. 63; Any choice of 2 of the 7 base intersection points will determine a large triangle. Similarly for the other

two sections. The total number of triangles is then 3 7  = 3 ⋅ 21 = 63 .


 2

4. P = 49; The side lengths are 12, 24, and 13.

5. 8748; The sum 3 + 3 + 3 + 3 + 3 + 3 + 3 + 2 + 2 = 25 gives the maximum product. 5 is not needed as a

summand since it could be replaced by 2 + 3 , yielding a larger product 6. Similarly, since 6 = 3 + 3 ,

7 = 2 + 2 + 3 , 8 = 2 + 3 + 3 , 9 = 3 + 3 + 3 , none of these digits are needed.

6. Age = 29; The sum 104 + 105 + 108 + 114 + 115 + 119 = 665 is five times the sum of all six ages. Since

all six ages sum to 665 = 133 , the oldest is 133 − 104 = 29 .
5

7. 29 ; Let AD = x ; then AE = x , BE = x − 2 . By the Pythagorean Theorem x 2 = (x − 2 )2 + 25 ;


4

then 4 x = 29 .

8. (a) 3; The teams are AB and CD; AC and BD; AD and BC

(b) 15; With AB there are 3 ways to form 2 teams of 2 using C, D, E, F to pair with AB. Similarly,

with each of AC, AD, AE, AF there are three ways to complete the split. The total is

3 ⋅ 5 = 15 .

9. Angle EDC = 15_; Label all angles in the triangle and use two facts: the sum of all three angles in any of

the triangles is 180_ and the base angles in each of the two isosceles triangles are equal.

10. (a) The first number in row J is 91. The sum 91 + 93 + 95 + 97 + 99 + 101 + 103 + 105 + 107 + 109
is 1000. This can be found by subtracting one arithmetic sum from another.
(b) 651; Row Y has 25 elements in it. The first 25 rows contain 1 + 2 + 3 + L + 25 = 325 odd numbers.
The 325th odd number is 2(325)− 1 = 649 . Row Z starts with 651.
UNIVERSITY OF NORTHERN COLORADO
MATHEMATICS CONTEST
FINAL ROUND
For Colorado Students Grades 7-12
February 2, 2008

1. Determine the number of 3 by 3 square arrays


whose row and column sums are equal to 2, 1 1 0 1 0 1
using 0, 1, 2 as entries. Entries may be 0 0 2 1 1 0
repeated, and not all of 0, 1, 2 need be used as the 1 1 0 0 1 1
two examples show.

2. Let S = {a, b, c, d } be a set of four positive integers. If pairs of distinct elements of S


are added, the following six sums are obtained: 5, 10, 11, 13, 14, 19. Determine the values of a,
b, c, and d. [Hint: there are two possibilities.]

3. A rectangle is inscribed in a square


creating four isosceles right triangles.
If the total area of these four triangles
is 200, what is the length of the diagonal
of the rectangle?

4. In the figure there are 8 line segments drawn from


vertex A to the base BC (not counting the segments AB or AC).
(a) Determine the total number of triangles of all sizes.
(b) How many triangles are there if there are n lines
drawn from A to n interior points on BC?

5. The sum of 400, 3, 500, 800 and 305 is 2008 and the product of these five numbers is
146,400,000,000 = 1464 x10 8 .
(a) Determine the largest number which is the product of positive integers whose sum is 2008.
(b) Determine the largest number which is the product of positive integers whose sum is n.

OVER
6. Points A and B are on the same side of
line L in the plane. A is 5 units away
from L, B is 9 units away from L.
The distance between A and B is 12. For
all points P on L what is the smallest
value of the sum AP + PB of the distances
from A to P and from P to B?

7. Determine the value of a so that the following fraction reduces to a quotient of two linear
expressions:

x 3 + (a − 10 )x 2 − x + (a − 6 )
x 3 + (a − 6 )x 2 − x + (a − 10 )

8. Triangle ABC has integer side lengths.


One side is twice the length of a second side.
(a) If the third side has length 40 what is the greatest possible perimeter?
(b) If the third side has length n what is the greatest possible perimeter?
(c) Now suppose one side is three times the length of a second side and the third side has length
of 40. What is the maximum perimeter?
(d) Generalize.

9. Let C n = 1 + 10 + 10 2 + L + 10 n−1 .
(a) Prove that 9C n = 10 n − 1 .

(b) Prove that (3C3 + 2 )2 = 112225 .


(c) Prove that each term in the following sequence is a perfect square:
25, 1225, 112225, 11122225, 1111222225, …

10. Let f (n, 2 ) be the number of ways of splitting 2n people into n groups, each of size 2.
As an example, the 4 people A, B, C, D can be split into 3 groups: AB CD ; AC BD ;

and AD BC . Hence f (2, 2 ) = 3 .

(a) Compute f (3, 2 ) and f (4, 2).


(b) Conjecture a formula for f (n, 2 ).

(c) Let f (n, 3)be the number of ways of splitting {1, 2, 3, K , 3n} into n subsets of size 3.
Compute f (2, 3), f (3, 3) and conjecture a formula for f (n, 3).
Brief Solutions Final Round
February 2, 2008

1. 21; There are 6 using 3 2’s and 6 more using no 2’s, and 9 using one 2.

2. {1, 4, 9, 10} or {2, 3, 8, 11}; Order the integers as a < b < c < d. The two cases are a + b = 5 ,
a + c = 10 , a + d = 11 , b + c = 13 , b + d = 14 and c + d = 19 giving c − b = 5 by subtracting the
first two yielding {1, 4, 9, 10} or a + b = 5 , a + c = 10 , b + c = 11 , a + d = 13 , b + d = 14 ,
c + d = 19 yielding {2, 3, 8, 11}.

3. 20; Let a, b be the side lengths of the small and large right triangles, respectively. Then

a 2 + b 2 = 200 and the length of the diagonal is d = 2a 2 + 2b 2 = 2 200 = 20. There are two
alternative solutions: compress the rectangle into the diagonal d or expand the rectangle to become
a square whose diagonal is parallel to the side of the original square.

4. (a) 310  ; Any choice of 2 of the 10 lines will result in a triangle.


2
(b) 3 n +
2
2  = 3(n + 1)(n + 2 ) 2

 

5. (a) 3668 ⋅ 2 2 ; for any n the summands 5, 6, 7, 8, 9, can always be replaced by 3’s and
2’s. The maximum number of 3’s will yield the maximum product.
(b) If n = 3k , 3 k is the maximum product. If n = 3k + 1 , 2 2 ⋅ 3 k −1 is the max product. If
n = 3k + 2 , 2 ⋅ 3 k is the maximum product.

6. 18; Reflect PB about the x axis. The x coordinate of B′ , the reflection of B, is x = 12 2 − 4 2 =


128 ( )
. The distance between A (0, 5) and B′ 128 , − 9 is 128 + 196 = 18 .

7. a = 8 ; If c and d are negatives of each other then the numerator and denominator of

x 3 + cx 2 − x + d (x + c ) x2 −1 x+c
3 2
each factor by grouping as 2
= . So set a − 10 = − (a − 6 ) and
x + dx − x + c (x − c ) x −1 x−c
a =8.

8. (a) 157 ; If the side lengths are x, 2x and 40 then 40 + x > 2 x or x < 40 . The maximum
perimeter occurs when x = 39 and is 39 + 40 + 78 = 157.
(b) 4n − 3 ; If the sides are x, 2x, and n then x + n > 2 x implies x < n and the sides are
(n − 1), 2(n − 1), n .
(c) 116 ; x + 40 > 3 x implies x < 20 . Then 19 + 57 + 40 =116.
9. (a) 10 C n = 10 + 10 2 + 10 3 + L + 10 n ; now subtract C n = 1 + 10 + L + 10 n−1 .
2
(b) = (3 111+2)2 = 3352 = 112225.

(c) Take n = 4 ; (3C4 + 2)2 = 9C42 + 12C4 + 4 = 9C4C4+12C4+4

= (104 – 1)C4 + 12C4 + 4 = 104C4 + 10C4 + C4 + 4

= 11110000 + 11110 + 1111 + 4 = 11122225

The proof for general n is similar.

10. (a) f (3, 2) = 15 ; We can split A, B, C, D, E, F into 3 groups of 2 as follows; A can pair

with any of the other 5 in 5 ways. The other four can split into 2 in f (2, 2) = 3 ways.

f (4, 2) = 7 _ 5 _ 3 in a similar manner.

(b) f (n, 2) = (2n-1)(2n-3)(2n-5) 3 =

(c) f (2, 3) = ; A can pair with any 2 of B, C, D, E, F.

f (3, 3) = ; A can pair with any 2 of the other 8. The remaining 6 can be split in

f (2, 3) = ways.

f (n, 3) = =
UNIVERSITY OF NORTHERN COLORADO

MATHEMATICS CONTEST

First Round
For all Colorado Students Grades 7-12
November 1, 2008

• The positive integers are 1, 2, 3, 4, 5, 6, 7, 8, 9, 10, 11, 12, ….


• The Pythagorean Theorem says that a 2 + b 2 = c 2 where a, b, and c are side lengths of a right
triangle and c is the hypotenuse.
• A scalene triangle has three sides of unequal length.

1. A unit fraction is a proper fraction of the form 1/n where is an integer greater than 1. The
numerator is always 1. Examples: 1/3, 1/29, 1/100 Find two ways to write 4/5 as the sum of three
different unit fractions.

2. Insert all 11 integers 1, 2, 3, , 10, 11 into


this shape so that the sum of all the vertical
squares is 43 and the sum of all the horizontal
squares is 28. What number is in the corner?

3. Find a set of three different positive integers given that their product is 72 and that their sum is a
multiple of 7.

4. The area of the scalene triangle shown is 84 sq. units. Two side lengths are given as AB=10 and
AC=21. Determine the length of the third side BC.

OVER
5. Determine the area of the trapezoid.

6. An army of ants is organizing a peace march across a room. If they form columns of 8 ants there
are 4 left over. If they form columns of either 3 or 5 ants there are 2 left over. What is the smallest
number of ants that could be in this army?

7. Let . How many subsets of consisting of 8 (eight) different


elements are such that the sum of the eight elements is a multiple of 5?

8. Let denote the maximum number of points of intersection strictly between lines and
formed by joining the m points on to the points on in all possible ways. as
shown in the diagram.

(a) Compute

(b) Compute

(c) Give a formula for

9. (a) How many subsets of have the property that contains at least 2 elements and

no two elements of differ by 1? As an example, satisfies these two properties but

does not.

(b) Repeat with the set {1, 2, 3, 4, 5, 6}.


Brief Solutions
First Round 2008

1. = +

2. n = 5; Let n be in the corner, x the sum of the non corner horizontal squares, and y the corner
vertical square. Then x + n = 28, y + n = 43; x + y + n = 1 + 2 + + 11= 66; solving gives
x = 23, n = 5.

3. Trial and error yields 1, 3, 24.

4. BC = 17; Let h be the altitude from B. Then and h = 8. Since 10 – 8 – 6 is a


Pythagorean triplet, the base AC could be expressed as the sum 21 = 6 + 15 but then BC is the
hypotenuse of a right triangle whose side lengths are 8 and 15. BC = 17.

5. 244; Drop altitudes to form a center rectangle and two triangles. The base could be expressed as
the sum 41 = 15 + 20 + 6, noting the potential for 17 –15 – 8 and a 10 – 8 – 6 right triangle. The
area is then 60 + 160 + 24 = 244.

6. 92; Let n be the number of ants. Then n = 8a + 4 = 3b + 2 = 5c + 2. Then 8a + 2 is both a multiple


of 3 and 5, hence of 15. Trying even k for 8a + 2 = 15k gives us k = 6, a = 11.

7. 9; Since 1 + 2 + 3 + + 10 = 55, the sum of all the integers in S is a multiples of 5. Then the
complement of the subset consisting of 8 elements must also be a multiple of 5. These doubletons
are easier to count: 1,4 1,9 2,3 2,8 3,7 4,6 5,10 6,9 and 7,8.

8. (a) f (3, 3) = 9

(b) f (3, 4) = 18

(c) f (2, n) = n (n –1) / 2

A point of intersection is determined by the intersection of two lines. Any choice of 2 points
on along with 2 points on will produce such an intersection point. Hence for (c) there are
points.

9. (a) 7; They are 13, 14, 15, 24, 25, 35 and 135 (without using set notation).
(b) 14; 10 doubletons and the 4 triplets 135, 136, 146, 246
University of Northern Colorado

MATHEMATICS CONTEST
FINAL ROUND 2009

For Colorado Students Grades 7-12

1. How many positive 3-digit numbers abc are there such that ? For example, 202 and 178
have this property but 245 and 317 do not.

2. (a) Let 1 2 . For how many n between 1 and 100 inclusive is a multiple of 5?

(b) For how many n between 1 and 100 inclusive is 1 2 3 4 a multiple of 5?

3. An army of ants is organizing a march to the Obama inauguration. If they form columns of 10 ants
there are 8 left over. If they form columns of 7, 11 or 13 ants there are 2 left over. What is the
smallest number of ants that could be in the army?

4. How many perfect squares are divisors of the product 1! · 2! · 3! · 4! · 5! · 6! · 7! · 8! ? (Here, for
example, 4! means 4 · 3 · 2 · 1.)

5. The two large isosceles right triangles are congruent.


If the area of the inscribed square A is 225 square
units, what is the area of the inscribed square B?

6. Let each of m distinct points on the positive


x-axis be joined to each of n distinct points on
the positive y-axis. Assume no three segments
are concurrent (except at the axes). Obtain
with proof a formula for the number of interior
intersection points. The diagram shows that
the answer is 3 when 3 and 2.

OVER
7. A polynomial has a remainder of 4 when divided by 2 and a remainder of 14 when divided
by 3. What is the remainder when is divided by 2 3 ?

8. Two diagonals are drawn in the trapezoid


forming four triangles. The areas of two of the
triangles are 9 and 25 as shown. What is the total
area of the trapezoid?

9. A square is divided into three pieces of


equal area by two parallel lines as shown.
If the distance between the two parallel
lines is 8 what is the area of the square?

10. Let 1, 2, 3, … , . Determine the number of subsets A of such that A contains at least two
elements and such that no two elements of A differ by 1 when

(a) 10 (b) 20 (c) generalize for any .

11. If the following triangular array of numbers is continued using the pattern established, how many
numbers (not how many digits) would there be in the 100th row? As an example, the 5th row has 11
numbers. Use exponent notation to express your answer.

1
2
3 4 5
6 7 8 9 10
11 12 13 14 15 16 17 18 19 20 21
22 23 24 25 26 27 28 29 30 31 32 33 34 35 36 37 38 39 40 41 42
Brief Solutions Final Round
January 31, 2009

1. 45; Count them as you let c range from 1 to 9. There are 1 2 3 9 45.

2. (a) 25; From the partial units digit table, is a multiple of 5 only when 2 4 , where ranges from 0 to 24.
There are 25.
n 1 2 3 4 5 6 7 8
1 1 1 1 1 1 1 1 1
2 2 4 8 6 2 4 8 6

(b) 75; is a multiple of 5 except when 4 , 1, …, 25. See table. There are 75 such .
1 2 3 4 5 6 7 8
1 1 1 1 1 1 1 1 1
2 2 4 8 6 2 4 8 6
3 3 9 7 1 3 9 7 1
4 4 6 4 6 4 6 4 6
10 20 20 14 10 20 20 14 …

3. 6008; Ants = 10 8 7 2 11 2 13 2. Then 10 6 must be a multiple of 7, 11, and 13;


then 10 6 1001 . The smallest that works is 6.

4. 360; 1! · 2! · 3! · 4! · 5! · 6! · 7! · 8! · 2 · 3 · 5 · 7 . A perfect square divisor must be of the form 2 · 3 · 5 · 7


where a, b, c, d are even integers satisfying 0 23 , 0 9, 0 4, 0 2. There are 12 choices for
a (they are 0, 2, 4, 6, 8, 10, 12, 14, 16, 18, 20, 22), 5 choices for b, 3 choices for c and 2 for d.

5. 200; If 225, the large right triangle has area 450. Then 450 and 200.

6. 2 2 ; Select 2 of the m points and 2 of the n points; connect with two lines (twist them to make a point). Each
point of intersection corresponds to the crossing of the two lines.

7. 2 8; Since the remainder must be linear 2 3 . Then 2 4 2


and 3 3 14 gives 2 and 8.

8. 64; Since area ABD = area ACD, X 25 Y 25 and X Y.


Since ABE and EBC have the same height h, X, 9

and hence X⁄9 ⁄ . Similarly, 25⁄X ⁄ and so


X 9 · 25, X 3·5 15. Total area 15 9 15 25 64.
9. 832; implies a . Then 8 8√ 8 √13

must equal . So 8√13 and the area of the square is 64 · 13 832.

10. (a) 133; Try this with 7 first, and organize by size of subset.
Size 2 3 4 . The total is 6 5 4 . For 8 the chart is:
2 3 4
# 15 10 1

Size 2 3 4
# 21 20 5 for a total of 7 6 5 . For 10 the total is
2 3 4

9 8 7 6 133
2 3 4 5

(b) 19 18 17 11
2 3 4 10

(c) 1 2 3
2 3 4

10. Alternate Solution


There is a one-to-one correspondence between the desired type of subsets of S consisting of k elements and the
number of ways of choosing k out of 10 1 objects. Each desired k-element subset matches with a way of
choosing k out of a sequence of 10 objects so that no two are adjacent. The following picture illustrates this with
3: ○●○●○○○○●○ 2,4,9
Delete 1 2 objects, one from each gap, to form ○●●○○○●○
In reverse ●●●○○○○○ corresponds to 1,3,5 . Any of the 8 ways of selecting 3 objects from these 8 corresponds to
3
a desired subset.

11. 2 1 ⁄3; The center element in row is 2 . The following chart helps establish the pattern:

Row 2 3 4 5 6
# of elements 2 1 2 1 2 3 2 5 2 11

Note that 2 11 2 2 5 2 2 2 3 2 2 2 2 2 1

So row 100 contains 2 2 2 2 2 1 2 1 ⁄3 numbers, by summing the geometric sum.


UNIVERSITY OF NORTHERN COLORADO

MATHEMATICS CONTEST

First Round
For all Colorado Students Grades 7-12
October 31, 2009
You have 90 minutes – no calculators allowed

• The average of n numbers is their sum divided by n.


• A regular hexagon has all six sides with equal length.
• The positive integers are 1, 2, 3, 4, 5, 6, 7, 8, 9, 10, 11, 12, 13, 14, …

1. The average of the integers a and b is 22. The average of b and 30 is . What is the average of a
and c?

2. In the 3 x 3 grid of squares shown, three marbles


are randomly placed in different squares.
Express the probability that no two marbles
lie in the same row or column as a fraction .

3. The 5th term of the sequence 2, 3, 5, 8, 12, 17, 23, 30, … is 12.
(a) What is the 64th term?
(b) Which term in the sequence is 3162?

4. Express the sum as a fraction . (Hint: Notice, for example, that


).

5. Two equilateral triangles overlap to form a star.


The shaded overlap region is a regular hexagon.
If the area of the hexagon is 72 sq. in., what is
the area of one of the original (large) equilateral triangles?

Over
6. Simplify the product:

To be explicit, find integers a, b and c so that your answer is in the form . [Hint: Use
].

7. The area of the square is 16. Four identical


circles fit tightly inside the square. What is the
radius of the largest circle that will fit
inside the central space?

8. Lucille was asked to compute the product where a, b and c represent three different positive
integers. Lucky Lucille mistakenly thought that (a four digit number) but her answer
was correct. What were the integers a, b and c so that ? {Caution: is the
product of the two numbers and , whereas abca represents a 4 digit number, whose first and
last digits are the same.}

9. The chart shows the number of students for Math 18


four different majors in the class of 2010 at Biology 25
CNU, a local university. Business 42
(a) What percentage (as a fraction) are either Music 15
business or music majors?
(b) How many math majors would have to switch to music so that there would be a total of 25%
music majors?
(c) How many more biology majors would have to join these 100 students so that the percentage
of biology majors would be 40%?
Brief Solutions – First Round

1. 37; gives . gives . Then

2. ; Place the first one anywhere. Then there are four favorable spots out of eight for the
second marble and then just one spot out of seven for the third. The probability is
.

3. (a) 2018; The -th term is . For , .


(b) ; Solve to first obtain . Since and
are consecutive integers around 80 ending in 4 and 5, 5 and 6 or a zero, trial and error
shows .

4. ;
after you notice the collapsing sum.

5. 108; Triangulate the hexagon to form six equilateral triangles, each having area .
Then the area of one large triangle is .

6. ; Multiply by and observe the collapsing, or absorbing:

7. ; The radius of each small circle is 1. Let be the diameter of the sought after
center circle. Then , using the Pythagorean theorem,
and .

8. ; Trial and error. Start with since won’t be productive.


Make a column of powers of 2 and a column of squares and see which product ends in a
2. You will find that . Since you can only have a 4 digit number, the sizes
of the powers are limited.

9. (a) 57%, (b) 10, (c) 25; For (a), . For (b), you must have
of 100. For (c), solve .
University of Northern Colorado

MATHEMATICS CONTEST
FINAL ROUND January 30, 2010
For Colorado Students Grades 7-12

• The ten digits are 0, 1, 2, 3, 4, 5, 6, 7, 8, 9.


• The positive integers are 1, 2, 3, 4, 5, 6, 7, 8, 9, 10, 11, 12, 13, 14, …
• The prime numbers are 2, 3, 5, 7, 11, 13, 17, 19, 23, 29, …

1. Find a 3-digit integer less than 200 where each digit is odd and the sum of the cubes of the digits is
the original number.

2. The rectangle has dimensions 67 by 75.


The diagonal AB is divided into five
segments of equal length. Find the
total area of the shaded regions.

3. Suppose r, s, and t are three different positive integers and that their product is 48, i.e., .
What is the smallest possible value of the sum ?

4. Factor completely.

5. (a) In the 4 by 4 grid shown, four coins are


randomly placed in different squares. What is
the probability that no two coins lie
in the same row or column?

(b) Generalize this to an n by n grid.

OVER
6. A is a 4-digit number abcd. B is a 5-digit number formed by augmenting A with a 3 on the right, i.e.,
. C is another 5-digit number formed by placing a 2 on the left A, i.e., . If B is
three times C, what is the number A?

7. P and Q are each 2-digit prime numbers (like 73 and 19), and all four digits are different. The sum
is a 2-digit number made up of two more different digits ( is not necessarily prime).
Further, the difference consists of yet two more different digits (again, is not
necessarily prime). The number R is a two digit prime number which uses the remaining two digits.
What is R?

8. Simplify , using exponential notation to


express your answer. Generalize this result.

9. (a) Find integers A, B, and C so that . Express your answers in exponential form.
(b) Find integers A, B, C and D so that .

10. Let where . What is the maximum number of elements in a subset A of S,


which has at least three elements, such that for all a, b, c in A? As an example, the subset
of has the property that the sum of any two elements is strictly bigger
than the third element, but the subset does not since is not greater than 5. Since there
is no subset of size 4 satisfying these conditions, the answer for is 3.

11. (a) The 3 by 3 square grid has 9 dots equally


spaced. How many squares (of all sizes)
can you make using four of these dots
as vertices? Two examples are shown.
(b) How many for a 4 by 4?
(c) How many for a 5 by 5?
(d) How many for an by ( grid of dots?
Brief Solutions Final Round
2010

1. 153; Proceed by trial and error, using only 1, 3, 5, 7, and 9 as choices.


2. 2010; Each of the 10 triangles has the same area since they each have the same base and the same altitude. The
shaded area is then .

3. 12; Since the factors of 48 are 1, 2, 3, 4, 6, 8, 12, 16, 24 and 48. Trial and error
shows that is the smallest sum.
4. ; Factor by grouping;
. Alternatively, notice that is a zero of the
polynomial.
5. (a) ; The first coin can go anywhere, the second in any 9 of the remaining 15 squares, the third is any 4 of the

remaining 14, and the last one in just 1 of the remaining 13 squares. The probability is

(b) where

; or

6. 8571; and Since B is three times C, and


yielding .
7. ; Examination of the 21 two digit prime numbers will show that ,
and finally .
8. ; Multiply the expression by and observe the telescoping effect.

9. ; The choice of the base 2 is natural since the two terms on the left in
must combine into one term. Then . Alternatively , let
. Then gives Solving
for integer values gives the desired result. Other answers:
(b) ; Again, the choice of the base 3 is natural. Or:
10. if n is even, if n is odd. Trial and error will confirm the table of data:
n 4 5 6 7 8 9 10 11
3 3 4 4 5 5 6 6
Proof is left to reader.
11. (a) 6
(b) ; There are 9 with size 1 x 1, 4 with 2 x 2, and 1 with 3 x 3. But
each 2 x 2 contains a square whose side length is ; each 3 x 3 contains 2 squares with side

(c) (Hint: In each 3 x 3 square there are two squares with


side )
(d)
UNIVERSITY OF NORTHERN COLORADO

MATHEMATICS CONTEST

First Round
For all Colorado Students Grades 7-12
November 6, 2010
You have 90 minutes – no calculators allowed

• An arithmetic sequence with common difference d and first term a is:


!, ! + !, ! + 2!, ! + 3!, …
• The perfect squares are 1, 4, 9, 16, 25, 36, 49, 64, 81, 100, 121, …

1. There are five “lines” of three circles each


in the diagram. Insert the numbers
1, 2, 3, 4, 5, 6, 7, one in each circle, so
that the sum of the three numbers
in each line is the same. Which
number must be in the top most circle?

2. Five of the ten discs are dark and five are light;

and they alternate as shown in the pattern. If you are only allowed to interchange the positions of
the two neighboring discs in a single move, what is the least number of moves required to get all
the dark discs together at the right hand side?

3. ABCD is a square with side length 8.


E and F are midpoints of sides. ED and FC
are radii of quarter circles centered at D and
C respectively. EF is the diameter of the
upper semicircle. What is the area of the
shaded, ice cream cone, area?

4. The four numbers ! − 3, ! − 1, ! + 1, ! + 3 are in arithmetic progression with common difference


! = 2. Show that their product plus 16 is a perfect square. Specifically, find A (in terms of a) so
that ! − 3 ! − 1 ! + 1 ! + 3 + 16 = !! .
5. An isosceles trapezoid has sides
15, 28, 15, and 52 as shown. Find the
length of a diagonal.

6. A sequence of numbers is generated by the following initial condition and recursion:


!!
                                                                                       !! = 3 !!!! = !!!
!
(a) Find !!
(b) Find !!"#

7. Consider the arrangement (to the right) of the A B C D E


integers greater than 1. 2 3 4 5
(a) In which column will 100 fall? 9 8 7 6
(b) In which column will 1000 fall? 10 11 12 13
(c) In which column will 2011 fall? 17 16 15 14
⋮ ⋮ ⋮

8. Determine the 2010th term of the following sequence:

1, 2, 3, 6, 7, 14, 15, 30, 31, 62, 63, …

where a term in an even numbered spot is twice the previous term, and a term in an odd numbered
spot is one more than the previous term.

9. Five positive integers a, b, c, d, and e are written on the blackboard. When you add them four at
time the sums are 89, 99, 106, 110 and 112. What is the value of the smallest of the integers a, b, c,
d, e?

10. Let !(!) be the number of ways of selecting three distinct integers from 1,2,3, … , ! so that they
are the lengths of the sides of a triangle. For example, ! 5 = 3; the only possibilities are
2-3-4, 2-4-5, and 3-4-5. Find ! 7 and  ! 8 .
Brief Solutions – First Round
November 8, 2010

1. 4; Since the top number is used three times and the other six are used twice, the top number must be 4, the
middle number of the entries 1, 2, 3, 4, 5, 6, 7.
2. 15; Interchanging one at a time starting at the right requires 1 + 2 + 3 + 4 + 5 = 15   swaps.

3. 32; From the picture the shaded area is 2! + 2!


or half the area of the 8x8 square.

4. ! ! − 5; ! − 3 ! − 1 ! + 1 ! + 3 + 16 = ! ! − 9 ! ! − 1 + 16 = ! ! − 10! ! + 25 = ! ! − 5 !

5. 41; The altitude is 9.


The diagonal d is the
hypotenuse of the right
triangle whose legs are 9 and 40.
Then ! = 81 + 1600 = 1681 = 41

!! !!
6. (a) !! = = .
!!!! !
!! ! !! !! !
(b) To find !!"# , start generating the sequence.    !! = = −  , !! = = 3, !! = = − , the
!!!! ! !!!! !!!! !
!
same as !! . This sequence repeats in cycles of length three. Since 501 is a multiple of 3, !!"# = − .
!

7. (a) 100 is in column D, as part of the arithmetic subsequence 4, 12, 20, …


You can solve 4 + 8! = 100  for ! = 12.
(b) 1000 is in column B as part of the subsequence 8, 16, 24, 32, …
You can solve 8 + 8! = 1000 for ! = 124.
(c) 2011 is in column C; 3 + 4! = 2011 gives ! = 502.

8. 2!""# − 2; Make a table


Term # 1 3 5 7 9 11
Term 1 3 7 15 31 63 = 2! − 1
Since 2! − 1  is the 11th term, the 2011th term would be 2!""# − 1. Then 2!""# − 2 would be the 2010th
term.
9. 17; ! + ! + ! + ! = 89
! + ! + ! + ! = 99
! + ! + ! + ! = 106
! + ! + ! + ! = 110
! + ! + ! + ! = 112
Adding both sides, 4! + 4! + 4! + 4! + 4! = 516 and ! + ! + ! + ! + ! = 129. Since 112 is the

largest, ! = 129 − 112 = 17 is the smallest.

10. ! 7 = 13 and ! 8 = 22
University of Northern Colorado

MATHEMATICS CONTEST
FINAL ROUND January 29, 2011
For Colorado Students Grades 7-12

• !!, read as n factorial, is computed as !! = 1 ∙ 2 ∙ 3 ∙ 4 ∙∙∙ !


• The factorials are 1, 2, 6, 24, 120, 720, …
• The square integers are 1, 4, 9, 16, 25, 36, 49, 64, 81, …

1. The largest integer n so that 3! evenly divides 9! = 1 ∙ 2 ∙ 3 ∙ 4 ∙ 5 ∙ 6 ∙ 7 ∙ 8 ∙ 9 is ! = 4. Determine


the largest integer n so that 3! evenly divides 85! = 1 ∙ 2 ∙ 3 ∙ 4 ⋯ 84 ∙ 85.

2. Let m and n be positive integers. List all the integers in the set
20  ,21, 22, 23, 24, 25, 26, 27, 28, 29, 30, 31 that cannot be written in the form ! + ! + !".
As an example, 20 can be so expressed since 20 = 2 + 6 + 2 ∙ 6.

3. The two congruent rectangles shown have


dimensions 5 in. by 25 in. What is the area
of the shaded overlap region?

4. Let ! = 2, 5, 10, 17, ⋯ , !! + 1, ⋯ be the set of all positive squares plus 1 and
! = 101, 104, 109, 116, ⋯ , !! + 100, ⋯ be the set of all positive squares plus 100.
(a) What is the smallest number in both A and B?
(b) Find all numbers that are in both A and B.

5. Determine the area of the square ABCD,


with the given lengths along a zigzag
line connecting B and D.

OVER
6. What is the remainder when 1! + 2! + 3! + ⋯ + 2011! is divided by 18?

7. What is the sum of the first 999 terms of the sequence 1, 2, 3, 6, 7, 14, 15, 30, 31, 62, 63, ⋯ that appeared
on the First Round? Recall that a term in an even numbered position is twice the previous term, while a
term in an odd numbered position is one more that the previous term.

8. The integer 45 can be expressed as a sum of two squares as 45 = 3! + 6! .


(a) Express 74 as the sum of two squares.
(b) Express the product 45 ∙ 74 as the sum of two squares.
(c) Prove that the product of two sums of two squares, !! + ! ! ! ! + ! ! , can be represented
as the sum of two squares.

9. Let !(!) be the number of ways of selecting three distinct numbers from 1, 2, 3, ⋯ , ! so that they are
the lengths of the sides of a triangle. As an example, ! 5 = 3; the only possibilities are 2-3-4, 2-4-5,
and 3-4-5.
(a) Determine a recursion for !(!).
(b) Determine a closed formula for !(!).

10. The integers 1, 2, 3, ⋯ , 50 are written on the blackboard. Select any two, call them m and n and replace
these two with the one number ! + ! + !". Continue doing this until only one number remains and
explain, with proof, what happens. Also explain with proof what happens in general as you replace 50
with n. As an example, if you select 3 and 17 you replace them with  3 + 17 + 51 = 71. If you select 5
and 7, replace them with 47. You now have two 47’s in this case but that’s OK.

11. Tie breaker – Generalize problem #2, and prove your statement.
Brief Solutions Final Round
January 29, 2011

1. n=41; Look at 1 ∙ 2 ∙ 3 ∙ 4 ∙ 5 ∙ 6 ⋯ 81 ∙ 82 ∙ 83 ∙ 84 ∙ 85 closely. Every 3rd one is divisible by 3 (there are 28 of


these); every 9th one has an extra 3 (there are 9 of these) and every 27th one has another extra 3 (there are 3 of
these, namely 27, 54, and 81); and then add 1 for 81.

2. 22, 28 and 30; One way is to test them starting with 2 + 6 + 12 = 20.    Alternatively, notice that
! + ! + !" = ! + 1 ! + 1 − 1. Call this number Q. Then ! + 1 is not a prime since each of ! + 1, ! + 1
are greater than 2. So those that do not work are one less than a prime. The only primes in the given set are 23, 29
and 31.

3. 65; ! ! = 25 + 25 − ! !
= 650 − 50! + ! ! ;  
650 = 50!; ! = 13. Area=5x13=65

4. (a) 101
(b) 101, 325, 2501; !! + 1 = ! ! + 100 implies ! − ! ! + ! = 3 ∙ 3 ∙ 11. Match up factors:
! − ! = 1  !"#  ! + ! = 99  gives ! = 50, ! = 49  and  !! + 1 = 2501; ! − ! = 3, ! + ! = 33  
gives  ! = 18, ! = 15  and  !! + 1 = 325;  ! − ! = 9, ! + ! = 11  gives  ! = 10, ! = 1  and  !! + 1 = 101.

! !"
5. 160;
!
= !!! gives  ! = 3. Then !!! = 45, !!! = 125,  
!
d1+d2=8 5.  If  y  is  the  side  length,  ! ! + ! ! = 8 5  
and  ! ! = 160,  the  area.        
 
 
                                                                                                                                     
Alternate  solution  to  #5.  Extend  drawing  as  shown.    
Then  compute  the  diagonal  of  the  square  to  be  8 5  .  
 
6. 9; 6! + 7! + ⋯ + 2011! is divisible by 18. The remainder when 1! + 2! + 3! + 4! + 5! = 153 is divided
by 18 is 9.

7. 2!"# − 1504;  Split the sequence into two sequences: 1 + 3 + 7 + 15 + ⋯ + 2!"" − 1 and
2 + 6 + 14 + 30 + ⋯ + 2!"" − 2 . Then   2! − 1 + 2! − 1 + ⋯ + 2!"" − 1 = 2!"# − 502 and
2! − 2 + 2! − 2 + ⋯ + 2!"" − 2 = 2!"# − 1002. Now add to get
2!"# − 502 + 2!"# − 1002 = 2 ∙ 2!"# − 1504 = 2!"# − 1504
8. (a) 74 = 5! + 7!
(b) 45 ∙ 74 = 3! + 6! 5! + 7! = 3 ∙ 5 + 6 ∙ 7 !
+ 6∙5−3∙7 !
= 57! + 9! . Another correct answer;
51! + 27! .
(c) ! ! + ! ! ! ! + ! ! = !" + !" !
+ !" − !" !

9. (a) First collect data, carefully, and make a table:

n 3 4 5 6 7 8 9 10 11
T(n) 0 1 3 7 13 22 34 50 70

If you add two consecutive terms you get 1, 4, 10, 20, 35, 56, 84, 120, the entries on the 4th diagonal in
!
Pascal’s Triangle. Hence ! ! + 1 + ! ! =   is a recursion for ! ! .
3

!−1 !−2 !−3 !−4


(b) Iteration of the recursion gives ! ! = − + − +⋯
3 3 3 3
The use of generating functions will also show this.

Alternate recursions and formulas include:


!!! !
       !"#  !  !"!#
!
! ! =! !−1 + !!! !!!
 !"#  !  !""
!

!!! !!! !!!!  


! ! =    !"#  !  !""
!"
!!! !!! !!!! ! ! ! !!! !!!!
+ −1 =    !"#  !  !"!#
!" ! !"

!
! ! = ! − 2 ! − 3 + ! − 4 ! − 5 + ⋯ + 3 ∙ 2  !ℎ!"  !  !"  !""
!

!
= ! − 2 ! − 3 + ! − 4 ! − 5 ⋯ + 2 ∙ 1  !ℎ!"  !  !"  !"!#.
!

10. First try 1, 2, 3, … , !  for ! = 2, 3, 4, 5. The crossing off process yields 5,  23,  119,  719 each one being one less
than a factorial. So for general n you should end up with   ! + 1 ! − 1. Now look at n=3 again and replace 1, 2, 3
with a,  b,  c (order does not matter). Crossing off gives you
! + ! + !" + ! + ! + ! + !" ! = ! + ! + ! + !" + !" + !" + !"#, reminding one of the coefficients in
! − ! ! − ! ! − ! = ! ! − ! + ! + ! ! ! + !" + !" + !" ! − !"#. Now let ! = −1, and watch what
happens remember  that   !, !, ! = 1,2,3 .
There are other approaches.
11. See solution to #2. Integers that are one less than a prime cannot be written in the form ! + ! + !.
UNIVERSITY OF NORTHERN COLORADO

MATHEMATICS CONTEST

First Round
For all Colorado Students Grades 7-12
November 3-6, 2011
You have 90 minutes- no calculators allowed

• A regular hexagon has six sides with equal length and six angles with equal measure.
• The positive integers are 1, 2, 3, 4, 5, 6, 7, 8, 9, 10, 11, 12, …

1. A 4 x 9 cardboard rectangle is cut up and the pieces rearranged, without gaps or overlap, to
form a square. What is the perimeter of that square?

2. Solve for N: 23 x 54 x 72 = 250 x N


C
3. In triangle ABC, side AB has length 6, side BC has length 5, side AC
has length 7. Segment CD is perpendicular to AB and point D divides
segment AB into two pieces. What is the length of the longer piece?
A DB
4
4. The ones digit in the number 2 = 16 is 6.
a. What is the ones digit in the number 26 ?
b. What is the ones digit in the number 28 ?
c. What is the ones digit in the number 22011 ?

5. The hexagon ABCDEF has one internal angle greater F


than 180 degrees, angle BCD. What is the largest number of A
internal angles greater than 180 degrees that any single
E
hexagon can contain? B C D

B
6. Find the shortest distance from point A to point B, measured on the curved
surface of the cylinder. Segment PQ is a diameter of the circular base, and
the base has circumference 6 centimeters. Point A is 2 centimeters above
point P. Point B is 6 centimeters above point Q.

A
Over Q
P
7. A drawer contains 24 utensils: one knife, one fork, and one spoon, each in 8 different colors. If
you pull items at random from the drawer without looking, what is the smallest number of items
you must take to be certain to have pulled out a complete matching table setting, containing a
knife, fork, and spoon of the same color?

8. Find the product of all of the positive integers n that satisfy the following inequality.

n < 12 < n + 17 < 2n + 10 < n2 ! 51

9. Square Meal You want to eat a lump of cookie dough in stages. A cookie
press converts the dough into a square of uniform thickness. On day 1 you
divide the square into 4 equal smaller square pieces, using a 2x2 grid, then
eat one of these 4 pieces. On day 2 you press the remaining dough into a
new square, subdivide it using a 3x3 grid, and eat one of these 9 pieces.
Continue pressing, subdividing, and eating pieces of the remaining dough.
What fraction of the original lump remains immediately after the 100th
meal? Give your answer as a fraction c /d, expressed in lowest terms.

10. Treasure Chest You have a long row of boxes.


The 1st box contains no coin. The next 2 boxes each contain 1 coin. The next 4 boxes each
contain 2 coins, the next 8 boxes each contain 3 coins, the next 16 boxes each contain 4 coins,
and so on. (The number of boxes that contain N coins is twice the number of boxes that contain
(N-1) coins.)
(a) How many coins are in the 100th box?
(b) How many coins are there when the contents of the first 100 boxes are combined?

11. Hex Consider the sequence of honeycomb-shaped figures below. The first figure has
one cell and is made of 6 line segments. The second figure has 7 cells and is made
of 30 line segments. How many line segments are there in the 20th figure? (The next
page is a sheet of paper tiled in hexagons for your use in considering this problem.)

First figure Second figure Third figure


Solutions for First Round
Fall 2011

1. The perimeter of the square is 24. The area of the 4 × 9 cardboard rectangle is 36. However
the rectangle is cut up and arranged into a square, that square will also have area 36. For a
square to have area 36, the side length must be 6. The perimeter of a square with side length
6 is 4 × 6 = 24.

2. N = 980. If you factor 250, you get 2 × 53 . Dividing both sides of the equation by 2 × 53 gives
22 × 5 × 72 = N . Multiplying these out gives N = 2 × 10 × 49 = 2 × 490 = 980.

3. The length of the longer piece is 5. Let’s call the length of the longer piece (AD) x. Then the
length of the segment DB is 6 − x since the length of AB is given as 6. We also do not know
the length of the segment CD, so let’s call that y. To solve for x, we can use the Pythagorean
theorem on the right triangle ADC. So x2 +y 2 = 72 (AC is the hypotenuse of the triangle and is
given to have length 7). To find y, we note that CDB is also a right triangle, so (x−6)2 +y 2 = 52
(BC is the hypotenuse and is given to have length 5). We can rewrite this as y 2 = 52 − (x − 6)2
and then substitute in to the other equation and solve:

x2 + (52 − (x − 6)2 ) = 72
x2 + 25 − (x2 − 12x + 36) = 49
12x − 11 = 49
x=5

4. Check the last digit of small powers of 2: 21 = 2, 22 = 4, 23 = 8, 24 = 16, 25 = 32, 26 = 64,


27 = 128, 28 = 256,. . . . It appears the pattern among last digits is 2, 4, 8, 6, 2, 4, 8, 6, . . .. This
pattern repeats every 4 numbers. When 2n has an n divisible by 4, the last digit is a 6. Now
2008 is divisible by 4, so we expect 22008 to end in a 6, so 22009 will end in a 2, 22010 will end
in a 4, and most importantly 22011 will end in an 8.
(a) The ones digit of the 6th power is 4.
(b) The ones digit of the 8th power is 6
(c) The ones digit of the 2011th power is 8.

5. The maximum number of angles with measure greater than 180 is 3. For example:

You cannot have a larger number of such angles because the sum of the interior angles of any
hexagon is 4 × 180 = 720◦ , and if you had 4 angles greater than 180◦ , that would already be
over 720◦ .

6. The length of the path connecting A to B, in centimeters, is 5. Unroll the cylinder! doing so
with reveal the line connecting A to B to be the hypotenuse of a right triangle with base 3 (half
of the circumference) and height 4 (the difference between the height of B, 6, and the height
of A, 2).
7. The smallest number of items one must select is 17. You could pull out all the 8 knives, all the
8 spoons, and still not have a set. However, the next utensil you pull will need to be a fork,
and will match the colors of the utensils you have already pulled. Alternatively, for each of the
8 colors, you can pull 2 utensils of that color without having a set - that’s 16 items. The next
one will have to give you a matching table setting.

8. The product of the positive integer solutions is 990. We need n < 12 so the possibilities for n
are 1, 2, 3, . . . , 11. We also need n + 17 < 2n + 10, or equivalently 7 < n. So now n can only
be 8, 9, 10, or 11. Now simply plug in these possibilities to the inequality and see which ones
work:

n=8: 8 < 12 < 25 < 26 < 13 ×


n=9: 9 < 12 < 26 < 28 < 30 !
n = 10 : 10 < 12 < 27 < 30 < 49 !
n = 11 : 11 < 12 < 28 < 32 < 70 !

Thus the values of n which satisfy the equation are 9, 10, and 11, whose product is 990.

9. c = 51 and d = 101. After day one, 34 of your original lump remains. After day two, 89 of what
you had at the end of day one is left, so 34 89 of your original lump is left. Continuing, after the
100th day, the fraction of what you started with remaining is
! "! "! " ! 2 " ! "
3 8 15 n −1 1012 − 1
··· ···
4 9 16 n2 1012
2
−1
We stop at 101
1012
1
because on day n we eat (n+1) 2 of what we currently have. To simplify this
2
long product, note that n − 1 = (n − 1)(n + 1). So we can rewrite:

(1 · 3) (2 · 4) (3 · 5) (n − 1)(n + 1) (100 · 102)


··· ···
(2 · 2) (3 · 3) (4 · 4) n·n (101 · 101)

The product collapses - there is massive cancellation. All that remains is


1 · 102 51
=
2 · 101 101

10. Here is a table of which boxes have how many coins:


Box numbers: 1 2-3 4-7 8-15 16-31 32-63 64 - 100
Coins per box: 0 1 2 3 4 5 6
(a) The number of coins in the 100th box is 6.
(b) The number of coins in all the first 100 boxes together is 480. We need to compute 2 · 1 +
4 · 2 + 8 · 3 + 16 · 4 + 32 · 5 + 37 · 6.

11. The number of segments in the 20th figure is 3540. One approach is to cut the figures into six
equal “wedges.” Draw lines from the center of the middle hexagon through each of its corners
- these lines will lie on top of edges of hexagons every other ring. Now count the number of
line segments.
First, just consider the line segments parallel to the lines we used to cut up the figures. For
those coinciding with our cutting lines, we will count only the one of the left (so we don’t over
count). The first figure has none of these lines. The second figure has just one. The third has
2 more, the forth has three more (one on the edge of our wedge), and so on. So to count these
lines, we must find 1 + 2 + 3 + · · · + 19 = 190.
Now for the edges in each wedge not parallel to our dividing lines. The first figure has 1, the
second figure has 3 more, the third has 5 more, the fourth has 7 more, and so on. So to count
these line segments, we sum 1 + 3 + 5 + 7 + · · · + 39 = 40 · 20/2 = 400.
So the total number of line segments each wedge contributes is 590. But there are 6 wedges,
so the total number of line segments is 6 · 590 = 3540.
UNIVERSITY OF NORTHERN COLORADO
MATHEMATICS CONTEST
FINAL ROUND
For Colorado Students Grades 7-12
January 21, 2012

You have three hours. No calculators are allowed. Show your work for each problem
on pages behind your answer sheet. Your score will be based on your answers and
your written work, including derivations of formulas you are asked to provide.

• The positive integers are 1, 2, 3, 4, 5, 6, 7, 8, 9, 10, 11, 12, …


• An ordinary die is a cube whose six faces contain 1, 2, 3, 4, 5, and 6 dots.

1. (a) What is the largest factor of 180 that is not a multiple of 15?
(b) If satisfies , then what is the largest perfect square
that is a factor of ?

2. Four ordinary, six-sided, fair dice are tossed. What is the probability that the sum of the
numbers on top is 5?

3. Mrs. Olson begins a journey at the


intersection of Avenue A and First Street in
the upper left on the attached map. She ends
her journey at one of the Starbucks on
Avenue D. There is a Starbucks on Avenue
D at every intersection from First Street
through Sixth Street! If Mrs. Olson walks
only East and South, how many different
paths to a Starbucks on Avenue D can she
take? Note that Mrs. Olson may pass one
Starbucks on her way to another Starbucks
farther to the East.

4. (a) What is the largest integer for which is divisible by ?


(b) For how many positive integer values of is divisible by ?

5. What is the remainder when is divided by seven?

over
6. How many 5-digit positive integers have the property that the product of their digits is 600?

7. A circle of radius 1 is externally tangent to a circle of radius 3


and both circles are tangent to a line. Find the area of the shaded
region that lies between the two circles and the line.

8. An ordinary fair die is tossed repeatedly until the face with six dots appears on top. On
average, what is the sum of the numbers that appear on top before the six? For example, if
the numbers 3, 5, 2, 2, 6 are the numbers that appear, then the sum of the numbers before the
six appears is . Do not include the 6 in the sum.

9. Treas u re C h es t. You have a long row of boxes. The 1st box contains no coin. The next
2 boxes each contain 1 coin. The next 4 boxes each contain 2 coins. The next 8 boxes each
contain 3 coins. And so on, so that there are boxes containing exactly coins.
(a) If you combine the coins from all the boxes that contain 1, 2, 3, or 4 coins you get 98
coins. How many coins do you get when you combine the coins from all the boxes that
contain 1, 2, 3, …, or coins? Give a closed formula in terms of . That is, give a
formula that does not use ellipsis (…) or summation notation.
(b) Combine the coins from the first boxes. What is the smallest value of for which
the total number of coins exceeds 20120? (Remember to count the first box.)

10. An integer equiangular hexagon is a six-sided polygon whose side


lengths are all integers and whose internal angles all measure
120 degrees.
(a) How many distinct (i.e., noncongruent) integer equiangular
hexagons have no side length greater than 6? Two such hexagons
are shown.
(b) How many distinct integer equiangular hexagons have no side
greater than ? Give a closed formula in terms of .
(A figure and its mirror image are congruent and are not
considered distinct. Translations and rotations of one another are
also congruent and not distinct.)

11. Construct a 4th degree polynomial that meets as many of the


following conditions as you can: The sum of the roots is 1, the sum of the squares of the roots
is 2, the sum of the cubes of the roots is 3, and the sum of the 4th powers of the roots is 4.
UNC MATH CONTEST SOLUTIONS must approach its destination from a neighboring intersection
that is one block away either north or west. Therefore, the num-
FINAL ROUND JANUARY 2012 ber of paths to any intersection is the sum of the numbers of
paths to these two nearest neighboring intersections. This is ex-
actly the rule for generating Pascal’s triangle, as you can see in
(1.A.) 36 the diagram. (Not all path counting problems will be this regu-
lar.) Take the sum of all such path-numbers along the Starbucks
180= 2 x 2 x 3 x 3 x 5. Factors of 180 are produced by selecting on Avenue D: 1 + 4 + 10 + 20 + 35 + 56 = 126.
subsets of these prime factors: 2 x 2 = 4 and 2 x 3 x 5 = 30, for
instance. Any factor that is not a multiple of 15 must leave out
either the 5 or both of the 3s. The largest factor is found by
leaving out the 5. That factor is 2 x 2 x 3 x 3= 36. This question
and 1.B extend Problem 2 from the First Round.
(1.B) 196 = 142

Factor 99000 into prime factors, so that the equation becomes


25 × 32 × 54 × 73 × 11 = 23 × 32 × 53 × 11 × N. Cancel com-
mon factors and deduce 22 × 5 × 73 = N. The largest perfect
square factor of N is constructed by taking each prime factor
to the largest available even power. You get 22 × 72 = 142 =
196. F IGURE 1. Path-counting, step by step
(2.) 1/324 Solution (ii). (33) + (43) + (53) + (63) + (73) + (83) = 126. Justification.
Consider first a typical path, described as a list of eastward and
To get a sum of 5 you must get three 1s and one 2. There are four southward steps. The path SSEESEE goes to the Starbucks on
different ways to get this combination: 2111, 1211, 1121, and Fifth Street. All paths going to this Starbucks will have seven
1112. Each of these has probability 1/64 , so the total probability letters and exactly four of those letters will be E’s and three will
is 4/64 = 1/324. be S. The number of such paths is (73). In general, to reach the
(3.) 126 intersection of Avenue D and Nth Street, going only east and
south, a path must go N − 1 blocks east and 3 blocks south, or
Solution (i). Starting in the upper left corner, begin tabulating N + 2 blocks in all. Out of these N + 2 blocks, exactly 3 will be
the number of pathways that reach intermediate destinations south. The number of choices will be ( N3+2). Compute this for
on the grid. Each path that ends at an intermediate destination N = First, Second,. . ., Sixth Streets, and add.
1
2 FINAL ROUND JANUARY 2012

Solution (iii) Here is a solution that counts the paths to all the street numbers of the locations of the four knots in solution (iii)
Starbucks on Avenue D at once. Imagine a rope that follows with 1 ≤ W ≤ X ≤ Y ≤ Z ≤ 6. We want to count the number
Mrs. Olson’s path from the starting point at the intersection of choices we have for the ordered list W, X, Y, Z. Imagine a
of Avenue A and First Street to her destination Starbucks on row of rooms 1, 2, 3, 4, 5, and 6 with five walls dividing them:
Avenue D, and then continues eastward to the lowest right corner of
the diagram: the intersection of Avenue D and Sixth Street. The rope
must always travel eight blocks. Place a knot at each intersec- #1 #2 #3 #4 #5 #6
tion, including the start and end. No matter which path Mrs.
Olson takes, there will be exactly nine knots on the rope. There
are three special knots on the rope where Mrs. Olson makes the F IGURE 3. Rooms with walls
decision to head south to the next Avenue after passing through
an intersection, and there is a final fourth knot where she de- Think of the letters as balls that will be dropped into the rooms
cides to stop, somewhere on Avenue D. Mark these four special corresponding to their values. Several balls can go into a single
knots X. For any rope that has four of its nine knots marked X,
"#$%&'(&")*&+,-*.&/012&+,-*.&/31
room and some rooms may be empty. For the example above
there will be one and only one path Mrs. Olson can choose that with W = 1, X = 1, Y = 3, and Z = 5 we get
will correspond to this marking of the knots. Therefore, Mrs.
Olson has exactly (94)=126 paths from which to choose. oo o o
#1 #2 #3 #4 #5 #6
.:;&!; +(<&!; -=<&!; >;?&!; @;?&!; A;?&!;
F IGURE 4. Balls with walls
4
56%&5
We can use the shorthand o o | | o | | o |. Each solution
W, X, Y, Z corresponds to a string of nine symbols, four of which
56%&7 4 are balls o and five of which are walls |. Therefore, there are
(94)=126 solutions in all. That is, the number of ways to pick an
ordered list of four numbers in nondecreasing order W ≤ X ≤
56%&8 4
Y ≤ Z from the set S = {1, 2, 3, 4, 5, 6} is (94). More generally,
the number of ways to choose four numbers in nondecreasing
56%&9 4 order W ≤ X ≤ Y ≤ Z from the set S = {1, 2, 3, 4, 5, 6, . . . n}
!;BC
is (n+ 3
4 ). This counting technique is sometimes called ”balls and
walls”, ”stars and bars”, or ”sticks and stones.”
A handful of students interpreted the problem to say that Mrs.
F IGURE 2. Nine knots with four X’s Olson could pass at most one Starbucks on her way to her fi-
Solution (iv) Here is another solution that counts all the paths nal destination Starbucks. So interpreted, the problem becomes
at once. We discuss this solution at length because it will play somewhat harder; and full credit was given for correct solu-
a role in the solution of Problem 10. Let W, X, Y and Z be the tions to this variant interpretation, whose answer is 91 paths.
UNC MATH CONTEST SOLUTIONS 3

(4.A) 289 The table reveals that powers of 12 and 11 repeat with periods
of length 6 and 3 respectively. Thus each power can be reduced
Use long division to get by removing whole multiples of its period. Reduce the power
2011 ≡ 1 mod 6, and deduce 122011 ≡ 52011 ≡ 51 ≡ 5 mod 7.
(n3 + 1631) 300 Similarly, reduce 2012 ≡ 2 mod 3, and deduce 112012 ≡ 42012 ≡
= (n2 − 11n + 121) + .
(n + 11) (n + 11) 42 ≡ 2 mod 7 . Therefore 122011 + 112012 ≡ 5 + 2 = 7 ≡ 0 mod 7.
This problem is an extension of Problem 4 on the First Round.
The polynomial n2 − 11n + 121 is an integer for all integer val-
ues of n. The largest n for which (n300
+11)
is integer is the n that 6. 210
makes the denominator 300: n = 300 − 11 = 289.
Factoring into primes, 600 = 2 × 2 × 2 × 3 × 5 × 5. These six
(4.B) 11 primes must be placed in five digit places, and some primes
must share a place. Clearly both 5s must be alone, occupy-
Each factor m of 300 that satisfies m ≥ 12 will produce a posi- ing two places. Now use trial and error to place the other four
tive integer solution n = m − 11. Count all the factors of 300, primes in three places. The possible unordered lists of five dig-
and then discard the seven small factors 1, 2, 3, 4, 5, 6, 10 that are its are found to be
smaller than 12. To find all the factors of 300, use the prime fac-
(i) 2,3,4,5,5; (ii) 1,8,3,5,5; (iii) 1,4,6,5,5; and (iv) 2,2,6,5,5.
torization 300 = 22 × 31 × 52 . Each factor of 300 can be written
as 2r × 3s × 5t with r = 0, 1, 2; s = 0, 1; and t = 0, 1, 2. There Now count the possible orderings of the digits in each list. To
are 3 × 2 × 3 = 18 choices for these powers, hence 18 factors of count the orderings of list (i)=2,4,3,5,5, first choose two slots for
300. After discarding the seven factors that are too small, eleven the 5s: there are 10 ways. Then rearrange the remaining three
factors remain. digits 3!=6 different ways. Thus there are 10 × 6 = 60 different
5 digit numbers whose digits are 2, 3, 4, 5 and 5. Similarly there
(5.) 0 are 60 orderings of list (ii) and 60 orderings of list (iii). List (iv)
has only 10 × 3=30 distinct orderings, because of the repeated
Strategy: Look for cyclic patterns in the powers. This can be 2. The total is 210.
done by computing powers of 11 and 12, or more efficiently by √
first reducing these mod 7 and then computing powers: 12 ≡ 7. 4 3 − 11π
6
5 mod 7, hence122011 ≡ 52011 mod 7; and similarly 112012 ≡
42012 mod 7. Draw a trapezoid ADFE by dropping perpendicular feet from
the two centers A and D to the points E and F on the tangent
Now tabulate the powers working mod 7, and look for cyclic line. The area sought is found by removing two circular sectors
patterns: from the trapezoid. Note that because AD has length 4, DG has
length 2, and the triangle is right, we see that √
triangle ADG is
k = 1 2 3 4 5 6 7 8 9 10 11 12 a 30-60-90 triangle. Therefore AG has length 2 3. The area of
12k ≡ 5 4 6 2 3 1 5 4 6 2 3 1 the trapezoid is the average of AE and DF √ times the distance
11k ≡ 4 2 1 4 2 1 4 2 1 4 2 1 between two tangent points on the line, or 4 3. Angle GAD is
4 FINAL ROUND JANUARY 2012

! computable by the methods used in Problem 9 below, but there


is also a shortcut method to do this problem:
D Note that the table contains a duplicate of itself after the first
3 roll. This means that 5/6 of the time the first roll is a win that
2
A1 G adds 3, and then we get to start rolling all over again. Thus
1 S = (1/6)(0) + (5/6)(3 + S), which implies S = 15.
E F
! (9.a) ( N − 1)2 N +1 + 2

F IGURE 5. Trapezoid minus two sectors


0
1 1
30 degrees, so angle EAD is 30+90=120 degrees and the sector 2 2 2 2
in the smaller circle is one third of that circle. It has area π3 . 3 3 3 3 3 3 3 3
Angle GDA is 60 degrees and the sector in the larger circle is 4 4 4 4 4 4 4 4 4 4 4 4 4 4 4 4
one sixth of that circle, hence has area 3π
2 .
√ √
Desired area= 4 3 − π3 − 3π 11π F IGURE 6. Coins in boxes
2 =4 3 − 6 .

8. 15 Let TN = 1 · 2 + 2 · 4 + 3 · 8 + 4 · 16 + . . . + N · 2 N be the total


number of coins in all the boxes in all the rows up to the row
On any single roll, 5/6 of the time you win some points. On with N coins per box. Let S N = 1 + 2 + 4 + 8 + 16 + . . . + 2 N be
average this win W adds (1 + 2 + 3 + 4 + 5)/5 = 3. On the the total number of boxes, including the box at the top with zero
other hand, 1/6 of the time you roll a losing value L = 6 that coins in it. The question asks for a formula for TN . First get a
halts the game. Now let S be the average sum of all possible formula for S N , which is a geometric series, by using a standard
strings of rolls. Below is a table of all possible strings. Each W method: investigate the effect of doubling the sum.
in a string contributes 3 points to its average value.
S N = 1 + 2 + 4 + 8 + 16 + . . . + 2 N
String Average Value Probability 2 · S N = 2 + 4 + 8 + 16 . . . + 2 N + 2 N +1
1
L 0
5
6
1 Subtract the first equation from the second one, and cancel pairs
WL 3 6 × 6 of duplicated terms to deduce that 2S N − S N = 2 N +1 − 1.
WWL 6 ( 6 ) × 16
5 2

WWWL 9 ( 56 )3 × 16 Now try a similar trick on the TN : investigate the effect of dou-
... ... ... bling the sum that defines Tn .
2 · TN = 1 · 4 + 2 · 8 + 3 · 16 + 4 · 32 + . . . + N · 2 N +1
Taking the sum of these values, weighted by the probability of
each case, gives S = (3 × 16 ) ∑∞ k 5
k =1 kp where p = 6 . This is TN = 1 · 2 + 2 · 4 + 3 · 8 + 4 · 16 + . . . + N · 2 N
UNC MATH CONTEST SOLUTIONS 5

Subtract to get
(10.b) (n+ 3
4 )
2TN − TN =
!"#$%&'(&")*&+*+,-&./01&+*+,-&.20

(0 − 1)2 + (1 − 2)4 + (2 − 3)8 + . . . + ( N − 1 − N )2 N + N2 N +1 c


= N2 N +1 − (2 + 4 + 8 + . . . + 2 N ) c A
c
= N2 N +1 − (S N − 1)
= N2 N +1 − (2 N +1 − 2) = ( N − 1)2 N +1 + 2 B b
b
(9.b) 2201
b
Note that if ( N − 1)2 N +1 + 2 ≈ 20, 000 then ( N − 1)2 N ≈ 10, 000. a
Tabulate powers of 2 and estimate N2 N . Guessing N ≈ 10, we a C
check that indeed with this choice a
(N − 1 ) 2 N +1 + 2 = 9 × 2048 + 2 = 18434,
which is ≈ 20000. So far we have taken all boxes up to all the F IGURE 7. Triangles erected exterior to hexagon
boxes with 10 coins. We still need to get 20121 − 18434 = 1687
more coins using boxes that now have 11 coins. Divide 1687/11 Label the sides of the equiangular hexagon so that A is longest,
to get 153 and a remainder that forces us to take one more box, and a is opposite it. Let B be the longer side adjacent to a, and
making 154 more in all. (There are 211 boxes with 11 coins each, C be the shorter side adjacent to a. Use lower-case letters to la-
so there are plenty available.) Thus we need S10 + 154 = 2047 + bel their opposite sides. Erect exterior equilateral triangles to
154 = 2201 boxes. It is possible to do part (b) without getting the lower-case sides. This frames the hexagon inside an equi-
the formula in (a). Compute explicitly how many coins are in lateral triangle. Equate the lengths of the three sides of the
the row with one coin per box, two coins per box, and so on. equilateral frame: c + A + b = c + B + a = b + C + a; hence
A number of students did this. This problem is an extension of A − a = B − b = C − c. Call this common difference d. It satis-
Problem 10 from the First Round. fies the inequality 0 ≤ d ≤ C − 1. The side length c = C − d sat-
isfies 1 ≤ c ≤ C. Every equiangular hexagon determines a list
(10.a) 126 of four such whole numbers A ≥ B ≥ C ≥ c ≥ 1. Conversely,
by reversing the steps in the construction, we see that for every
One approach is to enumerate the possibilities. In addition to list of four such whole numbers there exists exactly one equian-
understanding the geometric properties of equiangular hexagons, gular hexagon that has these given side lengths. The reversed
one must have a careful and systematic method to do the enu- construction starts by computing d = C − c, b = B − d, and
meration (see appendix). One student did this correctly. Obvi- a = A − d; then drawing an equilateral triangle with side length
ously, another approach is to complete part (b) for the explicit d (possibly zero); then assembling the equiangular hexagon by
case n=6. adjoining parallelograms as drawn in the figure below.
6 !"#$%&'(&")*&+*+,-.&/012&+*+,-.&/31
FINAL ROUND JANUARY 2012

c x is c = −(rst + rsu + rtu + stu), and the constant term d is


A d = rstu.
We are told that r + s + t + u = 1 and r2 + s2 + t2 + u2 = 2 and
r3 + s3 + t3 + u3 = 3 and r4 + s4 + t4 + u4 = 4. Therefore a =
B d −1. Next observe that (r + s + t + u)2 − (r2 + s2 + t2 + u2 ) =
d 2(rs + rt + ru + st + su + tu), which is 2b. Since we know that
b (r + s + t + u) = 1 and the sum of the squares is 2, 2b = 1 − 2 =
d −1 or b = − 21 . That is, the coefficient of x2 is b = − 21 . We use
similar observations to find c and d. It will be convenient to
name the sums of the powers of the roots: R1 = r + s + t + u =
1, R2 = r2 + s2 + t2 + u2 = 2, R3 = r3 + s3 + t3 + u3 = 3, and
a C R4 = r4 + s4 + t4 + u4 = 4. The coefficient c = −(rst + rsu +
rtu + stu). To find c, look at combinations of a, b, and R# s that
produce terms with three factors of the roots r, s, t, and u; that
F IGURE 8. Parallelograms around a triangle is, look at things like R3 or R1 b or R2 a. Keep in mind that the
first two useful equalities were − a = R1 and −2b = aR1 + R2 .
Observe that in fact −3c = bR1 + aR2 + R3 . Put in the values we
We now count all such lists that satisfy n ≥ A ≥ B ≥ C ≥ c ≥ 1 know for the right side and −3c = (− 12 )(1) + (−1)(2) + 3 = 21 .
by recognizing that this is version (iv) of Problem 3 above! The Thus c = − 61 . Similarly, −4d = cR1 + bR2 + aR3 + R4 = − 61 +
number of such lists is (n+ 3
4 ). (− 12 )(2) + (−1)(3) + 4 = − 16 and d = 24 1
. We have determined
the last coefficient that we need.
The appendix shows all 126 of the hexagons for n=6. They
are ordered and color-coded first by the length of the longest The identities relating the Rs and the a, b, c and d are very old
side. Then they are ordered by the lengths of the three other identities called the Newton-Girard formulae. The relationships
sides. between the roots of a polynomial and the coefficients are also
very old and are known as Vieta’s formulae. The R# s are some-
(11) x4 − x3 − 12 x2 − 16 x + 1
24 times called power functions and the expressions rs + rt + ru +
st + su + tu, rst + rsu + rtu + stu and so on are known as sym-
The polynomial P( x ) = ( x − r )( x − s)( x − t)( x − u) has roots metric functions in the roots r, s, t and u. Students were not
r, s, t, and u. Multiply out: expected to have seen these before! The challenge was to work
out the various relationships.
P( x ) = x4 − (r + s + t + u) x3 + (rs + rt + ru + st + su + tu) x2
The contest writing team this year included Oscar Levin, Rich Mor-
−(rst + rsu + rtu + stu) x + rstu. row, Richard Grassl, Katie Diaz, contest director Ricardo Diaz, and
Rocke Verser, who submitted Problems 3, 10, and 11.
The coefficient a of x3 is a = −(r + s + t + u), the coefficient
b of x2 is b = (rs + rt + ru + st + su + tu), the coefficient c of
UNC MATH CONTEST SOLUTIONS 7
Twenty-first Annual UNC Math Contest First Round October, 2012

Rules: 90 minutes; no electronic devices.

The positive integers are 1, 2, 3, 4, . . ..


The first twelve prime numbers are 2, 3, 5, 7, 11, 13, 17, 19, 23, 29, 31, 37.
A polygon is called regular if its sides have equal lengths and its angles are all equal.

1. Write the numbers 1 through 11, in order, clockwise around a circle. Starting with the
number 2 and moving clockwise, erase every other number; that is, erase 2, then 4, then 6,
and so on. Continue moving around the circle repeatedly, erasing every other remaining
number, until no numbers remain. Which is the last number erased?

2. Two chests contain gold coins. After one fourth of the gold coins in the first chest are
moved to the second chest, each chest contains 2550 coins. How many coins were in the
second chest at the start?

3. If the first day of April is a Wednesday, what day of the week is the Fourth of July that
year? (April has 30 days, May has 31, and June has 30.)

P A X 4. Square TPXU has side length 8. The point A splits


segment PX in half. The lengths of segments OT, OU,
and OA are equal. What is the area of triangle OUT?
O

T U

5. Dav scrambles letters lined up on a tray. He scrambles the original message MATH
CLUB ENJOYS PI into CHAPS TUMBLE IN JOY, using this rule: letter 1 goes to position 8,
letter 2 goes to position 3, . . . and letter 16 goes to position 12.
M A T H C L U B E N J O Y S P I
1 2 3 4 5 6 7 8 9 10 11 12 13 14 15 16
C H A P S T U M B L E I N J O Y
Dav uses the same rule each time he rearranges his letter tray.
(a) In the tenth rearrangement of the original message (that is, after 10 repetitions of Dav’s
rule), what is the position of the letter A on the tray?
(b) Dav discovers that when he follows his rule repeatedly, the original message MATH
CLUB ENJOYS PI is eventually converted back to itself. After how many repetitions does
this occur for the first time?

TURN PAGE OVER


Scale in cm: 2:1 (x), 2:1 (y)

6. The six regular pentagons in the figure are congru-


ent. At each point where three pentagons meet, they
X
leave an angular gap X. Find the measure of angle X
in degrees.

7. As you skate forward on ice, a crack forms that extends in a straight line in the direction
that you skate. Each time a new crack is made, you turn counter-clockwise to skate in a
new direction, and you create a crack in that new direction. The first time, you turn 5◦ ,
so that the second crack makes a 5◦ angle with the first crack. The second time you turn
10◦ , so the third crack makes a 10◦ angle with the second crack. The next time you turn
15◦ , so the fourth crack makes a 15◦ angle with the third crack, and so on. Each time you
turn five more degrees than you turned the previous time, always rotating in the same
counter-clockwise direction. After how many turns will the next new crack be parallel to
the first crack? (You are asked to count turns, not cracks.)

8. The first time after noon that the hour and minute hands on a twelve hour clock are
separated by exactly 99◦ , how many minutes is it past the hour?

9. Six friends sit at a round table. The first person


6 has 4 candies. The person to their left has 6. Contin-
4 8 uing around clockwise, they have 8, 4, 6, and 8 can-
dies, as shown in the diagram. They play the follow-
ing game: all at once, everyone passes exactly half
8 of their candy to the person on their left. Then each
4 player who has an odd number of candies eats one of
6 their candies. Then they repeat. How many candies
does the first person have after 2012 repeats?

10. (a) How many positive integers less than 201 are exactly divisible (meaning without
remainder) by at least one squared prime number? For example, 72 is exactly divisible by
2 × 2 and exactly divisible by 3 × 3. Therefore 72 is one such positive integer. (b) How
many positive integers less than 1001 are exactly divisible by at least one squared prime
number?

11. Write the numbers 1 through 200, in order, clockwise around a circle. Starting with the
number 2 and moving clockwise, erase every other number; that is, erase 2, then 4, then 6,
and so on. Continue moving around the circle repeatedly, erasing every other remaining
number, until no numbers remain. Which is the last number erased?

END OF CONTEST
University Of Northern Colorado Mathematics Contest

University Of Northern Colorado Mathematics Contest 2012-2013


Problems of First Round with Solutions

1. Write the numbers 1 through 11, in order, clockwise around a circle. Starting with the
number 2 and moving clockwise, erase every other number; that is, erase 2, then 4, then 6,
and so on. Continue moving around the circle repeatedly, erasing every other remaining
number, until no numbers remain. Which is the last number erased?
Answer: 7
Solution:
Let us just do it:
First round:

11 1
10 2

9 3

8 4
7 5
6
Second round:

11 1

9 3

7 5
Go ahead:

11 11

7 7

The green dot indicates the starting position after we goes around the circle once. So the last
number to be erased is 7.
The more elegant solution can be found after you have experienced Problem 11.

Problems are duplicated and solved by Ming Song (msongmath@yahoo.com) 4


University Of Northern Colorado Mathematics Contest

2. Two chests contain gold coins. After one fourth of the gold coins in the first chest are
moved to the second chest, each chest contains 2550 coins. How many coins were in the
second chest at the start?
Answer: 1700
Solution:
Let x be the number of coins in the first chest at the start. We have
1
x" x ! 2550 .
4
Solving for x we obtain x ! 3400 . Then the number of coins in the second chest at the start is
1
2550 " x ! 2550 " 850 ! 1700 .
4

3. If the first day of April is a Wednesday, what day of the week is the Fourth of July that
year? (April has 30 days, May has 31, and June has 30.)
Answer: Saturday
Solution:
The number of days from April 1 to July 4 is
30 # 31 # 30 # 3 ! 94 ! 3 mod 7 .
We count 3 days from Wednesday. Then July 4 is Saturday.

4. Square TPXU has side length 8. The point A splits segment PX in half. The lengths of
segments OT, OU, and OA are equal. What is the area of triangle OUT?
A
P X

T U

Answer: 12
Solution:
Let AO intersect TU at V. Obviously AV $ TU .
Let the radius of the circle be x.
8
Then OA ! OT ! x , OV ! 8 " x , and TV ! !4.
2
By Pythagorean Theorem we have in right triangle OVT

Problems are duplicated and solved by Ming Song (msongmath@yahoo.com) 5


University Of Northern Colorado Mathematics Contest

x2 % 4 2 $ !8 # x " .
2

A
P X

T V U

Solving for x we obtain x % 5 .


Then OV % 8 # 5 % 3 .
1
Therefore, the area of triangle OUT is & 8 & 3 % 12 .
2

5. Dav scrambles letters lined up on a tray. He scrambles the original message MATH CLUB
ENJOYS PI into CHAPS TUMBLE IN JOY, using this rule: letter 1 goes to position 8, letter
2 goes to position 3, ! , and letter 16 goes to position 12.

M A T H C L U B E N J O Y S P I
1 2 3 4 5 6 7 8 9 10 11 12 13 14 15 16
C H A P S T U M B L E I N J O Y
Dav uses the same rule each time he rearranges his letter tray.
(a) In the tenth rearrangement of the original message (that is, after 10 repetitions of
Dav’s rule), what is the position of the letter A on the tray?
(b) Dav discovers that when he follows his rule repeatedly, the original message MATH
CLUB ENJOYS PI is eventually converted back to itself. After how many repetitions
does this occur for the first time?
Answer: (a) 3; (b) 18
Solution:
We use Lx to represent “Letter x” and Py to represent “Position y”. Lx ' Py represents that
Letter x goes Position y.
We see
L1 ' P8 L 2 ' P3 L3 ' P 6 L4 ' P 2
L5 ' P1 L 6 ' P10 L 7 ' P7 L8 ' P9
L9 ' P11 L10 ' P13 L11 ' P14 L12 ' P15
L13 ' P16 L14 ' P5 L15 ' P 4 L16 ' P12
(a)
We trace the position of A in each rearrangement:

Problems are duplicated and solved by Ming Song (msongmath@yahoo.com) 6


University Of Northern Colorado Mathematics Contest

Steps A’s position


Beginning 2
1st 3
2nd 6
3rd 10
4th 13
5th 16
6th 12
7th 15
8th 4
9th 2
10 th 3
The answer is “Position 3”.
(b)
L1, L5, L8, L9, L11, and L14 take 6 rearrangements to come back to their original positions.
L7 stays at Position 7 forever.
All other letters take 9 rearrangements to come back to their original positions.
Therefore, the answer is the least common multiple of 6 and 9, which is 18.

6. The six regular pentagons in the figure are congruent. At each point where three pentagons
meet, they leave an angular gap X. Find the measure of angle X in degrees.

Answer: 36!
Solution:
540!
A regular pentagon has an interior angle measuring " 108! .
5
So angle X measures
360! $ 3 # 108! " 36! .

Problems are duplicated and solved by Ming Song (msongmath@yahoo.com) 7


University Of Northern Colorado Mathematics Contest

7. As you skate forward on ice, a crack forms that extends in a straight line in the direction
that you skate. Each time a new crack is made, you turn counter-clockwise to skate in a
new direction, and you create a crack in that new direction. The first time, you turn 5°, so
that the second crack makes a 5° angle with the first crack. The second time you turn 10°,
so the third crack makes a 10° angle with the second crack. The next time you turn 15°, so
the fourth crack makes a 15° angle with the third crack, and so on. Each time you turn five
more degrees than you turned the previous time, always rotating in the same counter-
clockwise direction. After how many turns will the next new crack be parallel to the first
crack? (You are asked to count turns, not cracks.)
Answer: 8
Solution:
For the new crack to be parallel to the first crack, you must turn a multiple of 180°.
Let n be the number of turns. We search the smallest value forn such that
5 " 10 " ! " 5n ! 180m ,
where m is a positive integer.
That is, 1 " 2 " ! " n ! 36m .
Or, n#n " 1$ ! 72m .
We need the smallest value for n. So let m ! 1 first.
We have n ! 8 fortunately. This is the smallest positive value of n for 5 " 10 " ! " 5n ! 180m to
be true.
The answer is 8.

8. The first time after noon that the hour and minute hands on a twelve hour clock are
separated by exactly 99°, how many minutes is it past the hour?
Answer: 18
Solution:
Assume that at x minutes past noon the two hands intersect at an angle of 99° as shown, where O
is the center of the clock, OA is the 12 o’clock position, OB is the hour hand position, and OC is
the minute hand position.
A B

XII
XI I
X II
99º
IX O III

VIII IV C
VII VI V

Problems are duplicated and solved by Ming Song (msongmath@yahoo.com) 8


University Of Northern Colorado Mathematics Contest

Note that the hour hand moves 360! in 12 hours, and the minute hand moves 360! in an hour.
So the hour hand moves 30! in an hour, and the minute hand moves 6! in a minute.
Look at the hour hand in the figure above.
x x x
x minutes account to hours. So $AOB " # 30 " in degrees.
60 60 2
Look at the minute hand. $AOC " 6 # x in degrees.
From $AOC " $AOB % 99! we have
x
6x " % 99 .
2
Solving for x we obtain x " 18 .
At 18 minutes past 12 o’clock the two hands first time intersect at an angle of 99°.

9. Six friends sit at a round table. The first person has 4 candies. The person to their left has 6.
Continuing around clockwise, they have 8, 4, 6, and 8 candies, as shown in the diagram.
They play the following game: all at once, everyone passes exactly half of their candy to
the person on their left. Then each player who has an odd number of candies eats one of
their candies. Then they repeat. How many candies does the first person have after 2012
repeats?
4

8 6

6 8

4
Answer: 4
Solution:
Let us play the game.
First round:
Pass 6 Eat 6

7 5 6 4

5 7 4 6

6 6

Problems are duplicated and solved by Ming Song (msongmath@yahoo.com) 9


University Of Northern Colorado Mathematics Contest

Second round:
Pass 6 Eat 6

5 5 4 4

5 5 4 4

6 6
Third round:
Pass 5 Eat 4

4 5 4 4

5 4 4 4

5 4
Then everyone will keep having 4 candies.
Therefore, the answer is 4.

10. (a) How many positive integers less than 201 are exactly divisible (meaning without
remainder) by at least one squared prime number? For example, 72 is exactly
divisible by 2 ! 2 and exactly divisible by 3! 3 . Therefore 72 is one such positive
integer.
(b) How many positive integers less than 1001 are exactly divisible by at least one
squared prime number?
Answer: (a) 78; (b) 392
Solution:
We count multiples of 22 " 4 , 32 " 9 , 52 " 25 , 7 2 " 49 , 112 " 121, etc.
(a)
Note that 132 " 169 # 201 and 172 " 289 $ 201 .
We will count the multiples of 22 " 4 , 32 " 9 , 52 " 25 , 7 2 " 49 , 112 " 121, or 132 " 169 .
There is only one multiple of 169, which is 169.
There is only one multiple of 121, which is 121.
There are four multiples of 49, which are 49, 98, 147, and 196.
Note that 196 is also a multiple of 4. We exclude 196 at this moment. We count 3 multiples of 49.

Problems are duplicated and solved by Ming Song (msongmath@yahoo.com) 10


University Of Northern Colorado Mathematics Contest

There are eight multiples of 25. Among them there are two multiples of 4, which are 25! 4 and
25! 8 . In multiples of 25, we count 8 # 2 " 6 .
Now we use the inclusive and exclusive principle to count the multiples of 4 or 9.
The number is
' 201 % ' 201 % ' 201 %
' 4 % $ ' 9 % # ' 4 ! 9 % " 50 $ 22 # 5 " 67 ,
( & ( & ( &
where (x & represents the greatest integer less than or equal to x.
The answer is 67 $ 1 $ 1 $ 3 $ 6 " 78 .
(b)
Note that 312 ) 1001 and 37 2 * 1001 .
We need to count the multiples of 2 2 " 4 , 32 " 9 , 52 " 25 , 7 2 " 49 , 112 " 121 , 132 " 169 ,
172 " 289 , 192 " 361 , 232 " 529 , 292 " 841 , or 312 " 961 .
There is only one multiple of 961, which is 961.
There is only one multiple of 841, which is 841.
There is only one multiple of 529, which is 529.
'1001%
Note that ' % " 2 . There are 2 multiples of 361.
( 361 &
'1001 %
' 289 % " 3 . There are 3 multiples of 289.
( &
'1001 %
' 169 % " 5 . There are 5 multiples of 169, one of which is 169 ! 4 that will be counted in
( &
multiples of 4. So in multiples of 169, we count 5 # 1 " 4 .
'1001 %
' 121 % " 8 . There are 8 multiples of 121, two of which are 121! 4 and 121! 8 that will be
( &
counted in multiples of 4. So in multiples of 121, we count 8 # 2 " 6 .
'1001%
' 49 % " 20 . There are 20 multiples of 49, five of which are multiples of 4 and two of which are
( &
multiples of 9. So in multiples of 49, we count 20 # 5 # 2 " 13 .
Now we use the inclusive and exclusive principle to count the multiples of 4, 9, or 25.
The number is
'1001% '1001% '1001% '1001% ' 1001 % ' 1001 % ' 1001 %
' 4 % $ ' 9 % $ ' 25 % # ' 4 ! 9 % # ' 4 ! 25 % # ' 9 ! 25 % $ ' 4 ! 9 ! 25 % .
( & ( & ( & ( & ( & ( & ( &
" 250 $ 111 $ 40 # 27 # 10 # 4 $ 1 " 361
The answer is 361 $ 1 $ 1 $ 1 $ 2 $ 3 $ 4 $ 6 $ 13 " 392 .

Problems are duplicated and solved by Ming Song (msongmath@yahoo.com) 11


University Of Northern Colorado Mathematics Contest

11. Write the numbers 1 through 200, in order, clockwise around a circle. Starting with the
number 2 and moving clockwise, erase every other number; that is, erase 2, then 4, then 6,
and so on. Continue moving around the circle repeatedly, erasing every other remaining
number, until no numbers remain. Which is the last number erased?
Answer: 145
Solution:
Note that we will have about half of numbers left if we go around the circle once. Let us consider
the powers of 2.
If there are 2 numbers, the last number to be erased is 1:
1

2
If there are 4 numbers, the last number to be erased is 1:
1 1

4 2

3 3
If there are 8 numbers, the last number to be erased is 1:
1 1 1
8 2

7 3 7 3

6 2
5 5 5
By induction we can easily prove that if the number of numbers is a power of 2, the last number
to be erased is 1.
It is more important to notice that 1 is the first number at which we start.
When the number of numbers is a power of 2, we see the answer immediately, which is the first
number.
Now 200 is not a power of 2. Let us make it.
The power of 2 closest to 200 and less than 200 is 128.

Problems are duplicated and solved by Ming Song (msongmath@yahoo.com) 12


University Of Northern Colorado Mathematics Contest

Note that 200 " 128 ! 72 .


So if we will erase 72 numbers first, there will be 128 numbers remaining.
The first 72 numbers to be erased are 2, 4, 6, ! , 144 .
When 144 is erased we see
200 1 3
199 5
198
139
141
143
147 144
146 145

The first number now


Around this circle we have 128 numbers with 145 being the first. The first number is the answer.
Therefore, the answer is 145.

Problems are duplicated and solved by Ming Song (msongmath@yahoo.com) 13


Twenty-first Annual UNC Math Contest Final Round January 19, 2013

Three hours; no electronic devices. Justify your answers. Clear and concise presentations will earn more points.
We hope you enjoy thinking about these problems, but you are not expected to solve them all.
The positive integers are 1, 2, 3, 4, . . . .

1. In the diagram, the two circles are tangent to the two parallel lines. The
distance between the centers of the circles is 8, and both circles have
radius 3. What is the area of the shaded region between the circles?

2. E XAMPLE : The number 64 is equal to 82 and also equal to 43 , so 64 is both a perfect square and a perfect cube.
(a) Find the smallest positive integer multiple of 12 that is a perfect square.
(b) Find the smallest positive integer multiple of 12 that is a perfect cube.
(c) Find the smallest positive integer multiple of 12 that is both a perfect square and a perfect cube.

3. Point C is the center of a large circle that passes through both A and
C B, and C lies on the small circle whose diameter is AB. The area of the
small circle is 9π. Find the area of the shaded lune, the region inside
A B the small circle and outside the large circle.

! "( x2 +7x+10)
4. Find all real numbers x that satisfy x2 − 72 x + 32 = 1.

5. If the sum of distinct positive integers is 17, find the largest possible value of their product. Give both a set of
positive integers and their product. Remember to consider only sums of distinct numbers, and not 3+7+7 or
2+3+4+4+4, etc., which have repeated terms. You need not justify your answer on this question.
E XAMPLE : Distinct Integers: {2, 3, 4, 8} Their Sum: 2 + 3 + 4 + 8 = 17 Their Product: 2 × 3 × 4 × 8 = 192

6. There is at least one Friday the Thirteenth in every year.


(a) What is the latest possible month in which the first Friday the Thirteenth can occur?
(b) In a year in which the first Friday the Thirteenth occurs in its latest month, what day of the week is January 1?
The table below shows the number of days in each month. February has 29 days in leap years and 28 in others.

JAN FEB MAR APR MAY JUN JUL AUG SEP OCT NOV DEC
31 28 or 29 31 30 31 30 31 31 30 31 30 31

7. Suzie and her mom dry half the dishes together; then mom rests, while Suzie and her dad dry the other half.
Drying the dishes this way takes twice as long as when all three work together. If Suzie’s mom takes 2 seconds
per dish and her dad takes 5 seconds per dish, how long does Suzie take per dish?
TURN OVER
8. E XAMPLE : The non-terminating periodic decimal 0.124124 . . . = 0.124 has period three and is abbreviated by
placing a bar over the shortest repeating block.
(a) If all digits 0 through 9 are allowed, how many distinct periodic decimals 0.d1 d2 . . . d6 have period exactly
six? Do not include patterns like 0.323 and 0.17 that have shorter periods.
(b) If only digits 0 and 1 are allowed, how many distinct periodic decimals 0.d1 d2 . . . d12 have period exactly 12?

9. The standard abbreviation for the non-terminating repeating decimal .34121121121121121 . . . is .34121, a string
of five digits. How many distinct non-terminating repeating decimals .d1 d2 d3 . . . have standard abbreviations
that have at most six digits? (Consider two nonterminating decimals distinct if they differ in any digit. Nonter-
minating means that the digits are not eventually all zero.)
C OMMENTS The standard abbreviation is also the shortest. For example, .34121121121121121 . . . = .34121 can
also be abbreviated as .341211, or as .3412112, or as .34121121 by sliding the bar rightward, making longer
strings. The nonterminating decimal .34121 has two parts: a repeating tail T = 121 and a non-repeating head
H = 34. If the string has no head, the decimal is periodic, which is acceptable. There must be a tail string
T, which by convention is NOT permitted to be T = 0, since that corresponds to a terminating decimal. The
examples .345, .9, and .7219 are all standard abbreviations for nonterminating repeating decimals.

10. Dav designs a robot, which he calls FrankenCoder, to print nonsense text by scrambling eleven-letter messages.
The robot always repeats the same scrambling rule.

FrankenCoder scrambles ENIGMACRUSH into 1 2 3 4 5 6 7 8 9 10 11


GENIUSCHARM. Using the same rule, Franken- 1st: E N I G M A C R U S H
Coder then scrambles GENIUSCHARM into 2nd: G E N I U S C H A R M
IGENARCMSHU, and so on. 3rd: I G E N A R C M S H U

1 ! 5 "
! " 9 11 FrankenCoder’s internal wiring for scrambling letters is dia-
2
4 & % grammed at left, depicted as a collection of cycles. The arrows
$
# 6 8 show how each of the eleven letters moves in a single scramble:
7 # 10 $
!

3 letter 7 stays in its place, the first four letters move in a cycle,
and the other six letters also trade positions in a cycle.

Dav sees that the messages printed by FrankenCoder repeat cyclically in paragraphs: eventually, the original
message ENIGMACRUSH reappears as the start of a new paragraph identical to the first paragraph.
(a) How many distinct messages does each paragraph contain?
(b) Dav tries to improve the robot, to get an even longer paragraph of distinct messages, by drawing different
wiring diagrams for the eleven letter positions. Experimenting with component cycles of various lengths, he
perfects his ultimate robot: FrankenCoder-II, a robot that produces the longest possible paragraph of distinct
eleven-letter messages. How many distinct messages does FrankenCoder-II produce?
(c) Draw a wiring diagram that could describe FrankenCoder-II. There may be ties, since different wiring dia-
grams can make robots that print paragraphs that have the same length. Draw just one wiring diagram.
(d) Dav realizes that because there are ties for the best wiring diagram, he can build an entire army of distinct
robots that are as good as FrankenCoder-II at creating long paragraphs. How many distinct robots can he
build that are as good as FrankenCoder-II? Include FrankenCoder-II in your count. (Two robots are regarded as
distinct if they scramble the starting message ENIGMACRUSH into different messages.)

11. (a) Stages 1 and 2 each contain 1 tile. Stage 6 contains 8 tiles. If
the pattern is continued, how many tiles will Stage 15 contain?
(b) What is the first Stage in which the number of tiles is a mul-
tiple of 2013?
END OF CONTEST
Twenty-first Annual UNC Math Contest Final Round Solutions January 19, 2013

1. In the diagram, the two circles are tangent to the two parallel lines. The
distance between the centers of the circles is 8, and both circles have
radius 3. What is the area of the shaded region between the circles?
A NSWER 48 − 9π

S OLUTION The shaded region is what remains after two half-circles


have been removed from a rectangle whose width is 8 and whose
height is 6. The area of the rectangle is 6 × 8 = 48. The two half-
circles of radius 3 have total area 9π. The area of the shaded region is
therefore 48 − 9π.
Decimal approximations were accepted. Students might want to be aware that some teachers require exact
answers unless approximations are explicitly requested. No deductions were made on this contest, though, for
replacing the exact answer with a decimal approximation.

2. E XAMPLE : The number 64 is equal to 82 and also equal to 43 , so 64 is both a perfect square and a perfect cube.
(a) Find the smallest positive integer multiple of 12 that is a perfect square. A NSWER 36
(b) Find the smallest positive integer multiple of 12 that is a perfect cube. A NSWER 216
(c) Find the smallest positive integer multiple of 12 that is both a perfect square and a perfect cube.
A NSWER 46, 656

S OLUTION (a) 12 = 22 × 3. Perfect squares are the numbers for which prime factors each occur an even number
of times. To get a perfect square here we require at minimum one more factor of 3. This gives 22 × 32 = 36.
(b) To get a cube we require each prime factor to be repeated three times: 23 × 33 = 8 × 27 = 216.
(c) The number of times each prime factor occurs must be even and also a multiple of three, so now each prime
factor must be repeated six times: 26 36 = 66 = (63 )2 = (216)2 = 46656.

3. Point C is the center of a large circle that passes through both A and
B, and C lies on the small circle whose diameter is AB. The area of the
C
small circle is 9π. Find the area of the shaded lune, the region inside
Z the small circle and outside the large circle. A NSWER 9
A D B
Y
S OLUTION (See figure.) The area sought is labeled X. The center of the
X
small circle is D.
From the given area of the small circle, we see that the small radius is DB = 3.
(i) The area X + Y is half the area of the small circle, so X + Y = 9π
2 .

(ii) Segment BC has length 3 2. This can be shown as follows. First note that AD=BD=3, so the triangles ADC
and BDC are congruent. Thus angles ADC and BDC are both congruent and supplementary (i.e. add up to 180

degrees). This makes them both right angles. By the Pythagorean theorem, BC has length 3 2. (Alternatively,
one could observe that triangle ABC is inscribed in the small circle with one side as diameter and so ABC is a
right triangle. Then find BC using the Pythagorean theorem.)
(iii) The triangle ABC has area Z = 9 because it has width 6 and height 3.


(iv) The area Y + Z = 2 because the region is one quarter of a circle of radius 3 2. The area of that quarter
πr2 π ×9×2
circle is 4 = 4 = 9π
2 .

Thus the area of the lune is X = ( X + Y ) − Y = ( X + Y ) − (Y + Z ) + Z = (i) -(iv) +(iii) = 2 − ( 9π
2 ) + 9 = 9.
! "( x2 +7x+10) √
7± 41
4. Find all real numbers x that satisfy x2 − 72 x + 32 = 1. A NSWER x = 1, −2, −5, and 4

S OLUTION The equation b p = 1 is true in each of the following cases: (i) when p = 0 and b #= 0 (ii) when b = 1
(iii) when b = −1 and p is an even integer. Check all these cases for solutions x.
(i) Set p( x ) = ( x2 + 7x + 10) = ( x + 2)( x + 5) = 0. This gives x = −2 and x = −5. Note that b is not zero for
either of these values of x, so these are both solutions.
(ii) Set b = x2 − 72 x + 3
= 1. Then x2 − 72 x + 12 = 0. The quadratic formula gives
2
# # #
7 √ √
± ( 72 )2 − 4 × ( 12 ) 7
± ( 49
) − 8 7
± 41 7 1
2 2 4 4 2 4 2 ± ( 2 ) 41 7 ± 41
x= = = = =
2 2 2 2 4

(iii) Set b = x2 − 72 x + 3
2 = −1. Then x2 − 72 x + 52 = 0. The quadratic formula gives
# # #
7
2 ± ( 72 )2 − 4 × ( 52 ) 7
2 ± ( 49
4 ) − 40
4
7
2 ± 9
4
7
± ( 32 ) 7±3
x= = = = 2 = 2
2 2 2 2 2

This gives x = 1 or x = 52 . Note that x = 1 makes the power p = 18, which is even, so x = 1 is a solution.
However, x = 5/2 makes the power p = 135/4, which is not even. Thus x = 5/2 is not a solution.

7± 41
In summary, the five values x = 1, −2, −5, and 4 are the solutions.

5. If the sum of distinct positive integers is 17, find the largest possible value of their product. Give both a set of
positive integers and their product. Remember to consider only sums of distinct numbers, and not 3+7+7 or
2+3+4+4+4, etc., which have repeated terms. You need not justify your answer on this question.
E XAMPLE : Distinct Integers: {2, 3, 4, 8} Their Sum: 2 + 3 + 4 + 8 = 17 Their Product: 2 × 3 × 4 × 8 = 192

A NSWER {2, 4, 5, 6} The product is 240

S OLUTION Consider 2+2+2+2+2+2+2+3=17 vs 8+9=17. The product of all the terms in the first sum is 3 ×
27 = 384 and the product of the two terms in the second sum is 8 × 9 = 72. You can make things bigger
multiplying more small things rather than fewer larger, in general. However, the problem requires distinct
integers. Therefore we try 2, 3, 4, 5. This is not quite 17, though. We have three extra to put somewhere. Not
enough to toss in the 6, so we fiddle and arrive at 2, 4, 5, 6.
Choose the set {2, 4, 5, 6}. 2 + 4 + 5 + 6 = 17 and 2 × 4 × 5 × 6 = 240.
6. There is at least one Friday the Thirteenth in every year.
(a) What is the latest possible month in which the first Friday the Thirteenth can occur? A NSWER OCTOBER
(b) In a year in which the first Friday the Thirteenth occurs in its latest month, what day of the week is January 1?
A NSWER SATURDAY
The table below shows the number of days in each month. February has 29 days in leap years and 28 in others.

JAN FEB MAR APR MAY JUN JUL AUG SEP OCT NOV DEC
31 28 or 29 31 30 31 30 31 31 30 31 30 31
S OLUTION (a) To analyze all possibilities you should look at regular (non-leap) years and also leap years.
Case 1. (See the left diagram for non-leap years.) The seven vertices of the polygon correspond to days of the
week, listed in clockwise order. Starting on the weekday of January 13, note that 31 days later, February 13 will
be located three days later in the week (indicated by an arrow jumping ahead three, traveling from Jan to Feb).
Repeating this month by month, we land on various days of the week, and we look to see which weekday takes
the longest time to be landed upon. The vertex that remains vacant longest is finally visited in September. If
that vertex is labelled Friday, then we see that Friday the thirteenth can be put off so that its first occurrence is
in September.

Jumping across each Month of a


Non-Leap Year, Sept has last
unchosen weekday position. LEAP YEAR: Oct has last
unchosen position
Jan, April, July 13
Jan 13, Oct 13
Sept Oct
April,
July May,Nov

Aug June May


Sept 13,
Dec 13

Feb 13, March Feb 13,


June March 13, Aug 13
Nov 13
!

A Non-Leap Year A Leap Year

Case 2. (See the right diagram for a leap year) Using a similar procedure, we see that the last vertex to be chosen
occurs in October, and we should label that vertex as Friday 13.
(b) The leap year diagram shows that Friday 13th falls in October when January 13 is one weekday sooner
(Thursday), and therefore January 15 is one weekday later (Saturday). Since January 15 and January 1 share
the same day of the week, the answer is Saturday. ( If you forget to consider leap years, you get answers
SEPTEMBER and TUESDAY. Partial credit was awarded for those answers.)
7. Suzie and her mom dry half the dishes together; then mom rests, while Suzie and her dad dry the other half.
Drying the dishes this way takes twice as long as when all three work together. If Suzie’s mom takes 2 seconds
per dish and her dad takes 5 seconds per dish, how long does Suzie take per dish?
A NSWER 10 seconds per dish

S OLUTION This is a time, distance, rate problem which uses the basic principle Progress = Rate × Time.
Let x stand for the number that is sought, namely, the number of seconds Suzie takes to dry a dish. (Note that
the rate at which Suzie dries dishes is 1x dishes/sec.) Consider what information is given in the question: a
relationship between the times required for various teams to complete certain tasks. That is,
The time T1 taken by Team 1 (Suzie and mom) PLUS the time T2 taken by Team 2 (Suzie and dad) is twice the
time T3 it takes for Team 3 ( Suzie, Mom, and Dad) to dry all the dishes.
The plan is to write these times in terms of x and solve for x.
First we list the given information. (The individuals’ drying rates are expressed in units of dishes/sec. One of
the features that makes this problem interesting is that the information given is seconds/dish information and
what we need to add together is dishes/second information.)
(a) Mom = 12 dish/sec
1
(b) Dad = 5 dish/sec
(c) Suzie = 1x dish/sec
(d) T1 + T2 = 2T3 .
Next compute the drying rates and drying times of the various teams. The number of dishes Mom and Suzie
dry per second is the sum of the number of dishes Mom dries in a second and the number Suzie dries in a
second- that is, the rate for a team is the sum of the individual rates of the team members.

Team Drying Rates and Times:


1 1 2+ x
R1 = x + 2 = 2x T1 = 12 ( All dishes) R1
1
1 1 5+ x
R2 = x + 5 = 5x T2 = 12 ( All dishes) R12
1 7x +10
R3 = x + 15 + 1
2 = 10x T3 = ( All dishes) R13

Thus by (d) (T1 + T2 = 2T3 ) deduce that


1 1 4 2x 5x 40x
R1 + R2 = R3 , i.e. 2+ x + (5+ x )
= 7x +10 .

Cancel x, then clear of all denominators to get the quadratic equation 9x2 − 70x − 200 = 0. Reject the negative
solution. Deduce x=10.
8. E XAMPLE : The non-terminating periodic decimal 0.124124 . . . = 0.124 has period three and is abbreviated by
placing a bar over the shortest repeating block.
(a) If all digits 0 through 9 are allowed, how many distinct periodic decimals 0.d1 d2 . . . d6 have period exactly
six? Do not include patterns like 0.323 and 0.17 that have shorter periods. A NSWER 998, 910
(b) If only digits 0 and 1 are allowed, how many distinct periodic decimals 0.d1 d2 . . . d12 have period exactly 12?
A NSWER 4020

S OLUTION Our goal is to count all cases exactly once, compensating for the overlaps, using the inclusion/exclusion
principle.
(a) The inclusion-exclusion Venn diagram at left lists the various types
! of types of repeating strings, and how these types overlap. For ex-
"#$%&'!*! "#$%&'!(! ample, a string that repeats with length 3 also repeats with length
!!!!)*+,! !!!)*+(,! 6. The parenthetical numbers indicate that there are 106 different
!
strings of six digits, 103 different strings of three digits, 102 strings
of two digits, and 10 strings that have one digit. The number of
"#$%&'!-! "#$%&'!.!
!!!!!)*+-,! !!!!)*+.,! strings with a repeat length of exactly six and nothing smaller is
! !
1, 000, 000 − 1000 − 100 + 10 = 1, 000, 010 − 1100 = 998, 910.

(b) There are 212 = 4096 strings of length 12. If the fundamental period
!!
length of a repeating string is smaller than 12, it is a proper divisor of
"#$%&'!(! Length 6 12, in which case, it is either (i) a divisor of 6, or (ii) a divisor of 4; and
!!!!)((*! (26) if both (i) and (ii), it is (iii) a divisor of 2.
!
Case (i) has 26 = 64 strings, case (ii) has 24 = 16 strings, and case
Length 4 Length 12 (iii) has 22 = 4 strings. Thus there are 26 + 24 − 22 = 76 strings that
(24) (212) have periods that are proper divisors of 12, and there are 212 − 76 =
! !
4096 − 76 = 4020 strings that have period exactly 12.
You can consider the strings that have period exactly 1 and 3, also. Then you will have more overlaps to consider.
Since the strings with periods 1 and 3 also repeat every 6 digits, it is not necessary to separate these cases.
9. The standard abbreviation for the non-terminating repeating decimal .34121121121121121 . . . is .34121, a string
of five digits. How many distinct non-terminating repeating decimals .d1 d2 d3 . . . have standard abbreviations
that have at most six digits? (Consider two nonterminating decimals distinct if they differ in any digit. Nonter-
minating means that the digits are not eventually all zero.)
C OMMENTS The standard abbreviation is also the shortest. For example, .34121121121121121 . . . = .34121 can
also be abbreviated as .341211, or as .3412112, or as .34121121 by sliding the bar rightward, making longer
strings. The nonterminating decimal .34121 has two parts: a repeating tail T = 121 and a non-repeating head
H = 34. If the string has no head, the decimal is periodic, which is acceptable. There must be a tail string
T, which by convention is NOT permitted to be T = 0, since that corresponds to a terminating decimal. The
examples .345, .9, and .7219 are all standard abbreviations for nonterminating repeating decimals.
A NSWER 5, 778, 810
S OLUTION Any choice of the head string H = hm . . . h2 h1 and repeating tail T = tn . . . t2 t1 , selected indepen-
dently, will create a repeating decimal .HT, but in order to count only distinct items, we must be sure to keep
only the choices that are in shortest form. A combination .HT can be shortened precisely when H and T have the
same last digit: h1 = t1 . If head and tail strings of digits do not have matching ending symbol, then when they
are combined, the resulting string cannot be shortened.
The key idea is to first choose the tail T, then count how many strings have this tail. It is permissible to add
either no head or any head H whose last digit does not match that of T.
Select T first, look at its terminal symbol t1 , and then count all head strings H that DO NOT match this given t1 .
That there are 9 ways to pick h1 that avoid matching t1 . Next we may adjoin arbitrary extra head digits .hm . . . h2
in the slots to the left of h1 , until we use up the maximum string length, six. There are 10 acceptable digits that
can be placed in each such slot.
Example. Suppose the periodic tail has length three, such as T = 312. What strings of length at most six have
exactly this tail? The head can have either one digit, two digit, three digits, or be empty (headless), so (i) H = h1
or (ii) H = h2 h1 or (iii) H = h3 h2 h1 or (iv) we choose to omit any head.
In case (i) there are 9 choices for h1 that avoid the digit 2; in case (ii) there are 10 × 9 choices; in case (iii) there
are 10 × 10 × 9 choices for the head, and in case (iv) we make the single choice to omit any head. Thus for a tail
of length three, there are a total of 1000 distinct strings that have this tail. Using these ideas, we make a table
describing all cases:
Tail Length Number of possible heads Number of tail choices Product=Total choices
Six 1 106 − 103 − 102 + 10 1 × (106 − 103 − 102 + 10)
Five 10 105 − 10 (10) × (105 − 10)
Four 102 104 − 102 (102 ) × (104 − 102 )
Three 103 103 − 10 (103 ) × (103 − 10)
Two 104 102 − 10 (104 ) × (102 − 10)
One 105 9 (105 ) × (9)
The numbers of periodic tails of various lengths were computed as in the previous problem. Note that the
lowest line in the table departs slightly from the basic pattern: the number of tails of length one is only 9, since
the tail 0 is not permitted. Total number of strings of length at most six: 5, 778, 810.
10. Dav designs a robot, which he calls FrankenCoder, to print nonsense text by scrambling eleven-letter messages.
The robot always repeats the same scrambling rule.

FrankenCoder scrambles ENIGMACRUSH into 1 2 3 4 5 6 7 8 9 10 11


GENIUSCHARM. Using the same rule, Franken- 1st: E N I G M A C R U S H
Coder then scrambles GENIUSCHARM into 2nd: G E N I U S C H A R M
IGENARCMSHU, and so on. 3rd: I G E N A R C M S H U

1 ! 5 " FrankenCoder’s internal wiring for scrambling letters is diagrammed


! " 9 11 at left, depicted as a collection of cycles. The arrows show how each of
2
4 & % the eleven letters moves in a single scramble: letter 7 stays in its place,
$
# 6 8 the first four letters move in a cycle, and the other six letters also trade
7 # 10 $
!

3
positions in a cycle.

Dav sees that the messages printed by FrankenCoder repeat cyclically in paragraphs: eventually, the original
message ENIGMACRUSH reappears as the start of a new paragraph identical to the first paragraph.
(a) How many distinct messages does each paragraph contain? A NSWER 12
(b) Dav tries to improve the robot, to get an even longer paragraph of distinct messages, by drawing different
wiring diagrams for the eleven letter positions. Experimenting with component cycles of various lengths, he
perfects his ultimate robot: FrankenCoder-II, a robot that produces the longest possible paragraph of distinct
eleven-letter messages. How many distinct messages does FrankenCoder-II produce? A NSWER 30
(c) Draw a wiring diagram that could describe FrankenCoder-II. There may be ties, since different wiring dia-
grams can make robots that print paragraphs that have the same length. Draw just one wiring diagram.
One possible internal wiring for FrankenCoder-II is diagrammed at
1 12 ! 5 "
! " 9 11
left. The five letters 1, 2, 3, 7, and 4 move in a cycle, and the other six,
2
A NSWER 4 & %
5, 11, 8, 10, 6, and 9 also trade positions in a cycle. Any example with
$
4 # 3
6 8
3 7
!

# 10 $ one cycle of 5 and one cycle of 6 is acceptable. Any example with a


7 !!!!!!!!!!!!!!!!! ! cycle of length 1, a cycle of length 2, a cycle of length 3, and a cycle of
length 5 is acceptable.
(d) Dav realizes that because there are ties for the best wiring diagram, he can build an entire army of distinct
robots that are as good as FrankenCoder-II at creating long paragraphs. How many distinct robots can he
build that are as good as FrankenCoder-II? Include FrankenCoder-II in your count. (Two robots are regarded as
distinct if they scramble the starting message ENIGMACRUSH into different messages.) A NSWER 2, 661, 120

S OLUTION (a) From the wiring diagram, we see that letters 1-4 return to their original positions every four
repeats. Letter 7 is always in its original position. The remaining letters return to their original positions every
six repeats. All the letters will first return to their original position after LCM(4,6)= 12 repeats. There are 12
distinct messages in a paragraph.
(b) We are looking for sets of cycle lengths that add up to 11 and have the largest possible LCM. For complete-
ness, we list all possibilities below, in order of increasing complexity (more pieces means more complexity). We
begin with one large cycle, move to the case of two separate cycles , then three separate cycles, etc.
One single cycle of length 11 gives LCM 11. (Obviously not great.)

Two separate cycles: 11= 3 + 8; LCM=24 Three separate cycles:


11= 1 + 10; LCM=10 11= 4 + 7; LCM=28 11= 1+1+9; LCM=9
11= 2 + 9; LCM=18 11= 5 + 6; LCM=30 11= 1+2+8; LCM=8
11= 1+3+7; LCM=21 11= 1+2+3+5; LCM=30 11= 1+2+2+2+4; LCM=4
11= 1+4+6; LCM=12 11= 1+2+4+4; LCM=4 11= 1+2+2+3+3; LCM=6
11= 1+5+5; LCM=5 11= 1+3+3+4; LCM=11 11= 2+2+2+2+3; LCM=6
11= 2+2+7; LCM=14 11= 2+2+2+5; LCM=10 Six separate cycles:
11= 2+3+6; LCM=6 11= 2+2+3+4; LCM=24
11=1+1+1+1+1+6; LCM=6
11= 2+4+5; LCM=20 11= 2+3+3+3; LCM=6
11=1+1+1+1+2+5; LCM=5
11= 3+3+5; LCM=15 Five separate cycles:
11=1+1+1+1+3+4; LCM=12
11= 3+4+4; LCM=12 11= 1+1+1+1+7; LCM=7
11=1+1+1+2+2+4; LCM=4
Four separate cycles: 11= 1+1+1+2+6; LCM=6
11=1+1+1+2+3+3; LCM=6
11= 1+1+1+8; LCM=8 11= 1+1+1+3+5; LCM=15
11= 1+1+2+7; LCM=14 11= 1+1+1+4+4; LCM=4 11=1+1+2+2+2+3; LCM=6

11= 1+1+3+6; LCM=6 11= 1+1+2+2+5; LCM=10 11=1+2+2+2+2+2; LCM=2

11= 1+1+4+5; LCM=20 11= 1+1+2+3+4; LCM=12 Seven separate cycles:


11= 1+2+2+6; LCM=6 11= 1+1+3+3+3; LCM=3 11=1+1+1+1+1+1+5; LCM=5

And so on. Nobody is expected to write out all the possibilities: checking in your head is acceptable! Clearly
none of the rest will give LCM as large as 30.
Conclusion. The greatest LCM is 30 and occurs in two distinct ways: (i) for cycles 5 and 6 and (ii) for cycles of
lengths 1, 2, 3, and 5. In both cases, FrankenCoder-II will will produce 30 distinct messages.
It is important that there are TWO optimal cycle decompositions. Both solutions must be found in order to
successfully tackle the following part (d) below. For part (d), it will not suffice to just find one of them.
(c) Choose either cycles of lengths 5 and 6 or cycles of lengths 1, 2, 3, and 5. Partition the numbers 1-11 into
appropriate groups, pick an order, and draw your wiring diagram.
(i) One possible internal wiring for FrankenCoder-II is diagrammed at
1 12 ! 5 "
! " 9 11 left. The five letters 1, 2, 3, 7, and 4 move in a cycle, and the other six,
2
4 & % 5, 11, 8, 10, 6, and 9 also trade positions in a cycle. Any example with
$
4 # 3
6 8
3 7
!

# 10 $ one cycle of 5 and one cycle of 6 is acceptable.


7 !!!!!!!!!!!!!!!!! ! (ii) Another possibility, (not diagrammed) has a cycle of length 1, a
cycle of length 2, a cycle of length 3, and a cycle of length 5. (It suffices
to draw just one example, either of type (i) or of type (ii).)
(d) We must count (i) the number of choices of robots with cycles of lengths 5 and 6 and (ii) the number of
choices of robots with cycles of lengths 1, 2, 3 and 5.
(i) Count first the case of cycles of lengths 5 and 6. Put the numbers 1-11 into the following blanks:
( , , , , )( , , , , , )
There are 11! ways to do this. Each 5 cycle will appear on the list 5 ways, one way with each element as first:
(1, 2, 3, 4, 5), (2, 3, 4, 5, 1), (3, 4, 5, 1, 2), etc. Likewise, each 6 cycle will appear 6 ways. The total number of
choices of wiring diagrams with cycles of lengths 5 and 6 is 511! ×6
(ii) Now count the case of cycles with lengths 1, 2, 3, and 5. Put the numbers 1-11 into the following blanks:
( )( , )( , , )( , , , , , )
There are 11! ways to do this. Each 2 cycle appears twice, the 3 cycles appear 3 ways, and the 5 cycles appear 5
ways. The total number of choices of wiring diagrams with cycles of lengths 1, 2, 3, and 5 is 2×11! 11!
3×5 = 5×6
11! 11!
Add the results from (i) and (ii). There are 2 × 5×6 = 5×3 =11 × 10 × 9 × 8 × 7 × 6 × 4 × 2= 990 × 56 × 48=
2,661,120 robots in all.
11. (a) Stages 1 and 2 each contain 1 tile. Stage 6 contains 8 tiles. If the
pattern is continued, how many tiles will Stage 15 contain?
A NSWER 610
(b) What is the first Stage in which the number of tiles is a multiple of
2013? A NSWER 60th
S OLUTION (a) Each figure is formed by combining the two previous figures. The number of tiles at one Stage is
the sum of the numbers of tiles in the previous two, with the first two Stages having one tile each. The numbers
produced this way are called the Fibonacci numbers, of course.
Stage: 1st 2nd 3rd 4th 5th 6th 7th 8th 9th 10th 11th 12th 13th 14th 15th
Number of tiles: 1 1 2 3 5 8 13 21 34 55 89 144 233 377 610
The number of tiles in the 15th Stage is 610.
(b) The problem is to find the index of the first Fibonacci number that is a multiple of 2013. Suppose we know
that a particular Fibonacci number Fk is a multiple of some integer n. Consider the Fibonacci sequence mod
n. That is, write the remainders of each Fibonacci number when divided by n. Call the new sequence f j .
Observe that the f j satisfy the same recursion identity that the Fibonacci numbers obey: f j = f j−1 + f j−2 .
What we are saying is that for some particular k, we know f k = 0. Then f k+1 = f k + f k−1 = f k−1 and also
f k+2 = f k+1 + f k = f k+1 = f k−1 . The sequence is ... f k−1 , 0 f k−1 , f k−1 , 2 f k−1 , 3 f k−1 , 5 f k−1 ,... We are starting over
with f k−1 , times the original sequence. When we go k more terms we will be back to 0 (mod n). We find that if
Fk is zero mod some n then every kth Fibonacci number will be zero mod n. In other words, if the kth Fibonacci
number is a multiple of n, then so is every k Fibonacci number in the sequence. (In fact, what we found is that
every kth Fibonacci number is a multiple of Fk . That is, every third number in the sequence is a multiple of the
third Fibonacci number; every 4th Fibonacci number is a multiple of the fourth one, and so on.)
To find the first Fibonacci number that is a multiple of 2013, first factor 2013=3 × 11 × 61. Look for the first n for
which Fn is a multiple of 3, the first n for which Fn is a multiple of 11, and the first n for which Fn is a multiple of
61. Then we will find the LCM of these indices and that will be the first index for which the Fibonacci number
is a multiple of all three factors and hence of 2013. Checking the list we made in part (a), we find that the first
Fibonacci number that is a multiple of 3 is the 4th one. (So every fourth Fibonacci number is a multiple of 3.)
The first one that is a multiple of 11 is 55, which is the 10th one. (So every tenth Fibonacci number is a multiple
of 11.) The first one that is a multiple of 61 is the 15th one. (So every 15th Fibonacci number is a multiple of 61.)
LCM(4,10,15)=60. Therefore, the first Fibonacci number that is a multiple of 2013 is the 60th one and the first
Stage in which the number of unit tiles is a multiple of 2013 is also the 60th . Note how nicely this works out for
the year 2013.
Twenty-second Annual UNC Math Contest First Round November, 2013

Rules: 90 minutes; no electronic devices.

The problems are arranged in no particular order of difficulty.

1. CHIN-UPS On Day 1, Jim does his first chin-up. On Day 2 he does 2 chin-ups. Each day he
does one more chin-up than he did the day before.
(a) On which day will Jim do his 10th chin-up?
(b) On which day will Jim do his 150th chin-up?

B
2. BARN The four solid edges (two vertical sides
A and two slanted top edges) have equal length. The
C four dashed edges (three cross-bars and the horizon-
tal floor) also have equal length. If the area of the small
shaded triangle is one, what is the total area enclosed
by the five-sided figure ABCDE?
E D

3. WHISKERS & PINTS Whiskers found several pints of ice cream in the freezer and quickly
ate one fifth of the whole amount. Not quite content, he slurped up one fifth of a pint more,
leaving exactly seven pints remaining. How many pints did Whiskers eat in all?

4. GOOGOLS A googol can be written in decimal form as 1 followed by 100 zeroes:


10,000,000,000,000,000,000,000,000,000,000,000,000,000,000,000,000,000,000,000,000,000,000,000,000,000,000,000,000,000,000,000,000,000

(a) How many digits are in the decimal form of the product googol × googol ?
(b) How many different prime factors does the number googol googol have?
A prime is an integer greater than one whose only divisors are itself and one.

5. ROBOT A room with a square floor 100 × 100 is


paved with identical square tiles whose dimensions are
1 × 1. A robot travels along a rectangular spiral that
starts at one corner and moves inward toward the cen-
ter, crossing each tile exactly once and then stopping.
(a) How many times does the robot turn a corner?
(b) When the robot has visited half of the tiles on the
floor, how many turns has it made?

TURN PAGE OVER


6. N & M Suppose that 0 < M < N, N − M = 7, and M × N = 60. What is N + M?

7. NUMBER LINE The points A, B, C, D, E, and F are equally spaced and in alphabetical order
on a number line. The point A is at 5 and the point F is at 15. Where is the point D?

<————–A————–B————–C————–D————–E————–F————–>

8. Q Find an integer Q that satisfies the inequalities 26 < Q + 16 < 2Q < 36.
The inequality sign < means strict inequality. That is, a < b means that a is less than b and not equal to b.

9. POCKET SWAP The left pocket contains only dimes, and the right pocket contains only
quarters. After an equal number of dimes and quarters change sides, the coins in the left
pocket will be worth three times as much as the coins in the right pocket. What is the smallest
positive total number of coins that could be in the two pockets?
A dime is worth 10 cents and a quarter is worth 25 cents.

10. ALPHABET COOKIE LEFTOVERS (a) Seven delicious alphabet cookies are listed alpha-
betically in a row as A, B, C, D, E, F, G. You eat some or all of the cookies and look at the
alphabetized pattern made by the leftovers. How many different possible alphabetized left-
over patterns are there which contain no pairs of consecutive letters? For example, you might eat
cookies A, B, D, and F, and leave C, E, and G. There are no pairs of consecutive letters in this pattern of three remaining
cookies. We do not care in what order you eat the cookies; count two patterns as the same if the same cookies are left
remaining. Count the example above and also the case of no cookies left in your total of possible patterns.
(b) Suppose you start with the eleven cookies A, B, C, D, E, F, G, H, I, J, K. Now how many
different possible alphabetized patterns of uneaten cookies are there which contain no pairs
of consecutive letters?

11. [LOG] (a) Compute the sum [log2 1] + [log2 2] + [log2 3] + . . . + [log2 10].
(The notation [x] means the greatest integer less than or equal to x. For example, [2.9]=2.
The notation logb a means logarithm base b of a. For example x=log3 a satisfies 3x = a.)

END OF CONTEST
ANSWERS:
1a. Day 4 On Day 1, Jim does sit-up number 1. On Day 2, he does two more. These are
sit-ups numbers 2 and 3. He has now done 1+2=3 sit-ups in all. On Day 3, he does three more
or 3+3=6 in all. On Day 4 he has done 6+4=10 in all. The last sit-up he does on Day 4 is his
tenth sit-up.
1.b Day 17 One can continue as above until the total has gone above 150. Alternatively,
cale in cm: 2.4:1 (x), 2.4:1 (y)
n ( n +1)
one can note that after day n Jim will have completed 1 + 2 + . . . + n = 2 chin-ups. Set
n ( n +1)
2 ≥ 150 and deduce that the smallest positive solution is n = 17.
B

2. Area= 15 . The large equilateral triangle BDE


has side length three times the side length of the small
A G H C
shaded equilateral triangle, hence has area 9. Three
congruent copies of the exterior isosceles triangles BCD
and EAB will exactly fill the large equilateral triangle
F
BDE, so each isosceles triangle has area 3. The total fig-
ure has five such isosceles triangles, so the total area is
15. (There are many ways to approach this one. The diagram is full
E D
of 30 and 60 degree angles and similar triangles.)

3. 2 pints First solve for x = the number of pints in the freezer. After Whiskers eats
a fifth of x, what remains is 45 x. Next this is reduced by an additional 15 . What remains is
4 1
5 x − 5 = 7. Thus x = 9. Finally, subtract the 7 remaining from the 9 original to compute what
Whiskers consumes, 9-7=2 pints.
4.a 201 digits Note that googol × googol = 10100 × 10100 = 10200 . This is written in decimal
form as 1 followed by 200 zeros. The total number of digits is 201.
4.b 2 primes The number 10 = 2 × 5 has two prime factors. All positive integer powers
of 10 also have only these two prime factors. This principle is applicable to both googol = 10100
and googol googol = 10100googol
5.a 198 turns A 4 × 4 room has a spiral with seven legs and six turns. A 6 × 6 room has
a spiral with eleven legs and ten turns. In general an n × n room has 2n − 2 turns. If n = 100
there are 2 × 99 = 198 turns.
5.b After 58 turns After every four corner turns, the unswept area that remains is another per-
fect square. A crude estimate is that half

the tiles have been swept when the perfect square
that remains has edge length close to 22 100 = 70.7. We now refine this estimate. The perfect
square with dimensions 70 × 70, whose unswept area is 4900, corresponds to 60 completed
corner turns and 5100 swept tiles, which is 100 tiles too many. Now refine this slightly, work-
ing backwards from this data point. The area reaches the halfway point after the 58th turn and
before the 59th turn.
6. 17 M = N + 7 so 60 = MN = ( N + 7) N = N 2 + 7N. That is, N 2 + 7N − 60 =
0. Factor as ( N + 12)( N − 5) or use the quadratic formula to deduce N = 5 or N = −12.
Rejecting the negative value, infer that N = 5 and therefore M = N + 7 = 12. Thus M + N =
12 + 5 = 17.
7. D = 11 The distance between A and F is ten units and this has been divided into five
equal lengths. Therefore, the spacing between consecutive points is 2 and D is 4 units below
F. 15 − 4 = 11.
8. Q = 17 The leftmost inequality implies 10 < Q. The second inequality implies
16 < Q. Therefore, the leftmost inequality is not relevant and we may discard it. The last
inequality implies Q < 18. The only integer Q with 16 < Q < 18 is Q = 17.
9. 5 coins (3 dimes and 2 quarters, you switch 2 coins)
10. a A7 = 34 is the number of leftover arrangements of seven cookies
10. b A11 = 233 It is not an accident that these are Fibonacci numbers! By checking small
values of N first, it is easy to check that A2 = 3 and A3 = 5. Furthermore, the number of
arrangements of N cookies must satisfy the recurrence A N +1 = A N + A N −1 . Indeed, when
an extra alphabet cookie is added to a tray that already has the first N cookies, the number
of possible arrangements of leftovers, A N +1 , can be accounted for by looking at two disjoint
cases: (i) if we eat the new cookie, then there A N choices for the remaining N cookies; and (ii)
if we do not eat the ( N + 1)th cookie, then we must eat the Nth cookie, which leaves N − 1
cookies that can be arranged in A N −1 ways. The recurrence pattern therefore compels all
subsequent values to be Fibonacci numbers.
11. 19
University of Northern Colorado Mathematics Contest

University Of Northern Colorado Mathematics Contest 2013-2014


Problems of First Round with Solutions

1. CHIN-UPS On Day 1, Jim does his first chin-up. On Day 2 he does 2 chin-ups. Each day
he does one more chin-up than he did the day before.
(a) On which day will Jim do his 10th chin-up?
Answer: 4th
Solution:
Note that 1 " 2 " 3 " 4 ! 10 .
So the last chin-up of the four on the 4th day is the 10th chin-up overall.
(b) On which day will Jim do his 150th chin-up?
Answer: 17th
Solution:
Note that
16 $ 17
1 " 2 " ! " 16 ! ! 136 # 150
2
and
1 " 2 " ! " 17 ! 136 " 17 ! 153 % 150 .
So the 150 chin-up is on the 17th day.
th

2. BARN The four solid edges (two vertical sides and two slanted top edges) have equal
length. The four dashed edges (three cross-bars and the horizontal floor) also have equal
length. If the area of the small shaded triangle is one, what is the total area enclosed by the
five-sided figure ABCDE?
B

A C

E D
Answer: 15
Solution 1:
The four solid segments are the four sides of a regular hexagon. The four dashed segments are the
four congruent diagonals of the hexagon.

Problems are duplicated and solved by Ming Song (msongmath@yahoo.com) 3


University of Northern Colorado Mathematics Contest

By recognizing these we see the answer without calculations as shown.


B

A C

E D

The answer is 15 with all small triangles having the same area.
Solution 2:
Let AC intersect BD and BE at G and H respectively.
B

h
A C
G H

E D
Let AG ! CH ! 1 .
Since #BED ! #BDE ! 60" , #AEG ! #CDH ! 30" . Triangles EAG and DCH are 30°-60°-90°
triangles. So AE ! CD ! 3 and EG ! DH ! 2 .
Since BAE and BCD are isosceles triangles, #ABE ! #AEB ! #CBD ! #CDB ! 30" .
Then #BAE ! #BCD ! 180" % 2 $ 30" ! 120" . So #BAG ! #BCH ! 120" % 90" ! 30" .
Then &ABG and &BCH are isosceles. We have BG ! AG ! BH ! CH ! 1 .
With GM || ED &BGH is equilateral. So GH ! 1 . Then DE ! AC ! 1 ' 1 ' 1 ! 3 .

3
Let h be the height of triangle BGH. Then h ! .
2
1 3 3
The area of triangle BGH is (1 ( ! .
2 2 4
1 3 15 3
The area of pentagon ABCDE is (3( '3( 3 ! .
2 2 4
The ratio of the area of pentagon ABCDE to the area of triangle BGH is 15.
Now the area of triangle BGH (shaded) is 1.
So the area of pentagon ABCDE is 15.

Problems are duplicated and solved by Ming Song (msongmath@yahoo.com) 4


University of Northern Colorado Mathematics Contest

3. WHISKERS & PINTS Whiskers found several pints of ice cream in the freezer and
quickly ate one fifth of the whole amount. Not quite content, he slurped up one fifth of a
pint more, leaving exactly seven pints remaining. How many pints did Whiskers eat in all?
Answer: 2
Solution:
Let x be the whole amount of ice cream. We have the following equation
4 1
x" !7.
5 5
So x ! 9 .
1 1
Then Whiskers ate $ 9 # ! 2 pints of ice cream.
5 5

4. GOOGOLS A googol can be written in decimal form as 1 followed by 100 zeroes:


| & 100 zeros %|
1000! 000 .

(a) How many digits are in the decimal form of the product google $ google ?
Answer: 201
Solution:
A googol = 10100 . So google $ google ! 10100 $ 10100 ! 10 200 . It has 201 digits.

(b) How many different prime factors does the number google google have?
A prime is an integer greater than one whose only divisors are itself and one.
Answer: 2
Solution:
google google is 10 k where k is a positive integer. So google google has two different prime factors:
2 and 5.

5. ROBOT A room with a square floor 100 $ 100 is paved with identical square tiles whose
dimensions are 1$ 1 . A robot travels along a rectangular spiral that starts at one corner and
moves inward toward the center, crossing each tile exactly once and then stopping.

(a) How many times does the robot turn a corner?

Problems are duplicated and solved by Ming Song (msongmath@yahoo.com) 5


University of Northern Colorado Mathematics Contest

Answer: 198
Solution:
Let a square floor be n ! n . Since 100 is even, we first consider n to be even.
Let f "n # be the number of turns for the robot to walk through all tiles of the n ! n floor.
After the robot just turns the forth corner, it has an "n $ 2 # ! "n $ 2 # floor remaining to go:
n–2

n n–2

So we have
f "n # & 4 % f "n $ 2# .
Continuing the recursion we have
f "n # & 4 % f "n $ 2 # & 2 ! 4 % f "n $ 4# & 3 ! 4 % f "n $ 6# & ! .
Since n is even, we obtain
n$2
f "n # & ! 4 % f "2 # .
2
Obviously f "2# & 2 as shown below:

So for even integer n,


n$2
f "n # & ! 4 % 2 & 2"n $ 1# .
2
Now for n & 100 , the answer is 2 ' "100 $ 1# & 198 .
Further Comment:
If n is odd,

Problems are duplicated and solved by Ming Song (msongmath@yahoo.com) 6


University of Northern Colorado Mathematics Contest

n %1
f !n " & 4 # f !n % 2" & 2 $ 4 # f !n % 4" & 3 $ 4 # f !n % 6" & ! & $ 4 # f !1" .
2
Obviously, f !1" & 0 .
So for odd integer n, we also have
n %1
f !n " & $ 4 # 0 & 2!n % 1" .
2
For any positive integer n we always have
f !n " & 2!n % 1" .
In fact, we can combine the two cases into one.
Let f !n " be the number of turns for the robot to walk through all tiles of the n $ n floor.

After the robot just turns the second corner, it has an !n % 1" $ !n % 1" floor remaining to go:
n–1

n
n–1

So we have
f !n " & 2 # f !n % 1" .
Continuing the recursion we have
f !n " & 2 # f !n % 1" & 2 $ 2 # f !n % 2 " & 3 $ 2 # f !n % 3" & ! & !n % 1" $ 2 # f !1" .
Obviously f !1" & 0 .

Therefore, f !n " & 2!n % 1" .


(b) When the robot has visited half of the tiles on the floor, how many turns has it made?
Answer: 58
Solution:
After the first 4 turns the robot has walked through 4$ 99 tiles. After the second 4 turns the robot
has walked through 4$ 97 tiles, etc. So we look for an odd number 2k # 1 such that

Problems are duplicated and solved by Ming Song (msongmath@yahoo.com) 7


University of Northern Colorado Mathematics Contest

100 2
4 ' #99 & 97 & ! & !2k & 1"$ % .
2
Note that the sum of the first n odd positive integers is n 2 . We have

99 & 97 & ! & 3 & 1 ( 50 2 and !2k ) 1" & ! & 3 & 1 ( k 2 .

! "
So 4 ' 50 2 ) k 2 %
100 2
2
. That is, k 2 % 1250 .

Note that 35 2 ( 1225 . So k % 35 . Take k ( 35 . Then 2k & 1 ( 71 .


99 ) 71
From 71 to 99 there are & 1 ( 15 odd numbers.
2
99 & 71
Calculate 4 ' !99 & 97 & ! & 71" ( 4 ' 15 ' ( 5100 .
2
After 15 rounds of four turns, 60 turns in total, the robot has walked through 5100 titles, 100 tiles
over half of 100 2 titles. Just after the 60th turn, the robot has a 70 by 70 floor remaining to go.
100

60th Turn

70 by 70 100

59th Turn 58th Turn

the robot has walked through 5000 tiles


For the robot to walk exactly 5000 tiles, we have to let the robot go back 100 tiles. Two turns
(60th and 59th) back are enough, as shown. So the answer is 60 ) 2 ( 58 .

6. N & M Suppose that 0 * M * N , N ) M ( 7 , and M ' N ( 60 . What is N & M ?


Answer: 17
Solution 1:
!N & M "2 ( !N ) M "2 & 4 NM ( 7 2 & 4 ' 60 ( 289 ( 17 2 .
With 0 * M * N we obtain N & M ( 17 .

Problems are duplicated and solved by Ming Song (msongmath@yahoo.com) 8


University of Northern Colorado Mathematics Contest

Solution 2:
By feeling two numbers whose product is 60 and difference is 7, we know that one number is 12
and the other is 5. Their sum is 17.

7. NUMBER LINE The points A, B, C, D, E, and F are equally spaced and in alphabetical
order on a number line. The point A is at 5 and the point F is at 15. Where is the point D?
A B C D E F

Answer: 11
Solution 1:
3 3
F " A ! 15 " 5 ! 10 . AD ! # AF ! # 10 ! 6 . So D ! A $ AD ! 5 $ 6 ! 11 .
5 5
Solution 2:
2 3 2 3
By the weighted average D ! # A $ # F ! # 5 $ # 15 ! 11 .
5 5 5 5
Further Comment:
Solution 2 makes the following problem easily be solved:
3 1 3
What is the number way from to .
5 3 4
1 2 3 3 7
The answer is # $ # ! .
3 5 4 5 12

8. Q Find an integer Q that satisfies the inequalities 26 % Q $ 16 % 2Q % 36 .


The inequality sign < means strict inequality. That is, a % b means that a is less than b and not equal to b.

Answer: 17
Solution:
From the first inequality 26 % Q $ 16 we have Q & 10 .
From the second inequality Q $ 16 % 2Q we have Q & 16 .
From the third inequality 2Q % 36 we have Q % 18 .
So the solution to the inequalities is 16 % Q % 18 .
Since Q is an integer, Q ! 17 as the answer.

9. POCKET SWAP The left pocket contains only dimes, and the right pocket contains only
quarters. After an equal number of dimes and quarters change sides, the coins in the left
pocket will be worth three times as much as the coins in the right pocket. What is the
smallest positive total number of coins that could be in the two pockets?
A dime is worth 10 cents and a quarter is worth 25 cents.

Problems are duplicated and solved by Ming Song (msongmath@yahoo.com) 9


University of Northern Colorado Mathematics Contest

Answer: 5
Solution:
Let x be the number of dimes in the left pocket, y be the number of quarters in the right pocket,
and n be the number of coins changing sides.
The value of all coins in the left pocket is 10 x in cents, the value of all coins in the right pocket is
25 y in cents. By switching one coin the net change in the left pocket is 15 cents more, and the
net change in the right pocket is 15 cents less. So we have the following Diophantine equation:
10 x % 15n $ 3!25 y # 15n " .
That is,
10 x $ 75 y # 60n .
Or
2 x $ 15 y # 12n .
y must be even. Let y $ 2 z . We have
x $ 15 z # 6n .
Since we want to have the smallest number of coins, we let z $ 1 . Then y $ 2 .
15 5
Since x & 0 , n ' $ .
6 2
For x to be the smallest, we let n $ 2 . Then x $ 3 . The total number of coins is 3 % 2 $ 5 .
As the answer 5 is the smallest total number of coins.

10. ALPHABET COOKIE LEFTOVERS (a) Seven delicious alphabet cookies are listed
alphabetically in a row as A, B, C, D, E, F, G. You eat some or all of the cookies and look
at the alphabetized pattern made by the leftovers. How many different possible
alphabetized leftover patterns are there which contain no pairs of consecutive letters?
For example, you might eat cookies A, B, D, and F, and leave C, E, and G. There are no pairs of consecutive
letters in this pattern of three remaining cookies. We do not care in what order you eat the cookies; count two
patterns as the same if the same cookies are left remaining. Count the example above and also the case of no
cookies left in your total of possible patterns.
(b) Suppose you start with the eleven cookies A, B, C, D, E, F, G, H, I, J, K. Now how
many different possible alphabetized patterns of uneaten cookies are there which contain no
pairs of consecutive letters?
Answer: (a) 34 and (b) 233
By studying from the small starting numbers of cookies we may easily conjecture the pattern to
get the answers.
However, we have two wonderful ways to prove the pattern.
Solution 1:
We will derive the answer for n cookies.
Let n cookies be A1, A2 , A3 , !, An .

Problems are duplicated and solved by Ming Song (msongmath@yahoo.com) 10


University of Northern Colorado Mathematics Contest

Let f n be the number of ways to have the leftover satisfying the condition.

We have two decisions in A1 : eaten or not eaten.


If A1 is eaten, then we can have any leftover in n ! 1 cookies A2 , A3 , !, An provided that there is
no pair of two consecutive cookies. There are f n !1 ways to have the leftover in n ! 1 cookies. So
there are f n !1 ways to have the leftover when A1 is eaten.
If A1 is not eaten, then we must eat A2 . Otherwise, A1 and A2 are consecutive. Now we can have
any leftover in n ! 2 cookies A3 , A4 , !, An provided that there is no pair of two consecutive
cookies. There are f n ! 2 ways to have the leftover in n ! 2 cookies. So there are f n ! 2 ways to
have the leftover when A1 is not eaten.
Therefore, f n # f n !1 " f n ! 2 .

If n # 0 , f 0 # 1 . There is one way to have the leftover: no piece left.


If n # 1 , f1 # 2 . There are two ways to have the leftover: no piece left and one piece left.
This leads the Fibonacci sequence: 1, 2, 3, 5, 8, 13, 21, 34, 55, 89, 144, 233, ! with the first
number to be f 0 .
Then f 7 # 34 and f11 # 233 as the answers.
Solution 2:
Lemma:
The number of positive integer solutions to equation
x1 " x2 " ! " xm # n
is
) n !1 &
'' $$
( m ! 1%
where n is a positive integer with n * m .
Proof of the lemma:
We put n balls in a row:

We insert m ! 1 sticks between the balls, one stick in a gap. Let the number of balls left to the
first stick be x1 , the number of balls right to the last stick be xm , and xi be the number of balls
between stick i ! 1 and stick i for i # 2, 3, ! , m ! 1 .
There is a one-to-one correspondence between solutions to the original equation and ways to
insert sticks. So the number of positive integer solutions to the original equation is equal to the
number of ways to insert m ! 1 sticks into n ! 1 gaps between n balls.
) n !1 &
Therefore, the answer is '' $$ .
( m ! 1%

Problems are duplicated and solved by Ming Song (msongmath@yahoo.com) 11


University of Northern Colorado Mathematics Contest

We will derive the answer for n cookies.


Put n cookies in a row:

Consider k cookies B1 , B2 , B3 , ! , Bk left as shown.

x0 B1 x1 B2 x2 xk–1 Bk xk

Let x0 be the number of eaten cookies left to B1 , xk be the number of eaten cookies right to Bk ,
and xi be the number of eaten cookies between Bi and Bi !1 for i # 1, 2, ! , k " 1 .
We have the number of all eaten cookies:
x0 ! x1 ! x2 ! ! ! xk # n " k
with x0 $ 0 , xk $ 0 , and xi % 0 for i # 1, 2, ! , k " 1 . xi % 0 guarantees that Bi and Bi !1 are not
consecutive for all i from 1 to k " 1 .
Adding 2 in both sides:
&x0 ! 1' ! x1 ! x2 ! ! ! xk "1 ! &xk ! 1' # n " k ! 2 .
Let x0( # x0 ! 1 and x(n # xn ! 1 . Then

x0( ! x1 ! x2 ! ! ! xk "1 ! x(k # n " k ! 2 .


Now we have x0( % 0 , x(n % 0 , and all xi % 0 for i # 1, 2, ! , k " 1 .
By the lemma the number of positive integer solutions to the above equation is
. n " k ! 2 " 1+ . n " k ! 1+
,, )) # ,, )) .
- k ! 1 "1 * - k *
This is the number of possible leftovers of k cookies satisfying the condition.
n !1
For this to be positive we must have n " k ! 1 $ k . So k / .
2
5 n ! 12
k can be from 0 to 3 0 where 6x 7 denotes the greatest integer less than or equal to x.
4 2 1
Therefore, if there are n cookies at the beginning, the number of possible leftovers is
5 n !1 2
3 2 0
4 1
. n " k ! 1+
8 ,,-
k #0 k
)) .
*
For n # 7 , we have the answer
4
. 7 " k ! 1+ . 8 + . 7 + . 6 + . 5 + . 4 +
8 ,,-
k #0 k
)) # ,, )) ! ,, )) ! ,, )) ! ,, )) ! ,, )) # 1 ! 7 ! 15 ! 10 ! 1 # 34 .
* - 0 * - 1 * - 2 * - 3* - 4 *

Problems are duplicated and solved by Ming Song (msongmath@yahoo.com) 12


University of Northern Colorado Mathematics Contest

For n ! 11 , we have the answer


6
(11 ) k " 1% (12 % (11% (10 % ( 9 % ( 8 % ( 7 % ( 6 %
* &&'
k !0 k
## ! && ## " && ## " && ## " && ## " && ## " && ## " && ## ! 1 " 11 " 45 " 84 " 70 " 21 " 1 ! 233 .
$ ' 0 $ ' 1 $ ' 2 $ ' 3$ ' 4$ ' 5 $ ' 6 $
Further comment:
0 n "1 -
. 2 +
/
( n ) k " 1%
,
The sum * &&'
k !0 k
## is the nth Fibonacci number for all n 1 0 , which is shown in the
$
following Pascal’s Triangle:
1
1
2
3
5
8
13
1 21
34
1 1 55
1 2 1 89
144
1 3 3 1
233
1 4 6 4 1
1 5 10 10 5 1
1 6 15 20 15 6 1
1 7 21 35 35 21 7 1
1 8 28 56 70 56 28 8 1
1 84 36 9 1
9 36 84 126 126

1 10 45 120 210 252 210 120 45 10 1


1 11 55 165 330 462 462 330 165 55 11 1
1 12 66 220 495 792 924 792 495 220 66 12 1

Problems are duplicated and solved by Ming Song (msongmath@yahoo.com) 13


University of Northern Colorado Mathematics Contest

11. [LOG] Compute the sum !log 2 1" # !log 2 2" # !log 2 3" # ! # !log 2 10" .
(The notation !x " means the greatest integer less than or equal to x. For example !2.9" $ 2 .The notation log b a
means logarithm base b of a. For example x $ log 3 a satisfies 3 x $ a .)

Answer: 19
Solution:
If 1 & n % 2 , !log 2 n" $ 0 .

If 2 & n % 2 2 $ 4 , !log 2 n" $ 1 .

If 4 & n % 2 3 $ 8 , !log 2 n" $ 2 .

If 8 & n % 2 4 $ 16 , !log 2 n" $ 3 .


So !log 2 1" # !log 2 2" # !log 2 3" # ! # !log 2 10" $ 1' 0 # 2 '1 # 4 ' 2 # 3 ' 3 $ 19 .

Problems are duplicated and solved by Ming Song (msongmath@yahoo.com) 14


Twenty-second Annual UNC Math Contest Final Round January 25, 2014

Three hours; no electronic devices. Show your work and justify your answers.
Clearer presentations will earn higher rank. We hope you enjoy thinking about these problems,
but you are not expected to do them all.
You may write answers in terms of the Fibonacci numbers Fn .
The Fibonacci numbers are F1 = 1, F2 = 1, F3 = 2, F4 = 3, F5 = 5, F6 = 8, . . .
They are defined by the equations F1 = F2 = 1 and, for n > 2, Fn = Fn−1 + Fn−2 .

1. The Duchess had a child on May 1st every two years until she had five children. This year the
youngest is 1 and the ages of the children are 1, 3, 5, 7, and 9. Alice notices that the sum of the ages
is a perfect square: 1 + 3 + 5 + 7 + 9 = 25. How old will the youngest be the next time the sum of
the ages of the five children is a perfect square, and what is that perfect square?

2. Define the Cheshire Cat function :) by


:)( x ) = − x if x is even and
:)( x ) = x if x is odd
Find the sum :)(1)+ :)(2)+ :)(3)+ :)(4) + . . . + :)(289)

1 1
3. Find x and y if =2 and =2
1+ 1x 1+ 1
1
1+
1+ 1y

4. On the first slate, the Queen’s jurors write the number 1. On the second slate they write the
numbers 2 and 3. On the third slate the jurors write 4, 5, and 6, and so on, writing n integers on
the nth slate.
(a) What is the largest number they write on the 20th slate?
(b) What is the sum of the numbers written on the 20th slate?
(c) What is the sum of the numbers written on the nth slate?

!
!!!!!
!
! 5. (a) The White Rabbit has a square garden with sides of length
!!!!!
!
! one meter. He builds a square cucumber frame in the center by
!
!
connecting each corner of the garden to the midpoint of a far side
of the garden, going clockwise, as shown in the diagram. What is
!
the area of the region that is enclosed in the inner square frame?
X
X (b) Suppose that the White Rabbit builds his square cucumber
frame by connecting each corner of the garden to a point a distance
x from the next corner, going clockwise, as shown in the diagram.
X Now what is the area of the region that is enclosed in the inner
X ! !
square frame?

TURN PAGE OVER


6. (a) Alice falls down a rabbit hole and finds herself in a circular room with five doors of five
different sizes evenly spaced around the circumference. Alice tries keys in some or all of the
doors. She must leave no pair of adjacent doors untried. How many different sets of doors left
untried does Alice have to choose from? For example, Alice might try doors 1, 2, and 4 and leave doors 3 and
5 untried. There are no adjacent doors in the set of untried doors. Note: doors 1 and 5 are adjacent.
(b) Suppose the circular room in which Alice finds herself has nine doors of nine different sizes
evenly spaced around the circumference. Again, she is to try keys in some or all of the doors and
must leave no pair of adjacent doors untried. Now how many different sets of doors left untried
does Alice have to choose from?
7. The Caterpillar owns five different matched pairs of socks. He keeps the ten socks jumbled in
random order inside a silk sack. Dressing in the dark, he selects socks, choosing randomly without
replacement. If the two socks he puts on his first pair of feet are a mismatched pair and the two
socks he puts on his second pair of feet are a mismatched pair, then what is the probability that
the pair he selects for his third set of feet is a mismatched pair?

8. In the Queen’s croquet, a game begins with the ball at the bottom
wicket. All players hit the same ball. Each player hits the ball from
the place the previous player has left it. When the ball is hit from
the bottom wicket, it has a 50% chance of going to the top wicket
and a 50% chance of staying at the bottom wicket. When hit from
the top wicket, it has a 50% chance of hitting the goal post and a
50% chance of returning to the bottom wicket.
(a) If Alice makes the first hit and alternates hits with the Queen, what is the probability that Alice
is the first player to hit the goal post with the ball?
(b) Suppose Alice, the King, and the Queen take turns hitting the ball, with Alice playing first.
Now what is the probability that Alice is the first player to hit the goal post with the ball?
9. In the Queen’s croquet, as described in Problem 8, what is the probability that the ball hits the
goal post the nth time the ball is hit?

10. The March Hare invites 11 guests to a tea party. He randomly


assigns to each guest either tea or cake, but no guest receives both.
The guests know that the March Hare always does this, but they
never know which guests will receive tea and which will receive
cake. The guests decide to play a game. Each one tries to guess
! who of all 11 guests will get cake and who will get tea. If one guest
has more correct guesses than all the others, that guest wins. When
several guests tie for the most correct guesses, then the Dormouse selects one to be the winner by
selecting at random one of the guessers who has tied.
(a) All the guests make their guesses at random, perhaps by tossing a coin. What is the probability
that Tweedledee, the last guest to arrive, is the winner?
(b) Tweedledum is the first guest to arrive. What is the probability that one or the other of Twee-
dledee and Tweedledum is the winner?
(c) Suppose that instead of guessing randomly, Tweedledee always makes the guess opposite to
Tweedledum’s guess. If Tweedledum guesses that a guest will have tea, then Tweedledee will
guess cake. If all the other guests have guessed randomly, what is the probability that one or the
other of Tweedledee and Tweedledum is the winner? Your answer should be an explicit number,
but partial credit may be given for reasonable formulae.
END OF CONTEST
UNC MATHEMATICS CONTEST FINAL ROUND January 25, 2014
Three hours; no calculators or phones allowed. Put your answers below but also show your work for
each problem, in order and clearly numbered, on pages behind your answer sheet.

Name (Print): _____________________________________________________________ Grade __________

School:_________________________________ Teacher: __________________________________________

email: phone:

1. (10pts) Age of youngest=_________________ Sum of ages=______________.

2. (10 pts) _________________

3. (10 pts) x= __________________ y= ___________________

4. (10 pts) (a) ____________________ (b) _____________________ (c) _______________________

5. (10 pts) (a) ____________________ (b) _____________________

6. (10 pts) (a) 5 Doors:_____________ (b) 9 Doors_____________

7. (10 pts) __________________________

8. (a) (5 pts) __________________ (b) (5 pts) __________________

9. (10 pts) _________________________

10. (a) (1 pt) _____________ (b) (1 pt) _____________

(c) (8 pts) ___________________


_____________________________________________________________________________________________
I pledge on my honor that the work I am submitting is my own and that I have received no

inappropriate assistance. Signature: ____________________________________________________

I give permission to UNC Math Contest to use


photos taken at contest activities. Signature:____________________________________________________
UNC Math Contest Final Round: SOLUTIONS January 25, 2014

You may write answers in terms of the Fibonacci numbers Fn .


The Fibonacci numbers are F1 = 1, F2 = 1, F3 = 2, F4 = 3, F5 = 5, F6 = 8, . . .
They are defined by the equations F1 = F2 = 1 and, for n > 2, Fn = Fn−1 + Fn−2 .

1. The Duchess had a child on May 1st every two years until she had five children. This year the
youngest is 1 and the ages of the children are 1, 3, 5, 7, and 9. Alice notices that the sum of the ages
is a perfect square: 1 + 3 + 5 + 7 + 9 = 25. How old will the youngest be the next time the sum of
the ages of the five children is a perfect square, and what is that perfect square?
ANSWER: The youngest will be 16 and the perfect square is 100
SOLUTION: After n years the sum of the ages is
(1 + n) + (3 + n) + (5 + n) + (7 + n) + (9 + n) = 25 + 5n = 5(5 + n). For this to be a perfect
square, n must be a multiple of 5. Write n = 5k. Then 5(5 + n) = 5(5 + 5k ) = 25(1 + k ). This
will be a square when 1 + k is a square. The choice k = 0 corresponds n = 0, or this year. The
next choice is k = 3. This makes n = 15, so the youngest will be 16 and the sum of the ages will
be 25 + 5n = 25 + 75 = 100. Alternatively, one could also look for squares of the form 25 + 5k by
counting up from 25 by 5s, looking for squares: 25, 30, 35, 40, . . . The next perfect square you come
to is 100.

2. Define the Cheshire Cat function :) by


:)( x ) = − x if x is even and
:)( x ) = x if x is odd
Find the sum :)(1)+ :)(2)+ :)(3)+ :)(4) + . . . + :)(289)
ANSWER: 145
SOLUTION:1 − 2 + 3 − 4 + . . . + 287 − 288 + 289 = (1 − 2) + (3 − 4) + . . . (287 − 288) + 289 =
(−1) + (−1) + . . . + (−1) + 289
where there are 288/2 of the terms (−1), so the sum is −144 + 289 = 145.

1 1
3. Find x and y if =2 and =2
1+ 1x 1+ 1
1
1+
1+ 1y
ANSWER: x = −2 and y = −3/4
SOLUTION: 1 1 = 2 so 1 + 1x = 1/2 and 1
x = 1/2 − 1 = −1/2 and x = −2. Similarly for y.
1+ x

4. On the first slate, the Queen’s jurors write the number 1. On the second slate they write the
numbers 2 and 3. On the third slate the jurors write 4, 5, and 6, and so on, writing n integers on
the nth slate.
(a) What is the largest number they write on the 20th slate?
(b) What is the sum of the numbers written on the 20th slate?
(c) What is the sum of the numbers written on the nth slate?
ANSWERS: (a) 210 (b) 4010 (c) n(n2 + 1)/2
SOLUTIONS: (a) The last number on the nth slate is 1 + 2 + 3 + . . . + n = n(n + 1)/2. If you are
not familiar with this formula, you can add these integers up by writing two copies of the sum,
one in ascending order and one in descending order, like this:
1 + 2 + 3 + ... n
n + ( n − 1) + ( n − 2) + . . . + 1
Combine the first terms from each line and get n + 1. Combine the second terms from each line
and get n + 1 and so one. Pair by pair you get n + 1. You find that two copies of the sum will add
up to n copies of n + 1. Therefore, the original sum is n(n + 1)/2. This is called the Gauss trick for
adding an arithmetic sum and it applies in many situations. The last number on the 20th slate is
20(21)/2 = 10(21) = 210. It is also reasonable to write out the numbers explicitly and add them
up for parts (a) and (b).
(b) The last number on the slate before the nth slate is (n − 1)n/2 and the first number on the
nth slate is (n − 1)n/2 + 1. The numbers on the nth slate are the n numbers (n − 1)n/2 + 1, (n −
1)n/2 + 1, (n − 1)n/2 + 2, . . . (n − 1)n/2 + n. Adding these, we obtain n copies of (n − 1)n/2 plus
the sum of 1, 2, . . . n or
n2 (n − 1)/2 + 1 + 2 + . . . + n = n2 (n − 1)/2 + n(n + 1)/2
= (n/2)(n2 − n + n + 1) = (n/2)(n2 + 1).
This is, of course, the answer to (c), but it is a fine way to obtain the answer to (b). Putting n = 20
we get (20/2)(202 + 1) = 10(401) = 4010.
This question is similar to Problem 1 on the First Round, the question about chin-ups.

!
!!!!!
!
! 5. (a) The White Rabbit has a square garden with sides of length
!!!!!
!
! one meter. He builds a square cucumber frame in the center by
!
!
connecting each corner of the garden to the midpoint of a far side
of the garden, going clockwise, as shown in the diagram. What is
!
the area of the region that is enclosed in the inner square frame?
X
X (b) Suppose that the White Rabbit builds his square cucumber
frame by connecting each corner of the garden to a point a distance
x from the next corner, going clockwise, as shown in the diagram.
X Now what is the area of the region that is enclosed in the inner
X ! !
square frame?

(1− x )2
ANSWERS: (a) 1/5 (b) 1+ x 2

SOLUTIONS:

(a) The diagram is full of similar triangles and there are many ways
to use these similar triangles to solve the problem. One can decom-
pose the garden into similar triangles as shown in this diagram and
count that 4 of the 20 triangles lie inside the cucumber frame.
!
!

Another way to see the answer is to rearrange pieces of the garden


to form four copies of the inner square, one on each side of the cu-
cumber frame. This question is similar to Problem 2 , the question
about the BARN figure, on the First Round.

(b) If x is the reciprocal of an integer, or even just rational, we can again decompose into similar
triangles. For general x we can still use similar triangles to obtain the answer, as follows.
The upper left corner of the big square is a common vertex for three right triangles that are similar
but of different sizes and orientations. First tabulate what can be seen easily from the diagram:
Triangle Size Short Leg Long Leg Hypotenuse

Large x 1 1 + x2
Medium • • 1
Small • • x
Next solve for the missing entries (marked •) by multiplying by the appropriate proportionality
factors. From the rightmost column above we see that Medium = √ 1 2 × Large, and Small
1+ x
= √ x 2 × Large. Thus the completed table is
1+ x

Triangle Size Short Leg Long Leg Hypotenuse



Large x 1 1 + x2
Medium √ x √ 1 1
1+ x 2 1+ x 2
2
Small √x √ x x
1+ x 2 1+ x 2
Refer to the original cucumber frame diagram to see that the side length of the inner square is the
difference between the lengths of the long legs of the medium and small triangles: √1− x 2 . Now
1+ x
square this to obtain the area of the inner square.

6. (a) Alice falls down a rabbit hole and finds herself in a circular room with five doors of five
different sizes evenly spaced around the circumference. Alice tries keys in some or all of the
doors. She must leave no pair of adjacent doors untried. How many different sets of doors left
untried does Alice have to choose from? For example, Alice might try doors 1, 2, and 4 and leave doors 3 and
5 untried. There are no adjacent doors in the set of untried doors. Note: doors 1 and 5 are adjacent.
(b) Suppose the circular room in which Alice finds herself has nine doors of nine different sizes
evenly spaced around the circumference. Again, she is to try keys in some or all of the doors and
must leave no pair of adjacent doors untried. Now how many different sets of doors left untried
does Alice have to choose from?
ANSWERS: (a) 11 (b) 76
SOLUTIONS:
(a) It is practical to list the patterns for five doors and count that there are eleven of them:
TTTTT UTTTT TUTTT TTUTT TTTUT TTTTU UTUTT UTTUT TUTUT TUTTU TTUTU
(b) It is possible to list the 76 patterns for nine doors. One must be careful and systematic. How-
ever, one can instead count the number of patterns for lower numbers of doors and study the
results. Let Cn be the number of patterns with n doors in a circle and no pair of adjacent doors un-
tried. Question 10 from the First Round, about alphabet cookies, was essentially the same as this
question about doors, but with the doors arranged in a line rather than a circle. That is, the first
door and the last door were not counted as adjacent in that problem. Call the number of patterns
for n doors in the straight line case An (for alphabet cookies). By explicitly computing the cases
you can obtain the following table.
n 1 2 3 4 5
Cn 2 3 4 7 11
An 2 3 5 8 13
Fn 1 1 2 3 5
In the solutions to the First Round, we found that An = Fn+2 , the n + 2nd Fibonacci number.
From the table, we may also conjecture that Cn = Cn−1 + Cn−2 and that Cn = An−1 + An−3 =
Fn+1 + Fn−1 .
Proof: We will prove that C5 = A4 + A2 . The same argument works in general.
Case (i). Any of the A2 acceptable linear patterns such as UT can be extended to a linear pattern
of length five by adding the suffix TUT , creating UT TUT . Note that the linear pattern will still
be acceptable when it is converted to a circle. In this case the circular pattern contains the string
TUT . Designate the middle U as the last door in the circle, door 5.
Case (ii) Similarly, any of the A4 acceptable linear patterns such as UTTU can be extended to
length five by adding the suffix T , and this can then be converted into a circular pattern UTTU T
that is still acceptable. In this case the T is taken to be door 5.
Conversely every acceptable circular pattern of length five has its last door either ending in T or
U. If the ending is T we get a pattern described in case (ii). If it ends in U then that U must be
flanked on either side by T, so the pattern contains TUT . Thus we get a pattern described in case
(i).
ANOTHER SOLUTION: We can deduce directly a recursion relation for the Cn . We count patterns
with no pair of adjacent doors untried. Let Cn = the number of such patterns for n doors in a circle,
as before. We look for a way to write the Cn in terms of counts for smaller numbers of doors in
circles, this time.
Case (i) Suppose door n is tried and doors n − 1 and 1 are not both untried. There is a 1-1 corre-
spondence between such patterns and Cn−1 patterns: Given such a Cn pattern, removing door n
from the picture leaves an acceptable Cn−1 pattern. Given any acceptable Cn−1 pattern, adding a
door n, tried, produces an acceptable Cn pattern in which door n is tried and doors n − 1 and 1 are
not both untried.
Case (ii) Suppose either door n is tried and doors n − 1 and 1 are both untried or else door n is
untried. Note that if door n is untried then both door n − 1 and door 1 must be tried. There is
a 1-1 correspondence between these patterns and Cn−2 pattern: Given a Cn pattern as described,
removing doors n and n − 1 leaves an acceptable Cn−2 pattern. Given an acceptable Cn−2 pattern,
insert doors n − 1 and n. If door 1 is tried then door n − 1 should be tried and door n untried.
If door 1 is untried, then door n − 1 should be untried and door n tried. This will result in an
acceptable Cn pattern of the type described if n ≥ 4. If n = 3 then the resulting pattern UTU is not
acceptable, because the adjacent doors 1 and 3 remain untried.
We deduce that Cn = Cn−1 + Cn−2 , for n ≥ 4.
Find the first few Cn by listing the patterns of untried doors.
C2 = 3: UT TU TT
C3 = 4: UTT TUT TTU TTT
Now we can use recursion to find the next in the sequence:
C4 = 3 + 4 = 7, C5 = 4 + 7 = 11, C6 = 7 + 11 = 18, C7 = 11 + 18 = 29,
C8 = 18 + 29 = 47, C9 = 29 + 47 = 76
The numbers 1, 3, 4, 7, 11, . . . are known as the Lucas numbers.
7. The Caterpillar owns five different matched pairs of socks. He keeps the ten socks jumbled in
random order inside a silk sack. Dressing in the dark, he selects socks, choosing randomly without
replacement. If the two socks he puts on his first pair of feet are a mismatched pair and the two
socks he puts on his second pair of feet are a mismatched pair, then what is the probability that
the pair he selects for his third set of feet is a mismatched pair?
ANSWER: 112/125
SOLUTION: The Caterpillar draws the ten socks in any order with equal probability. Consider the
drawing of the first two pairs of socks.
There are several different acceptable color patterns that can occur when the first four socks are
drawn. The distinct acceptable color patterns are listed below, in order of increasing amounts of
color duplication. In each case, take inventory of what socks still remain in the sack. (Example.
In Case i, all of the first four socks are distinct colors, so four distinct letters are listed, with no
repetitions.)
First Sock Color Second Sock Third Sock Fourth Sock Remaining socks in sack
Case i A B C D { A, B, C, D, E, E}
Case ii.a A B A C { B, C, D, D, E, E}
Case ii.b A B B C { A, C, D, D, E, E}
Case iii.a A B A B {C, C, D, D, E, E}
Case iii.b A B B A {C, C, D, D, E, E}
We count the cases in which neither of those first pairs is a matched set, first classifying them
according to how many matched pairs remain in the sack after the first four socks are drawn: (i)
One matched pair remains. In this case, neither sock in the second pair matches a sock in the first
pair (ii) Two matched pairs remain. In this case, exactly one sock in the second pair matches a sock
in the first pair. (iii) Three matched pairs remain. In this case, both socks in the second pair match
socks in the first pair.
Case (i) Neither sock in the second pair matches a sock in the first pair. There are 10 possibilities
for the first sock and then 8 possibilities for a second sock that does not match the first one. There
are 6 choices for the third sock in which the third sock is different from the first two and then four
possibilities for the remaining sock. This is 10 × 8 × 6 × 4 draws fitting case (i).
Case (ii) Exactly one sock in the second pair matches a sock in the first pair. There are 10 possibil-
ities for the first sock and 8 possibilities for a second sock that does not match the first one. There
are 6 choices for the third sock in which the third sock is different from the first two. In this case,
the fourth sock must be chosen from one of the two that match socks in the first pair. There are
two choices for the third sock in which the third sock matches a sock from the first pair. Then there
are six choices for the fourth sock. In all, 10 × 8 × (6 × 2 + 2 × 6) = 10 × 8 × 24 draws are of the
pattern in case (ii).
Case (iii) Both socks in the second pair match socks in the first pair. There are, as in the other cases,
10 possibilities for the first sock and 8 possibilities for a second sock that does not match the first
one. There are 2 possibilities for the third sock and then just one possibility remains for the fourth
one: 10 × 8 × 2 draws fit this pattern.
Adding the number of equally likely draws in these three cases we obtain 10 × 8 × (24 + 24 + 2) =
10 × 8 × 2 × 12 + 12 + 1 = 10 × 8 × 2 × 25.
Therefore, given that the first two pairs are mismatched,
10×8×6×4 12
the probability of case (i) is 10 ×8×2×25 = 25 .
10×8×24 12
The probability of case (ii) is 10×8×2×25 = 25 .
10×8×2 1
The probability of case (iii) is 10×8×2×25 = 25 .

Now consider the drawing of the third pair of socks.


In case (i) the silk sock sack contains four unmatched socks and one matched pair. The probability
is 4/6 = 2/3 that one of the unmatched socks is next and then any of the remaining 5 are accept-
able for the last one. The probability is 2/6 = 1/3 that one of the matched pair is selected. Then
only 4 of the remaining 5 are acceptable for the last one.
10+4
Together this is 23 + ( 13 × 45 ) = 23× 5 1×4
×5 + 3×5 = 3×5 = 3×5 .
14

In case (ii) the sack contains two unmatched socks and two matched pairs. The probability is
2/6 = 1/3 that one of the unmatched socks is next. Then any of the remaining 5 are acceptable for
the last one. The probability is 4/6 = 2/3 that one of the matched socks is next. Following that, 4
of the remaining 5 are acceptable choices for the last one.
Together, this is 13 + ( 23 × 45 ) = 3×5 5 + 3×8 5 = 313
×5
In case (iii) the sack contains three matched pairs of socks. One of these matched socks will be the
fourth sock chosen and then any of the 5 remaining except its mate will be acceptable.
This is 45 = 312
×5 .
Combining the above, we obtain
(12×14)+(12×13)+12 +13+1)
( 12 14 12 13 1 12
25 × 3×5 ) + ( 25 × 3×5 ) + ( 25 × 3×5 ) = 25×3×5 = 12×(314
×125 = 4125
×28 = 112
125

8. In the Queen’s croquet, a game begins with the ball at the bottom
wicket. All players hit the same ball. Each player hits the ball from
the place the previous player has left it. When the ball is hit from
the bottom wicket, it has a 50% chance of going to the top wicket
and a 50% chance of staying at the bottom wicket. When hit from
the top wicket, it has a 50% chance of hitting the goal post and a
50% chance of returning to the bottom wicket.
(a) If Alice makes the first hit and alternates hits with the Queen, what is the probability that Alice
is the first player to hit the goal post with the ball?
(b) Suppose Alice, the King, and the Queen take turns hitting the ball, with Alice playing first.
Now what is the probability that Alice is the first player to hit the goal post with the ball?
ANSWERS: (a) 2/5 (b) 8/31
SOLUTIONS:
(a) Call the probability that Alice wins A and the probability that the Queen wins Q.
We obtain equations in A and Q as follows: Half the time, after Alice’s first hit, the ball remains
at the bottom wicket and it is as though the game restarts with the roles of Alice and the Queen
reversed. In this case, Alice’s probability is the same as the Queen’s overall probability of winning,
namely, Q. The other half of the time, Alice hits the ball to the top wicket. Half of that time, the
Queen hits the ball to the goal and Alice has no probability of winning. The other half of that
time, the Queen hits the ball back to the bottom wicket and Alice is in the same position she was
at the start. The game restarts with Alice and the Queen in their original roles, so her probability
of winning is again A. That is
A= Q2 + 0 + ( 12 × 12 × A) or 3A4 = 2.
Q

We get a second equation by considering the Queen’s situation: Half the time, after Alice’s first
hit, the ball remains at the bottom wicket and it is as though the Queen hits first. In this case, the
probability the Queen wins is the same as Alice’s overall probability of winning, namely A. The
other half of the time, the Queen first hits from the top wicket. In this case, the Queen hits the goal
post half the time and half the time she hits the ball back to the bottom wicket and it is as though
a new game starts. That is, Q, the probability that the Queen is first to hit the goal post, is equal to
(1/2) A (Alice has left the ball at the bottom wicket) plus (1/2) × (1/2) (Alice hits the ball to the
top wicket and the Queen hits the goal post with her first hit) plus (1/2) × (1/2) × Q (Alice hits
the ball to the top wicket and the Queen hits the ball back to the bottom wicket- now the Queen
has the same probability of winning as she had at the beginning.). That is, Q = A2 + 14 + Q4 .
Note that it is tempting to write down immediately that A + Q = 1. However, in order to use that,
one should really show that there is not a nonzero probability of neither player winning! Perhaps
there is a nonzero probability that the game never ends. (No, actually, there is not. But one should
check.) Solve the two simple equations for A. A = 2/5.
(b) As before, call the probability that Alice wins A and the probability that the Queen wins Q.
Call the probability that the King wins K. Reasoning as in (a), we obtain the three equations
A = K2 + Q4 ,
A 1 K
Q= 2 + 4 + 4, and
A 1 K
K= 2 + 8 + 8.
Solve these using your favorite method to arrive at A = 8/31.

9. In the Queen’s croquet, as described in Problem 8, what is the probability that the ball hits the
goal post the nth time the ball is hit?
F
ANSWER: Zero probability for n = 1 and n2−n 1 for n ≥ 2, where Fn−1 is the n − 1st Fibonacci
number.
SOLUTION: Let Bn , Tn , and Gn be the probability that the ball is at the bottom wicket, the top
wicket, and the goal post after n hits. Reasoning as in Problem 8, we obtain
B T
Bn = n2−1 + n2−1
B
Tn = n2−1
T
Gn = n2−1
B B
Then Bn = n2−1 + n4−2
T B B B G G
and Gn = n2−1 = n4−2 = 14 ( n2−3 + n4−4 ) = n2−1 + n4−2
The 2s suggest defining Hn = 2n Gn . Then
G G
Hn = 2n Gn = 2n ( n2−1 + n4−2 ) = 2n−1 Gn−1 + 2n−2 Gn−2 = Hn−1 + Hn−2 .
The Hn follow the rule for the Fibonacci numbers. We compute explicitly H1 = 2G1 = 0. H2 =
4G2 = 1. H3 = 8G3 = 1. Thus the H are almost the usual Fibonacci numbers. They are
0, 1, 1, 2, 3, . . . We have Hn = Fn−1 . Therefore, Gn , the probability that the ball hits the goal post on
Fn−1
the nth hit, is Gn = H 2n = 2n , for n ≥ 2.
n

10. The March Hare invites 11 guests to a tea party. He randomly


assigns to each guest either tea or cake, but no guest receives both.
The guests know that the March Hare always does this, but they
never know which guests will receive tea and which will receive
cake. The guests decide to play a game. Each one tries to guess
! who of all 11 guests will get cake and who will get tea. If one guest
has more correct guesses than all the others, that guest wins. When

several guests tie for the most correct guesses, then the Dormouse selects one to be the winner by
selecting at random one of the guessers who has tied.
(a) All the guests make their guesses at random, perhaps by tossing a coin. What is the probability
that Tweedledee, the last guest to arrive, is the winner?
(b) Tweedledum is the first guest to arrive. What is the probability that one or the other of Twee-
dledee and Tweedledum is the winner?
(c) Suppose that instead of guessing randomly, Tweedledee always makes the guess opposite to
Tweedledum’s guess. If Tweedledum guesses that a guest will have tea, then Tweedledee will
guess cake. If all the other guests have guessed randomly, what is the probability that one or the
other of Tweedledee and Tweedledum is the winner? Your answer should be an explicit number,
but partial credit may be given for reasonable formulae.
1 2 m +1 −1
ANSWERS: (a) 1/11 (b) 2/11 (c) 1023/5120 = ( m+ 1) 2m , where there are m guessers,
not including Tweedledee and Tweedeldum. Here m = 9.
SOLUTION: (a) The 11 guessers are equally likely to win. The probability of any single guesser
winning is 1/11.
(b) Again, the 11 guessers are equally likely to win. As there is just one winner, the probabilities
add: 1/11 of the time one wins and a different, non-overlapping, 1/11 of the time the second
guesser wins. The probability of one or the other of any two players winning is 1/11 + 1/11 =
2/11.
(c) Suppose Tweedledum chooses randomly, Tweedledee always chooses opposite to Tweedle-
dum, and there are m other guessers choosing randomly. In the given problem, m = 9. Each
guesser makes a string of eleven guesses, each guess either correct or incorrect. One or the other
of Tweedledum and Tweedledee will have guessed more than half correctly. We want to determine
the probability that this guess is the winning guess.
A random guesser is choosing among equally likely strings of the form TTCCTTTTCTC. Half the
strings have more correct guesses than incorrect guesses and half have more incorrect guesses
than correct guesses. (The case of an even number of guesses is slightly different, but we have
11 guesses in the problem before us.) Therefore, each random guesser has a probability 1/2 of
selecting a string with more correct than incorrect guesses. The probability that exactly k of the m
guessers choose strings with more correct guesses than incorrect guesses is therefore the same as
the probability of tossing exactly k heads in m tosses of a fair coin.
What is the probability of tossing exactly k heads in m tosses of a fair coin? There are 2m equally
likely outcomes HHTTHT . . .. Of these, (mk) = k!(mm!−k)! are strings with k heads. Thus the proba-
bility is ( 21m )(mk) = ( 21m ) k!(mm!−k)! .
One or the other of Tweedledee and Tweedledum has chosen one of the strings with more correct
guesses than incorrect guesses. The probability that exactly k of the other guessers have also
chosen strings with more correct guesses than incorrect guesses is ( 21m ) k!(mm!−k)! . Consider this
subgame: Tweedledee and Tweedledum together select one string from among the strings with
more correct than incorrect and k other guessers do the same. If there is a tie for the most correct
guesses, the Dormouse will break the tie randomly, as in the original game. The acceptable strings
are equally likely for each guesser. By symmetry, the guessers all have the same probability of
winning. There are k + 1 guessers and the rules force just one winner, so the probability that
any single guesser is the winner is k+1 1 . This is the great simplification that makes this problem
tractable. There is no need to look at the possible outcomes in detail. (We discuss below what to
do if one does not see this crucial simplification.)
Therefore, the probability that Tweedledee and Tweedledum’s better guess is the winner is
m
1 1 m! 1 m m!
∑ ( m)
k + 1 2 k!(m − k)!
= m∑
2 k =0 ( k + 1 ) ! ( m − k ) !
k =0

1 1 m ( m + 1) ! 1 1 m +1 ( m + 1 ) !
=( )( m ) ∑ =( )( m )( ∑ − 1)
m + 1 2 k=0 (k + 1)!(m + 1 − (k + 1))! m+1 2 j =0
j!(m + 1 − j)!

1 1 1 2m +1 − 1
=( )( m )(2m+1 − 1) = ( )
m+1 2 m+1 2m
10
1 2 −1 1 1024−1 1023
For m = 9 this is ( 10 ) 29 = ( 10 ) 512 = 5120

Observe that the number of guesses plays no role in the solution. In this problem the number of
guessers was the same as the number of guesses each guesser made, but the solution is the same
if the number of guesses is any odd number.

Suppose one does not see the simplification of considering the subgame among k guessers who
have guessed more than half correct. One can instead find the probability that the better guess
of Tweedledee and Tweedledum has j correct guesses and no other guesser has more, for each
j = 6, 7, 8, 9, 10, 11. Break down into cases. You will get sums of sums. Do not forget to break the
ties! This route is tedious and hard to complete with no electronic devices and in the time allowed.
However, correct work and formulae involving sums are worth some credit.

This problem was inspired by the article "The Evil Twin Strategy for a football pool" by DeStefano,
Doyle, and Snell, which can be found at
http://www.math.dartmouth.edu/∼doyle/docs/twin/twin/twin.html
(The tilde (∼) immediately preceding the name doyle in this web address may not copy paste
well. You may need to delete that symbol and put it in manually from your keyboard.)
END OF CONTEST
University of Northern Colorado Mathematics Contest

University Of Northern Colorado Mathematics Contest 2015-2016


Problems of First Round

1. How many positive integers less than 100 are multiples of 5 but not multiples of 2?

2. A zig-zag path has three straight segments that meet at right angles and have lengths 1, 6,
and 7, as shown in the diagram. What is the distance between the endpoints of the path?
That is, find the length of the dashed segment.
7

3. Three barrels currently contain 60 lbs, 50 lbs, and 10 lbs of sand. Sandy wants to equalize
the weight of sand in the barrels by redistributing the sand among the barrels. What is the
least total weight Sandy must move between barrels?

4. A spider has a web in the shape of the grid shown in the diagram. How many different
ways can the spider move from corner A to corner B by traveling along exactly seven
segments?
B

5. Find the area of the region in the x-y!plane that consists of the points (x, y ) !for which

x + y ≤ 3.

The notation x stands for the “absolute value” of x. That is x = x if x ≥ 0 and x = − x if


x ≤ 0.

Problems are duplicated and solved by Ming Song (msongmath@yahoo.com) 1


University of Northern Colorado Mathematics Contest

6. The points (2, 5) and (6, 5) are two of the vertices of a regular hexagon of side length two
on a coordinate plane. There is a line L that goes through the point (0, 0) and cuts the
hexagon into two pieces of equal area. What is the slope of line L? Express that slope as a
decimal number. A regular hexagon is a hexagon whose sides have equal length and whose
angles are congruent.

7. A rectangular sheet of paper whose dimensions are 12ʺ × 18ʺ is folded along a diagonal,
which creates the M- shaped region drawn at the right. Find the area of the shaded region.

Fold

8. A pyramid is built from solid unit cubes that are stacked in square layers which 3 × 3 = 9
cubes. The layer below that has 5 × 5 = 25 cubes, and so on, with each layer having two
more cubes on a side than the layer above it. The pyramid has a total of 12 layers. Find the
exposed surface area of this solid pyramid, including the bottom.

9. Yoda chooses two integers from


{1, 2, 3, 4, 5, 6, 7, 8, 9, 10, 11, 12, 13, 14, 15, 16, 17, 18, 19, 20 }.
He whispers one number to R2-D2. He whispers the other number to BB-8. He then
announces to both R2-D2 and BB-8, “R2-D2’s number is smaller than BB-8’s number. But,
they have the same number of divisors.”
R2-D2 says, “I don’t know BB-8’s number.”
Then BB-8 says, “With the information Yoda gave us, I did not know R2-D2’s number,
either. However, I now know R2-D2’s number.”

Problems are duplicated and solved by Ming Song (msongmath@yahoo.com) 2


University of Northern Colorado Mathematics Contest

What are R2-D2’s and BB-8’s numbers?


A divisor of an integer is an integer that divides into the integer with no remainder. For
example, the divisors of 24 are 1, 2, 3, 4, 6, 8, 12, and 24.

10. A treasure chest starts with 4 copper coins, 4 silver coins, and 5 gold coins. When Midas
randomly touches any colored coin, it magically disappears, and is replaced by two new
coins that are of the complementary colors. For example, if Midas touches a silver coin, it
transforms into one copper coin and one gold coin. After two consecutive random Midas
touches, what is the probability that the gold coins are still more numerous than either of
the other two colors?

11. A spider has a web in the shape of the grid shown in the diagram. How many different
ways can the spider move in a loop from corner A to corner B and back to corner A by
traveling along exactly fourteen distinct segments, if her path must never cross or touch
itself until it arrives back at corner A? The spider may not move along any segment more
than one time, and the spider’s path may not touch any intersection it has previously visited
until it returns to corner A. Count a clockwise loop as different from its counter-clockwise
counterpart.
B

Problems are duplicated and solved by Ming Song (msongmath@yahoo.com) 3


University of Northern Colorado Mathematics Contest

Problems with Solutions of First Round

1. How many positive integers less than 100 are multiples of 5 but not multiples of 2?
Answer: 10
Solution:
These are easy to list: 5, 15, 25, 35, 45, 55, 65, 75, 85, and 95. There are ten of them. One can
#100 !
also reason as follows: There are # ! = 20 numbers which are multiples of 5. Among them
$ 5 "
# 100 !
there are # ! = 10 numbers which are multiples of 2.
$5⋅ 2"
Therefore, the answer is 20 − 10 = 10 .

2. A zig-zag path has three straight segments that meet at right angles and have lengths 1, 6,
and 7, as shown in the diagram. What is the distance between the endpoints of the path?
That is, find the length of the dashed segment.
7

1
Answer: 10
Solution:
Let the zig-zag path be ABCD as shown.
C 7
E D

6 E

A B
1
Draw AE ⊥ DC intersecting line DC at E. Then AE = 6 and DE = 1 + 7 = 8 . Triangle AED is a
3-4-5 triangle.
Therefore, AD = 10

3. Three barrels currently contain 60 lbs, 50 lbs, and 10 lbs of sand. Sandy wants to equalize
the weight of sand in the barrels by redistributing the sand among the barrels. What is the
least total weight Sandy must move between barrels?
Answer: 30

Problems are duplicated and solved by Ming Song (msongmath@yahoo.com) 4


University of Northern Colorado Mathematics Contest

Solution:
Since one of three barrels contains 10 lbs, at least 30 lbs must be moved into this barrel.
30 lbs is attainable: move 20 lbs from the first barrel into the third barrel and move 10 lbs from
the second barrel into the third barrel. Then all barrels have 40 lbs each.
The answer is 30.

4. A spider has a web in the shape of the grid shown in the diagram. How many different
ways can the spider move from corner A to corner B by traveling along exactly seven
segments?
B

A
Answer: 35
Solution 1:
We mark the numbers:
4 10 20 35
1 B
3 6 10
1 15

2 3 4
1 5

A
1 1 1 1 1

A number at a cross-section indicates the number of shortest routes (with exactly 7 segments)
from A to the cross-section.
The answer is 35.
Solution 2:
From A to B there are 7 blocks in a row: four horizontal and three vertical. In 7 blocks there are
&7#
$$ !! ways to choose 3 blocks to be vertical.
% 3"
&7#
Therefore, the answer is $$ !! = 35.
3
% "

Problems are duplicated and solved by Ming Song (msongmath@yahoo.com) 5


University of Northern Colorado Mathematics Contest

5. Find the area of the region in the x-y!plane that consists of the points (x, y ) !for which

x + y ≤ 3.

The notation x stands for the “absolute value” of x. That is x = x if x ≥ 0 and x = − x if


x ≤ 0.
Answer: 18
Solution:
The region is shaded below:
y
x+y=3 –x + y = 3
3

x
–3 3

–3
x – y = 3 –x – y = 3

1
The area is ⋅ 6 ⋅ 6 = 18 .
2

6. The points (2, 5) and (6, 5) are two of the vertices of a regular hexagon of side length two
on a coordinate plane. There is a line L that goes through the point (0, 0) and cuts the
hexagon into two pieces of equal area. What is the slope of line L? Express that slope as a
decimal number. A regular hexagon is a hexagon whose sides have equal length and whose
angles are congruent.

Answer: 1.25

Problems are duplicated and solved by Ming Song (msongmath@yahoo.com) 6


University of Northern Colorado Mathematics Contest

Solution:
Any line passing through the center of the regular hexagon divides this hexagon into two pieces
of equal area.

(2, 5) (4, 5) (6, 5)

5
The center of the hexagon is (4, 5) . The line L passes (0, 0) and (4, 5) . The slope is = 1.25 .
4

7. A rectangular sheet of paper whose dimensions are 12ʺ × 18ʺ is folded along a diagonal,
which creates the M- shaped region drawn at the right. Find the area of the shaded region.

Fold

Answer: 138
Solution:
D

D A
E
Fold
C B

C B

A
Let ABCD be the rectangle. The folding is along diagonal BC. After folding, let AC and BD
intersect at E.

Problems are duplicated and solved by Ming Song (msongmath@yahoo.com) 7


University of Northern Colorado Mathematics Contest

Let AE = DE = x . Then CE = 18 − x .

In right triangle CDE we have (18 − x )2 = x 2 + 122 . Solving for x, we obtain x = 5 .

1 1
The area of triangle BAC is ⋅ 12 ⋅ 18 = 108 , and the area of triangle BAC is ⋅ 12 ⋅ 5 = 30 .
2 2
The total area of the shaded region is 108 + 30 = 138 .
A quick student may find the 5-12-13 triangle immediately.

8. A pyramid is built from solid unit cubes that are stacked in square layers which 3 × 3 = 9
cubes. The layer below that has 5 × 5 = 25 cubes, and so on, with each layer having two
more cubes on a side than the layer above it. The pyramid has a total of 12 layers. Find the
exposed surface area of this solid pyramid, including the bottom.
Answer: 1634
Solution:
In the 12th layer there are 23 × 23 cubes.
The pyramid looks like

From one lateral side we see

From the top or bottom we see the 23 × 23 square grid. Therefore, the total exposed surface area
is 4 ⋅ (1 + 3 + 5 + ! + 23) + 2 ⋅ 232 = 4 ⋅ 122 + 2 ⋅ 232 = 1634 .

9. Yoda chooses two integers from


{1, 2, 3, 4, 5, 6, 7, 8, 9, 10, 11, 12, 13, 14, 15, 16, 17, 18, 19, 20 }.
He whispers one number to R2-D2. He whispers the other number to BB-8. He then
announces to both R2-D2 and BB-8, “R2-D2’s number is smaller than BB-8’s number. But,
they have the same number of divisors.”
R2-D2 says, “I don’t know BB-8’s number.”
Then BB-8 says, “With the information Yoda gave us, I did not know R2-D2’s number,
either. However, I now know R2-D2’s number.”
What are R2-D2’s and BB-8’s numbers?

Problems are duplicated and solved by Ming Song (msongmath@yahoo.com) 8


University of Northern Colorado Mathematics Contest

A divisor of an integer is an integer that divides into the integer with no remainder. For
example, the divisors of 24 are 1, 2, 3, 4, 6, 8, 12, and 24.
Answer: 12 and 20
Solution:
Let us list the number f of factors for each number n from 1 to 20:

n 1 2 3 4 5 6 7 8 9 10 11 12 13 14 15 16 17 18 19 20
f 1 2 2 3 2 4 2 4 3 4 2 6 2 4 4 5 2 6 2 6
There is only one number having one factor, which is 1. Obviously 1 is not a candidate.
There is only one number having 5 factors, which is 16. 16 is not a candidate.
There are only two numbers having 3 factors, which are 4 and 9. If one holds 4 or 9, the other can
immediately know what the component holds. So 4 and 9 are not candidates.
All prime numbers each have 2 factors.
Assume that both R2-D2 and BB-8 hold two primes.
R2-D2 cannot hold 19 because BB-8’s number is larger.
R2-D2 cannot hold 17 because R2-D2 can claim BB-8’s number immediately.
After R2-D2 says “I don’t know BB-8’s number”, BB-8 knows that R2-D2’s number is one of 2,
3, 5, 7, 11, 13. BB-8 cannot conclude what R2-D2’s number is.
So they don’t hold two prime numbers.
They are five numbers having 4 factors, which are 6, 8, 10, 14, 15. By the same reasoning, these
are not candidates.
Now only 12, 18, and 20 are remaining, which each have 6 factors. R2-D2 and BB-8 hold two of
them.
If one holds 18, he can claim the component’s number immediately. It is not the case.
Therefore, R2-D2’s number is 12, and BB-8’s number is 20.
R2-D2 holds 12. Since there are two larger numbers 18 and 20 having the same factor as 12, R2-
D2 cannot conclude what BB-8’s number is.
BB-8 holds 20. Since there are two smaller numbers 12 and 18 having the same factor as 20, at
the beginning BB-8 cannot conclude what R2-D2’s number is.
After R2-D2 says “I don’t know BB-8’s number”, BB-8 knows that R2-D2’s number is 12.
Therefore, the answers are 12 and 20.

10. A treasure chest starts with 4 copper coins, 4 silver coins, and 5 gold coins. When Midas
randomly touches any colored coin, it magically disappears, and is replaced by two new
coins that are of the complementary colors. For example, if Midas touches a silver coin, it
transforms into one copper coin and one gold coin. After two consecutive random Midas
touches, what is the probability that the gold coins are still more numerous than either of
the other two colors?
If we understand “more than either of the other two colors” as “more than any of the other two
colors”, we have the following solution.

Problems are duplicated and solved by Ming Song (msongmath@yahoo.com) 9


University of Northern Colorado Mathematics Contest

32
Answer:
91
Solution:
Case 1: the first touch is copper
4
The probability for this to happen is . Then we will have 3 copper, 5 silver, and 6 gold coins.
13
If the second touch is gold, then the gold cannot be more than silver coins. For the gold to be
more than any of the two other colors, the second touch can be copper and silver. The probability
3+ 5 4
for this to happen is = .
3+ 5+ 6 7
In this case, the probability that the gold are still more than either of the other two colors is
4 4 16
⋅ = .
13 7 91
Case 2: the first touch is silver
16
Because of the symmetry between copper and silver, the probability in this case is as well.
91
Case 3: the first touch is gold
5
The probability for this to happen is . Then we will have 5 copper, 5 silver, and 4 gold coins.
13
For whatever the second touch is, the gold cannot be more than either of the other two colors.
16 16 32
Therefore, the answer is + = .
91 91 91
If we understand “more than either of the other two colors” as “more than one of the other two
colors”, we have the following solution.
Answer: 81/91
Solution:
Case 1: the first touch is copper
4
The probability for this to happen is . Then the second touch can be any, which leads to that
13
the gold are still more than either of the other two colors.
Case 2: the first touch is silver
4
Because of the symmetry between copper and silver, the probability in this case is as well.
13
Case 3: the first touch is gold
5
The probability for this to happen is . Then we will have 5 copper, 5 silver, and 4 gold coins.
13
If the second touch is copper or silver, the gold are more than either of the other two colors.
5 10 25
In this case, the probability is ⋅ = .
13 14 91

Problems are duplicated and solved by Ming Song (msongmath@yahoo.com) 10


University of Northern Colorado Mathematics Contest

4 4 25 81
Therefore, the answer is + + = .
13 13 91 91

11. A spider has a web in the shape of the grid shown in the diagram. How many different
ways can the spider move in a loop from corner A to corner B and back to corner A by
traveling along exactly fourteen distinct segments, if her path must never cross or touch
itself until it arrives back at corner A? The spider may not move along any segment more
than one time, and the spider’s path may not touch any intersection it has previously visited
until it returns to corner A. Count a clockwise loop as different from its counter-clockwise
counterpart.
B

A
Answer: 100
Solution 1:
In a short time, we may not able to find an elegant solution. I would like to find the answer by
listing since 3 and 4 are not large numbers.
Let us list.
In the following diagrams, the back routes from B to A are red, and the green shaded rectangles
help in counting.
Case 1:
B

A
C
Pay attention to C and D.
& 5#
The number of routes which don’t cross or touch the red route is $$ !! = 10 .
% 2"
Case 2:

Problems are duplicated and solved by Ming Song (msongmath@yahoo.com) 11


University of Northern Colorado Mathematics Contest

A
C
& 5# & 2#
The number of routes which don’t cross or touch the red route is $$ !! − $$ !! = 9 .
% 2" % 2"
Case 3:
B

A
C

& 5# & 3#
The number of routes which don’t cross or touch the red route is $$ !! − $$ !! = 7 .
% 2" % 2"
Case 4:
B

A
C

& 5# & 4#
The number of routes which don’t cross or touch the red route is $$ !! − $$ !! = 4 .
% 2" % 2"

Problems are duplicated and solved by Ming Song (msongmath@yahoo.com) 12


University of Northern Colorado Mathematics Contest

Case 5:
B

A
E
Pay attention to E and D.
& 4#
The number of routes which don’t cross or touch the red route is $$ !! = 6 .
% 2"
Case 6:
B

A
F
Pay attention to F and D.
& 3#
The number of routes which don’t cross or touch the red route is $$ !! = 3 .
% 2"
Case 7:
B

A
Pay attention to F and D.
There is only 1 route which doesn’t cross or touch the red route.

Problems are duplicated and solved by Ming Song (msongmath@yahoo.com) 13


University of Northern Colorado Mathematics Contest

Case 8:
B

A
E

& 4# & 2#
The number of routes which don’t cross or touch the red route is $$ !! − $$ !! = 5 .
% 2" % 2"
Case 9:
B

A
E

& 4# & 3#
The number of routes which don’t cross or touch the red route is $$ !! − $$ !! = 3.
% 2" % 2"
Case 10:
B

A
F

& 2#
The number of routes which don’t cross or touch the red route is $$ !! = 2 .
%1"
The total number of the routes which don’t cross or touch a given back route is
10 + 9 + 7 + 4 + 6 + 3 + 1 + 5 + 3 + 2 = 50 .
Consider that the forward route and back route can be switched.
The answer is 50 ⋅ 2 = 100 .

Problems are duplicated and solved by Ming Song (msongmath@yahoo.com) 14


University of Northern Colorado Mathematics Contest

Solution 2:
I am not satisfied with a solution by listing. It take me quite much time to find the following
solution.
We assume two spiders moving from A to B. We count the pairs of their paths which don’t cross
or touch each other.
Look at the figure:
F
B

A
C
One spider must be from C to D, and the other must be from E to F such that their paths don’t
cross or touch each other.
We will count the pairs in each of which two paths cross or touch each other.
The following figure shows a pair of path X (green) from C to D and path Y (blue) from E to F
which cross each other.
F
B

D
Y
E
X
A
C
Mark the point where the two paths first meet (the black point).
F
B

D
Y
E
X
A
C

Problems are duplicated and solved by Ming Song (msongmath@yahoo.com) 15


University of Northern Colorado Mathematics Contest

Switch the parts of path X and path Y after the black point.
F
B

D
V
E
U
A
C
We have two paths: one (red) is from C to F and the other (pink) is from E to D.
Red path U = the part of path X before the black point + the part of path Y after the black point
Pink path V = the part of path Y before the black point + the part of path X after the black point
Let us look at another example: a pair of path X (green) from C to D and path Y (blue) from E to
F which touch each other.
F
B
Y
D
X
E

A
C
Mark the point where the two paths first meet (the black point).
F
B
Y
D
X
E

A
C
Switch paths:
F
B
V
D

E
U
A
C

Problems are duplicated and solved by Ming Song (msongmath@yahoo.com) 16


University of Northern Colorado Mathematics Contest

We have two paths: one (red) is from C to F and the other (pink) is from E to D.
Red path U = the part of path X before the black point + the part of path Y after the black point
Pink path V = the part of path Y before the black point + the part of path X after the black point
We see that for any pair of paths: one from C to D and the other one from E to F, which cross or
touch each other, we have a pair of paths: one from C to F and the other from E to D.
Now look at a pair of path U (red) from C to F and path V (pink) from E to D. The two paths
must cross each other.
F
B
U
D

E
V
A
C
Mark the point where the two paths first meet (the black point).
F
B
U
D

E
V
A
C
Switch paths:
F
B
Y
D

E
X
A
C
Green path X = the part of path U before the black point + the part of path V after the black point
Blue path Y = the part of path V before the black point + the part of path U after the black point

Problems are duplicated and solved by Ming Song (msongmath@yahoo.com) 17


University of Northern Colorado Mathematics Contest

Take a look at another pair of path U (red) from C to F and path V (pink) from E to D.
F
B

D
V
E
U
A
C
Mark the first point where the two paths meet (the black point).
F
B

D
V
E
U
A
C
Switch paths:
F
B

D
Y
E
X
A
C
Green path X = the part of path U before the black point + the part of path V after the black point
Blue path Y = the part of path V before the black point + the part of path U after the black point
We see that for any pair of paths: one from C to F and the other one from E to D, we have a pair
of paths: one from C to D and the other from E to F, which cross or touch each other.
We find the one-to-one correspondence.
& 5# & 5#
From C to D there are $$ !! paths, and from E to F there are $$ !! paths as well. So there are
%2" %2"
& 5# & 5#
$$ !! ⋅ $$ !! pairs.
% 2" % 2"
& 5# & 5# & 5# & 5#
From C to F there are $$ !! paths, and from E to D there are $$ !! paths. So there are $$ !! ⋅ $$ !!
% 3" % 1" % 3" % 1 "
pairs.

Problems are duplicated and solved by Ming Song (msongmath@yahoo.com) 18


University of Northern Colorado Mathematics Contest

& 5# & 5#
That is, there are $$ !! ⋅ $$ !! pairs of paths: one from C to D and the other from E to F, which cross
% 3" % 1 "
or touch each other.
Therefore, the number of pairs of paths: one from C to D and the other from E to F, which don’t
cross or touch each other is
( 5 % ( 5 % ( 5% ( 5%
&& ## ⋅ && ## − && ## ⋅ && ## = 10 ⋅10 − 10 ⋅ 5 = 50 .
' 2 $ ' 2 $ ' 3$ ' 1 $
Consider that two spiders can switch their paths. The answer is
2 ⋅ 50 = 100 .
In general, for the m × n grid, the answer is
&, m − 1 + n − 1) 2 , m − 1 + n − 1) , m − 1 + n − 1)#
2$** '' − ** '' ⋅ ** ''! .
$%+ m − 1 ( + m ( + m − 2 (!"

Problems are duplicated and solved by Ming Song (msongmath@yahoo.com) 19


Twenty-fourth Annual UNC Math Contest Final Round Solutions Jan 2016

Rules: Three hours; no electronic devices. The positive integers are 1, 2, 3, 4, . . .

1. Pythagorean Triplet The sum of the lengths of the three sides of a right triangle is 56. The
sum of the squares of the lengths of the three sides of the same right triangle is 1250. What is
the area of the triangle?

Answer: The area is 84.


Solution Call the lengths of the three sides a, b, and c, with c the hypotenuse. Then a + b + c =
56, a2 + b2 + c2 = 1250, and a2 + b2 = c2 . Conclude 2c2 = 1250, c2 = 625, and c = 25.
Subtract c from both sides of the equation a + b + c = 54 to conclude a + b = 56 − 25 = 31.
Subtract c2 from both sides of a2 + b2 + c2 = 1250 to conclude a2 + b2 = 1250 − 625 = 650. You
can solve these equations for a, b, and c, but what we are asked for is the area of the triangle,
and that is 12 ab. We can take a small shortcut. Compute ( a + b)2 = a2 + 2ab + b2 . Use the facts
that a + b = 31 and a2 + b2 = 625 to conclude 312 = 625 + 2ab. Thus, 2ab = 312 − 625 =
961 − 625 = 336. The area is 12 ab = 14 336 = 84.

2. Fearsome Foursome Factorial Find the complete prime factorization of

(4!)!
[(3!)!]4
(The answer will be a product of powers of eight distinct primes.)

Answer: 26 × 32 × 73 × 112 × 13 × 17 × 19 × 23.


Solution One can write the expressions out carefully and cancel common factors systemati-
cally. However, this can also be solved quickly using Legendre’s simple but useful formula
for finding the prime factorization of a factorial. The formula makes use of the greatest integer
function [[ ]] that rounds down to the nearest integer. As an example, the power of 2 in the
prime factorization of 24!, computed by Legendre’s method, is
[[24/2]] + [[24/22 ]] + [[24/23 ]] + [[24/24 ]] + . . . = 12 + 6 + 3 + 1 = 22

3. Polyhedral Die A cube that is one inch wide has


had its eight corners shaved off. The cube’s vertices
have been replaced by eight congruent equilateral tri-
angles, and the square faces have been replaced by six
congruent octagons. If the combined area of the eight
triangles equals the area of one of the octagons, what is
that area? (Each octagonal face has two different edge
lengths that occur in alternating order.)
√ √
2 √3 12−2 3
Answer: = 11 .
1+2 3
Solution Let x be the amount that is cut off of each edge of the original square at the corners,

so that one side length on the resulting octahedron is 1 − 2x and the other side length is 2x.
Find the area of each octagon by subtracting the area of the four cut off corners from area of
the square. Each cut off corner is a right isosceles triangle with legs of length x. The area of
each of these cut off corners is 12 x2 . Thus the area of one of the octagons is 1 − 2x2√
. Each of the
√ 3

new triangular faces is an equilateral triangle
√ √ √
with side√
length 2x and height 2 2x. The
1 3 3 2
area of each equilateral corner face is 2 2 2x 2x = ( 2 ) x . Set eight of these equal to the
area of one octagon: √
8 3 2
x = 1 − 2x2 .
2
Solve the equation for x. Since what we are asked for is the area 1 − 2x2 , we might as well
solve for x2 .

4. Number Sieve How many positive integers less than 100 are divisible by exactly two of the
numbers 2, 3, 4, 5, 6, 7, 8, 9? For example, 75 is such a number: it is divisible by 3 and by 5, but
is not divisible by any of the others on the list. (If you show the integers you find, then you
may be assigned partial credit if you have accurately found most of them, even if you do not
find all of them.)

Answer. There are eighteen such numbers:


4, 9, 10, 14, 15, 21, 27, 35, 44, 50, 52, 68, 75, 76, 81, 92, 98, 99
Solution A reasonable way to do this one is to check each integer. Many of the integers will
be rejected quickly- the multiples of 4, perfect squares, and the primes, for instance.
5. Rock and Roll. Zeus has decreed that Sisyphus must spend each day removing all the rocks
in a certain valley and transferring them to Mount Olympus. Each night, each rock Sisyphus
places on Mount Olympus is subject to the whims of Zeus: it will either be vaporized (with
probability 10%), be rolled back down into the valley (with probability 50% ), or be split by a
thunderbolt into two rocks that are both rolled down into the valley (with probability 40%).
When the sun rises, Sisyphus returns to work, delivering rocks to Olympus. At sunrise on
the first day of his punishment, there is only one rock in the valley and there are no rocks
on Mount Olympus. What is the probability that there are exactly two rocks in the valley at
sunrise on the third day? (If a rock is vaporized, it is gone.)

Answer: 0.332
Solution. A tree diagram shows that the possibilities are
(i) the first rock is preserved the first night and split the second night (.5)(.4) = .2;
(ii) the first rock is split into two the first night, and the second night both rocks are preserved
(.4)(.5)(.5) = .1;
(iii) the first rock is split into two the first night, and the second night one of those two is split
and the other is vaporized: 2(.4)(.1)(.4) = .032.
In total, .2 + .1 + .032 = 0.332
6. Rock and Roll Forever? (a) Given the situation in Question 5, what is the probability that
Sisyphus must labor forever? That is, if Sisyphus begins with one rock in the valley on his
first morning, what is the probability that the Olympian rocks are never all vaporized?
(b) Suppose that the whims of Zeus obey the following rules instead: a rock will either be
vaporized (with probability 10%), be rolled back down into the valley (with probability 20%),
be split by a thunderbolt into two rocks that are both rolled down into the valley (with prob-
ability 30%), or be split by two thunderbolts into three rocks that are all rolled down into the
valley (with probability 40%). Now what is the probability that Sisyphus must labor forever?

Answer: (a) 34 .

Solution Let P be the probability that starting with one rock, the subsequent rocks are even-
tually all vaporized. Observe that if Zeus gives you two rocks, the probability that the strings
of successors of both rocks disappear eventually is P2 , because these are independent events.
Now use recursiveness to compute the probability from the viewpoint of the second day. Note
P = .1 (1/10 of the time that first rock is gone on the second day)
+.5P (half the time you start over with one rock on the second day and P is the probability
that it and all its successors are eventually gone )
+.4P2 (4/10 of the time you get two rocks and the probability that the strings of succes-
sors of both rocks disappear eventually is P2 ) .
Solve this quadratic for P. You get two solutions: P = 1 and P = .25.
Another approach: The polynomial p( x ) = .1 + .5x + .4x2 encodes the whims of Zeus: the
coefficient of x k in p( x ) gives the probability of there being k rocks in the valley at sunrise on
the second day. The coefficient of x k in the composite function p( p( x )) gives the probability
of there being k rocks in the valley at sunrise on the third day. The coefficient of x k in the
three-fold composition p( p( p( x ))) gives the probability of there being k rocks in the valley at
sunrise on the fourth day, and so on.
The constant term of the polynomial is telling the probability that there are zero rocks in the
valley, that is, the probability that all the rocks have been vaporized.
Look at the (very simple) graph of y = p( x ) and y = x to track down p(0) and then p( p(0))
and so on: use a cobweb picture to find that p( p( p(. . . (0) . . .)))) is the fixed point x where
x = p( x ). Solve to find x = 14 . Conclude that the complementary event that the labor never
ends is 34 .

Out[2]=

Figure 1: The curves cross at the fixed points x = p( x ) where x = .25 and x = 1

15− 65
(b) Answer: 8 .

Solution This time you solve P = .1 + .2P + .3P2 + .4P3 . Observe that P = 1 is one solu-
tion, so divide by the known linear factor P − 1√and solve the remaining quadratic,√which is
−7± 65 −7+ 65
4P2 + 7P − 1 = 0. The solutions are P = 8 √. Choose the positive P = 8 . The
15− 65
complementary probability desired is 1 − P = 8 .

7. Evaluate

4n
S= ∑ ( n2 − 1)2
n =2

Answer: 54
Solution Using the partial fraction decomposition (n24n −1)2
= (n−11)2 − (n+11)2 and then expand-
ing out the sum, we see a telescoping pattern of many canceling paired terms.
!
( 112 − 312 ) + ( 212 − 412 ) + ( 312 − 512 ) + ( 412 − 612 ) + . . .
The only terms that do not cancel are the 112 and the 212 . The final infinite sum is ( 112 + 212 ) =
1 + 14 = 54 .

A 8. Tree Each circle in this tree diagram is to be as-


signed a value, chosen from a set S, in such a way
that along every pathway down the tree, the assigned
B C values never increase. That is, A ≥ B, A ≥ C,
C ≥ D, C ≥ E, and A, B, C, D, E ∈ S. (It is permis-
sible for a value in S to appear more than once in the
D E tree.)
(a) How many ways can the tree be so numbered, using
only values chosen from the set S = {1, . . . , 6}?
(b) Generalize to the case in which S = {1, . . . , n}. Find a formula for the number of ways the
tree can be numbered.
For maximal credit, express your answer in closed form as an explicit algebraic expression in n.
Answers.
A. 994
5 4 5n3 5n2
1
n(n + 1)(n + 2) 8n2 + 11n + 1 = n15 + 7n n

B. 120 24 + 12 + 24 + 60
Solution Once values for A and C are selected, the restrictions on the remaining values imply
that they can be chosen in this many ways:
(i) B can be chosen A ways;
(ii) D and E can be chosen independently, each in C ways.
Thus the total number of choices is T = ∑ A =n C= A 2
A =1 ( A ∑ C =1 C ).
The inner sum simplifies to 6 A(1 + A)(1 + 2A) and the total sum is T = ∑nA=0 p( A) where
1

1 2
p( A) = A (1 + A)(1 + 2A)
6
This polynomial can be summed by a variety of standard methods, such as the method of un-
determined coefficients, Bernoulli number expansions, or the Newton interpolation formula
based on forward differences.
Below we outline a method based on the idea of expressing p( A) as a linear combination of
binomial coefficients and then applying the Hockey Stick Summation Lemma to each term.
The Hockey Stick Lemma states that the entries in Pascal’s Triangle satisfy
for any fixed k ≥ 0,
N    
A+k N+k+1
∑ k
=
k+1
A =0
This is useful for summation of polynomials. As an example, observe that the binomial coeffi-
( A)( A+1)( A+2)
cient ( A+ 2
3 ) = 3! has a numerator expressible as a product of linear factors whose
roots are consecutive integers. Incidentally, this product formula allows us to extend the binomial
coefficients in a meaningful way, even to cases where A is negative.
This example illustrates a basic principle: Every binomial coefficient is a product of linear factors
that are shifted by consecutive integers. Conversely, any such product of linear factors is expressible as
a binomial, hence can be summed by the Hockey Stick Lemma.
With this in mind we rewrite the factorization
p( A) = 16 [( A − 1) + 1] A(1 + A)(1 + 2A)
= 16 [( A − 1) A( A + 1)(1 + 2A)] + 16 A(1 + A)(1 + 2A)
= 16 [( A − 1) A( A + 1)(2( A + 2) − 3)] + 61 A(1 + A)(2( A + 2) − 3)
so that it has been expressed in terms of products of consecutive linear factors. Some further
multiplication gives
p( a) = − 21 ( a − 1) a( a + 1) − 21 a( a + 1) + 13 ( a − 1) a( a + 2)( a + 1) + 31 a( a + 2)( a + 1) which
preserves the structure of the products of consecutive factors.
Now for example, the first term involving ∑na=2 21 ( a − 1) a( a + 1) can be evaluated by changing
the summation index to k = a − 2 and then using the Hockey Stick:
−2 3 n −2 k +3 n −2+4
∑nk= 0 3! ( k + 1)( k + 2)( k + 3) = 3 ∑k =0 ( 3 ) = 3( 4 ) . The same method works for the
other three terms to be summed.

9. Chess Masters Four identical white pawns and four


identical black pawns are to be placed on a standard
8 × 8, two-colored chessboard. How many distinct ar-
rangements of the colored pawns on the colored board
are possible?
No two pawns occupy the same square. The color of a pawn
need not match the color of the square it occupies, but it might.
You may give your answer as a formula involving factorials or
combinations: you are not asked to compute the number.

64!
Answer: There are 56!4!4! arrangements of the colored pawns on the standard board.
64!
Solution First choose four squares from the 64 on the board for the black pawns. There are 60!4!
possibilities. Then choose four squares from the 60 remaining squares for the white pawns.
64! 60! 64!
This gives 60!4! 56!4! = 56!4!4! arrangements.
10. Chess Wallpaper How many distinct plane wallpa-
per patterns can be created by cloning the chessboard
arrangements described in Question 9?
Each periodic wallpaper pattern is generated by this method:
starting with a chessboard arrangement from Question 9 (the
master tile), use copies of that tile to fill the plane seamlessly,
placing the copies edge-to-edge and corner-to-corner. Note that
the resulting wallpaper pattern repeats with period 8, horizon-
tally and vertically.
When the tiling is done, the chessboard edges and corners van-
ish, leaving only an infinite periodic pattern of black and white
pawns visible on the wallpaper.

Regard two of the infinite wallpaper patterns as the same if and only if there is a plane translation that
slides one wallpaper pattern onto an exact copy of the other one. You may slide vertically, horizontally, or a
combination of both, any number of squares. (Rotations and reflections are not allowed, just translations.)
Note that the wallpaper pattern depicted above can be generated by many different master tiles (by regard-
ing any square 8 × 8 portion of the wallpaper as the master tile chessboard). The challenge is to account for
such duplication. Remember that each master tile has exactly four pawns of each color.
You may give your answer as an expression using factorials and/or combinations (binomial coefficients).
You are not asked to compute the numerical answer.

Answer: There are


1 h 64!  32!   16! i
+3 + 12 =
32 56!4!4! 28!2!2! 14!1!1!
h   i h  i h i
1 64 60 32 30 16 15 16 15 1 32 30 16 15 1 16 15
32 ( 4 )( 4 ) − 3 ( 2 )( 2 ) − 3 ( 1 )( 1 ) − 7 ( 1 )( 1 ) + 16 3 ( 2 )( 2 ) − 3 ( 1 )( 1 ) + 8 7 ( 1 )( 1 )

= 9, 682, 216, 530 different wallpaper patterns.


Solution By "8 × 8 pattern" we will mean a pattern as described in the question, that is, a
standard 8 × 8 chessboard that has a light square in the upper left corner and that contains
four black pawns and four white pawns.
The difficulty in counting the wallpaper patterns generated by these 8 × 8 patterns is that two
different 8 × 8 patterns sometimes produce the same wallpaper pattern. Moreover, the num-
ber of distinct 8 × 8 patterns that produce a particular wallpaper depends on the subpatterns
of the pattern. We cannot simply divide the number of 8 × 8 patterns by a single integer to ac-
count for the duplication. We must sort the 8 × 8 patterns according to how much duplication
they produce.
Begin with any 8 × 8 pattern and extend it to its wallpaper. The other 8 × 8 patterns that
produce the same wallpaper are exactly the patterns obtained by placing an 8 × 8 window
over the wallpaper so that the upper left corner of the window is one of the light colored
squares on the original board. There are 32 possible 8 × 8 patterns obtained this way, but
some of these 32 may be duplicates. This duplication occurs precisely when the 8 × 8 board
comes from a wallpaper that has sub patterns. This duplication is what we must account for.
We show two different solutions. The first is a general system known as the Burnside method.
The second solution directly counts the number of different patterns.
Examine the possibilities, as follows. Consider any two squares in an 8 × 8 pattern and look
at the two new 8 × 8 patterns obtained by placing an 8 × 8 window so that each of the two
selected
squares is in the upper left corner. If the resulting
8 × 8 patterns are identical then we say the two squares
"match" for these boards and wallpapers. This means
the two squares are functionally the same for this pat-
tern. For instance, in the example shown here (the
one shown in Question 9) all of the squares with black
pawns match each other. All of the squares with white
pawns match each other. All of the squares just to the right of a white pawn match each other,
and so on. (It is patterns like this, the ones with subpatterns, that produce duplications.)
Again consider a particular 8 × 8 pattern. If exactly k squares match one of the squares in the
pattern, then exactly k squares match each of the other squares in the pattern. In the example,
k = 4. The 64 squares in the 8 × 8 pattern are partitioned into families of matching squares
with k members each. Thus k is a factor of 64. Because there are just four pawns of each color,
k cannot be more than four. We conclude that k = 1, 2, or 4. Call the number of squares in each
matching family the "symmetry number" of the 8 × 8 pattern. For the example, k = 4.
Matching squares are the same color, so each family is either light or dark, and there are 32 k
families of light squares and 32 k families of dark squares.
The families are constructed so that putting the upper left corner of the 8 × 8 window at one
square in the family always matches the 8 × 8 pattern that is obtained by putting the window
at another square of the family and never matches the pattern obtained by putting the upper
left corner of the window at a square from a different family. Each family produces one 8 × 8
pattern. Since the upper left corner is always light and since there are 32 k families of light
squares, there are exactly 32k different 8 × 8 patterns that generate the wallpaper pattern. For
32
the pattern in the example, there are 4 = 8 different 8 × 8 patterns that generate the wallpaper
pattern: put any of the light squares in the top two rows in the upper left corner. Each choice
makes a different 8 × 8 pattern and all will produce the same wallpaper pattern. If you put
any other square in the upper left corner, your 8 × 8 pattern will be the same as the pattern
you get by choosing the square in the top two rows that matches your square.
Rewrite the fact that there are 32 k different patterns for the wallpaper for a pattern with sym-
metry number k using multiplication instead of division,
[symmetry number] × [number of different 8 × 8 patterns that generate the wallpaper] = 32
The 8 × 8 patterns can be grouped by their associated wallpaper patterns. There is a set of 8 × 8
patterns for each wallpaper pattern. If you combine all the sets for all the distinct wallpaper
patterns you will get all the 8 × 8 patterns.
Here is the first nice trick: The product
[symmetry number] × [number of different 8 × 8 patterns that generate the wallpaper]
can be viewed as the sum of one copy of the symmetry number for each pattern that generates
the wall paper pattern. It is what you get if you add up all the symmetry numbers of all the
8 × 8 patterns for a single wallpaper pattern. That is, in symbols
[symmetry number] × [number of different 8 × 8 patterns that generate the wallpaper] = ∑ k
where the sum is taken over all the 8 × 8 patterns that generate the wallpaper pattern.
For a single wallpaper pattern the symmetry numbers are all the same. The sum is a compli-
cated way to write a simple quantity.
So far we have obtained that for a single wallpaper pattern
∑ k = 32
where the sum is taken over all the 8 × 8 patterns that generate the wallpaper and k is the
symmetry number of each 8 × 8 pattern.
Now add up all the symmetry numbers for all the 8 × 8 patterns - not just for one single
wallpaper, but for all the wallpaper patterns. That is, sum over all wallpaper patterns. On the
right hand side, you will get 32 times the number of different wallpaper patterns!!!!!!!!!!!! That
is, calling the number of different wallpaper patterns W,
∑ k = 32W.
where the sum is now taken over all of the different 8 × 8 patterns for all of the wallpaper
patterns and k is symmetry number.
The plan is to find the sum of all the symmetry numbers of all the patterns and divide by 32.
Remember that the symmetry number of a particular 8 × 8 pattern is the number of squares
in that pattern that "match" the square in the upper left corner (include the corner itself). The
sum of all the symmetry numbers is the sum of the number of squares that match the upper
left corner, taken over all 8 × 8 patterns. We need to count all the squares that match the upper
left corners in all the patterns. We could do this by counting those squares, 8 × 8 pattern by
8 × 8 pattern. Think of having a stack of cards, each with one of the patterns, and going
through them, one by one, counting the squares that match the upper left corner.
Here is the second nice trick: Turn this around and instead of counting pattern by pattern,
look square by square. For each square in the 8 × 8 chessboard, count the number of 8 × 8
patterns for which that square matches the upper left corner. That is, instead of going card by
card, go square by square. Pick a square and go through all the cards, counting the number of
cards for which this square matches the upper left corner. Go through the stack once for each
of the 64 squares. (This is similar to adding up all the entries in a matrix by either going row
by row or going column by column.)
It turns out that it is not hard to count square by square:
None of the dark squares ever match the upper left corner, so the contributions from those
squares are all zero. Now we must look at the 32 light squares.
No square in any even numbered column or even numbered row can match the upper left
corner. There are several ways to see this. Suppose the square diagonally below the upper
left corner, for instance, matches the corner. Then so also does the next one diagonally down.
And so on: there will have to be 8 matching squares. However, there are never more than
four matching squares. We conclude that the square diagonally below the upper left corner
will never match the corner. The reasoning is similar for all squares in even numbered rows
or columns.
There remain 16 squares to look at.
The upper left square matches itself for all 8 × 8 patterns, so the number for this square is
64!
56!4!4!
Suppose the third square in the top row matches the corner. Then so also do the fifth and
seventh. We find that the pattern must consist of repeats of the first two columns and that
there must be exactly one white pawn and one black pawn in those first two columns. Also,
any such pattern will make this third square in the top row match the corner. Choose one of
the 16 squares for the black pawn and one of the remaining 15 squares for the white pawn.
16!
There are 14! such patterns.
Similar analysis produces the numbers for the other squares. For instance, count how many
patterns have the fifth square in the top row matching the corner. In this case we find the
pattern consists of a repeat of the first four columns and that any pattern that consists of
repeats of its first four columns will do. There will be two pawns of each color in those first
four columns. Some of the patterns will have subpatterns in those first four columns, e.g. the
second pair of columns could be a repeat of the first pair. The beauty of this method is that
this does not matter here! We have 32 squares in which to place 2 black pawns and 2 white
pawns. Choose 2 squares out of 32 and then 2 more out of the remaining 30. The number of
32!
patterns is 28!2!2!
64!
Continuing, we obtain that the upper left corner square matches itself for all 56!4!4! patterns,
three of the squares (the one in the fifth column and first row, the one in the fifth row and first
32!
column, and the one in the fifth row and fifth column) match the corner for 28!2!2! patterns,
16!
and the remaining twelve squares each match the corner for 14! patterns. Summarizing, the
count over all the 8 × 8 patterns of all the squares matching the upper left corner is
64! 32! 16!
+ 3( ) + 12( ).
56!!4!4! 28!2!2! 14!1!1!

Divide by 32 to get the number of different wallpaper patterns.

A second way to solve the problem: Direct enumeration


Follow the first solution until you come to the "first nice trick." Instead of using the trick, we
will attempt to enumerate the number of different patterns directly by counting the number
of patterns with each symmetry number.
Denote by Wk the number of different wallpaper patterns that have symmetry number k
and denote by Bk the number of different 8 × 8 patterns that have symmetry number k.
The statement above says that 32 k × Wk = Bk . The thing we have been asked to count is
W = W1 + W2 + W4 , the total number of distinct wallpaper patterns. To obtain W, we can
1 1
count the Bk and compute W = ( 32 × B1 ) + ( 16 × B2 ) + ( 81 × B4 )
First we find B4 , the number of distinct 8 × 8 patterns with symmetry number four. For each
such pattern, the 64 squares fall into 16 families of 4 matching squares. From each family select
the square that is in the highest row. If there are several in the same row, then take the one of
those that is farthest left. This will partition the 8 × 8 pattern into four blocks of 16 squares.
Each block will contain one black and one white pawn. The block containing the upper left
square can be either the top two rows, the left two columns, or a 4x4 square. The pawns can
occupy any two squares in the block, so there are 16 × 15 different possible blocks of each
shape. Beginning with a two row block, one pattern results from repeating the block four
times down the 8 × 8 pattern. Another results from cloning the first two rows and shifting
them right two squares for the second two rows, to the right another two squares for the third
pair of rows, and right another two squares for the last pair of rows. A third pattern results
from cloning and shifting right four squares each time. A fourth pattern results from cloning
and shifting right six squares. After that, the we come back to the original cloning with no
shift. We conclude that four different 8 × 8 patterns can be obtained from a single choice of
two rows that contains one pawn of each color. We get 4 × 16 × 15 patterns. Beginning with
a square block, we obtain patterns in three ways. Cloning and tiling the 8 × 8 pattern with
four of the clones gives one pattern. Clone the square block and put one copy in the upper
left corner and one in the lower left corner. Put the other two in the right side of the square,
but offset them so that they are shifted down two squares relative to the left side. This turns
out to produce a pattern that will have as its assigned block its top two rows, so we do not
obtain a new pattern. Lastly, we can put two of the 4x4 squares in the top half of the 8 × 8
pattern and put the other two below, offset by two squares. We find that each of the 16 × 15
different 4x4 blocks produces two new 8 × 8 patterns. Beginning with a two column block,
one pattern results from repeating the block four times across the 8 × 8 pattern. Another
results from cloning the first two columns and shifting them down two squares for the second
two columns, down another two squares for the third pair of columns, and down another
two squares for the last pair of columns. A third pattern results from cloning and shifting
down four squares each time. A fourth pattern results from cloning and shifting down six
squares. After that, the we come back to the original cloning with no shift. The second and
fourth possibilities turn out to have already been counted among the patterns with the top
two rows for blocks. The third possibility was already counted with the patterns whose block
is 4x4. Thus only one of the ways to use the two column block produces new patterns. We
conclude that in all there are 7 × 16 × 15 different 8 × 8 blocks with symmetry number 4.
4
B4 = 7 × 16 × 15 and W4 = 32 × B4 = 18 × B4 = 81 × 7 × 16 × 15.
Now we find B2 , the number of distinct 8 × 8 patterns with symmetry number two. This time,
the 64 squares fall into 32 families of 2 matching squares. Choose one square from each family
just as in the case above. This time we obtain two possible shapes for the blocks and each block
contains two pawns of each color. This time the block that contains the upper left corner can
be either the top four rows or the left four columns. The pawns can occupy any four squares
in the block, so there are (32 30
2 ) × ( 2 ) different ways to distribute the pawns in each block. In
the case of B2 , however, not all of the choices for filling the 32 squares produce a pattern with
symmetry number 2. Blocks that have a subsymmetry will produce patterns with symmetry
number 4. Reasoning as we did for finding B4 , we find that if there is a subpattern, then
the pattern is made in one of three ways. For the case of the block with four rows and eight
columns either the right half of the rectangle is a clone of the left half, the bottom half of the
rectangle is a clone of the top half, or the bottom half of the rectangle is the same as the top
half shifted half way across the rectangle. The structure is analogous for the rectangular block
with eight rows and four columns. In all these cases, we can obtain a pattern by placing one
pawn of each color in a single one of the 16-square halves of the pattern. Thus the number of
such blocks that produce patterns with symmetry number 4 instead of 2 is 3 × 16 × 15. We
conclude that there are (32 30
2 ) × ( 2 ) − 3 × 16 × 15 useful basic blocks of 32 that are four rows
by eight columns and the same number that are eight columns by four rows. There are two
different 8 × 8 patterns that can be produced from each of these basic blocks- either clone the
basic block to the other half of the 8 × 8 square or clone it and shift it half way. Cloning a
vertical one and shifting half way will also be obtained by cloning a horizontal pattern and
shifting half way. Thus, B2 = 3 × ((32 30
2 ) × ( 2 ) − 3 × 16 × 15)
and W2 = 32 2 1
× B2 = 16 × 3 × ((32 30
2 ) × ( 2 ) − 3 × 16 × 15).
Finding B1 is now easy, because each 8 × 8 pattern that does not have symmetry number 2 or
4 must have symmetry number 1. That is,
B1 = total number of 8 × 8 patterns (computed in problem 9) − B2 − B4
= 64!
56!4!4! − 3 × ((32 30
2 ) × ( 2 ) − 3 × 16 × 15) − 7 × 16 × 15
1
and W1 = 32 B1
Now we compute
1 1
W = ( 32 × B1 ) + ( 16 × B2 ) + ( 81 × B4 )
    1
1 64! 32 30 1 32 30
= 32 56!4!4! − 3 (( 2 )( 2 ) − 3 × 16 × 15 ) − 7 × 16 × 15 + 16 3 ( 2 )( 2 ) − 3 × 16 × 15 + 8 (7 ×
16 × 15)
For neatness, we can write 16 × 15 as (16 15 64! 64 60
1 )( 1 ) and 56!4!4! as ( 4 )( 4 ). This gives
h   i h  i h i
W = 321
(64
4 )( 60
4 ) − 3 ( 32 30
2 )( 2 ) − 3 ( 16 15
1 )( 1 ) − 7 ( 16 15
1 )( 1 ) + 1
16 3 ( 32 30
2 )( 2 ) − 3 ( 16 15
1 )( 1 ) + 1
8 7 ( 16 15
1 )( 1 )

END OF CONTEST

Anda mungkin juga menyukai